Integrals UntilModule24Only

You might also like

Download as pdf or txt
Download as pdf or txt
You are on page 1of 206

Prepared by: Nathaniel M.

Cabansay, BSCpE

INTEGRALS
FIRST 24 MODULES
Prerequisite: Limits (up to Module 4 at least) & Derivatives (up to Module 15 at least)

Prepared by: Nathaniel M. Cabansay

BSCpE-1 (1st Year BS Computer Engineering)

Cebu Institute of Technology-University


Prepared by: Nathaniel M. Cabansay, BSCpE

LICENSING

NON-COMMERCIAL

This set of modules is not for sale and is freely distributed. It is also used for educational purposes only.

LICENSING

This work is licensed under the Creative Commons Attribution-NonCommercial-ShareAlike 4.0 International
License. To view a copy of this license, visit http://creativecommons.org/licenses/by-nc-sa/4.0/.
Prepared by: Nathaniel M. Cabansay, BSCpE

TABLE OF CONTENTS

Licensing .................................................................................................................................................................................................... 2
Non-Commercial................................................................................................................................................................................ 2

Licensing ................................................................................................................................................................................................ 2
Chapter 1 Introduction to Integrals................................................................................................................................................... 1

Module 1 Review of Derivatives and Differentials ..................................................................................................................... 1


Review of Derivative Rules ......................................................................................................................................................... 1

Differentials ..................................................................................................................................................................................... 2

Examples of Differentials ............................................................................................................................................................. 3


Applying Differentials .................................................................................................................................................................... 3

Module 2 Antiderivatives.................................................................................................................................................................. 4
Reversing the Process .................................................................................................................................................................. 4

What Is An Antiderivative ........................................................................................................................................................... 4

Changes of Variables ..................................................................................................................................................................... 4

Why Do We Need Integrals? ..................................................................................................................................................... 4

One Final Important Notice ....................................................................................................................................................... 4


Chapter 2 Indefinite Integrals .............................................................................................................................................................. 5

Module 3 Basic Antiderivatives ....................................................................................................................................................... 5


What are Integral Rules? .............................................................................................................................................................. 5

Basic Integral Rules and Integration Techniques ................................................................................................................... 5

Deriving Basic Integral Rules ....................................................................................................................................................... 5

Deriving Basic Integration Techniques ..................................................................................................................................... 7

Examples of Using Basic Integral Rules .................................................................................................................................... 8

Examples of Integration By Substitution ................................................................................................................................. 9

Example of Integration by Parts...............................................................................................................................................10


Module 4 Integrals of Exponential and Logarithmic Functions............................................................................................. 13

Integrals with Logarithmic Results ..........................................................................................................................................13

Deriving Integrals with Logarithmic Results .........................................................................................................................13

Examples of Integrals with Logarithmic Results ..................................................................................................................14

Integrals of Logarithmic and Exponential Functions ...........................................................................................................14

Deriving Integrals of Logarithmic and Exponential Functions ..........................................................................................15

Examples of Integrals of Logarithmic and Exponential Functions ...................................................................................16


Prepared by: Nathaniel M. Cabansay, BSCpE

Module 5 Integrals of Trigonometric Functions .......................................................................................................................18


Integrals of the Two Basic Trigonometric Functions .........................................................................................................18

Deriving Integrals of the Two Basic Trigonometric Functions ........................................................................................18


Examples of Integrals of the Two Basic Trigonometric Functions .................................................................................18

Integrals with Trigonometric Results .....................................................................................................................................21

Deriving Integrals with Trigonometric Results ....................................................................................................................21

Examples of Integrals with Trigonometric Results .............................................................................................................22

Integrals of Remaining Four Trigonometric Functions ......................................................................................................22


Tangent Half-Angle Substitutions ............................................................................................................................................23

Deriving Integrals of Remaining Four Trigonometric Functions .....................................................................................23


Examples of Integrals of Trigonometric Functions .............................................................................................................28

Module 6 Integrals of Inverse Trigonometric Functions ........................................................................................................30

Integrals with Inverse Trigonometric Results ......................................................................................................................30


Deriving Integrals with Inverse Trigonometric Results .....................................................................................................30

Examples of Integrals with Inverse Trigonometric Results ..............................................................................................32


Integrals of Inverse Trigonometric Functions ......................................................................................................................33

Deriving Integrals of Inverse Trigonometric Functions .....................................................................................................33


Examples of Integrals of Inverse Trigonometric Functions ..............................................................................................39

Module 7 Integrals of Hyperbolic Functions .............................................................................................................................41


Integrals of the First Two Hyperbolic Functions.................................................................................................................41

Deriving Integrals of the First Two Hyperbolic Functions ............................................................................................... 41

Examples of Integrals of the First Two Hyperbolic Functions.........................................................................................42


Integrals with Hyperbolic Results ............................................................................................................................................44

Deriving Integrals with Hyperbolic Results ...........................................................................................................................44


Examples of Integrals with Hyperbolic Results ....................................................................................................................46

Integrals of Remaining Four Hyperbolic Functions .............................................................................................................46

Hyperbolic Tangent Half-Angle Substitutions ......................................................................................................................46

Deriving Integrals of Remaining Four Hyperbolic Functions ............................................................................................47

Examples of Integrals of Hyperbolic Functions ....................................................................................................................51


Module 8 Integrals of Inverse Hyperbolic Functions...............................................................................................................52

Integrals with Inverse Hyperbolic Results .............................................................................................................................52


Deriving Integrals with Inverse Hyperbolic Results ............................................................................................................52
Prepared by: Nathaniel M. Cabansay, BSCpE

Examples of Integrals with Inverse Hyperbolic Results .....................................................................................................57


Integrals of Inverse Hyperbolic Functions .............................................................................................................................58

Deriving Integrals of Inverse Hyperbolic Functions ............................................................................................................58


Examples of Integrals of Inverse Hyperbolic Functions .....................................................................................................61

Chapter 3 Definite Integrals ...............................................................................................................................................................62

Module 9 Riemann Sums ................................................................................................................................................................62

Reviewing Summation Notation ..............................................................................................................................................62

The Area Problem .......................................................................................................................................................................62


Negative and Net Areas.............................................................................................................................................................64

Examples of Riemann Sums.......................................................................................................................................................64


Module 10 Definite Integrals and the Fundamental Theorem of Calculus ........................................................................67

What Are Definite Integrals .....................................................................................................................................................67

Properties of Summations and Some Convenient Formulas ...........................................................................................67


Properties of Definite Integrals ................................................................................................................................................68

Deriving the Properties of Integrals .......................................................................................................................................68


Examples of Using Properties of Definite Integrals ............................................................................................................73

Fundamental Theorem of Calculus .........................................................................................................................................74


Proving the Fundamental Theorem of Calculus ..................................................................................................................74

Using The Fundamental Theorem of Calculus .....................................................................................................................76


Chapter 4 Integration Techniques....................................................................................................................................................78

Module 11 More on Integration by Substitution .....................................................................................................................78

Substitution Rule for Indefinite Integrals ...............................................................................................................................78


More Examples of Integration by Substitution for Indefinite Integrals..........................................................................78

Substitution Rule for Definite Integrals .................................................................................................................................79


Examples of Substitution Rule for Definite Integrals .........................................................................................................80

Module 12 More on Integration by Parts...................................................................................................................................82

Integration by Parts .....................................................................................................................................................................82

Examples of Integration by Parts .............................................................................................................................................82

Tabular Method of Integration By Parts ................................................................................................................................83


Examples of Using Tabular Method of Integration By Parts ............................................................................................84

Module 13 Trigonometric Transformations ..............................................................................................................................86


Important Trigonometric Identities ........................................................................................................................................86
Prepared by: Nathaniel M. Cabansay, BSCpE

Integrals Involving Products of Sine and Cosine With Different Arguments ..............................................................86
Examples of Integrals Involving Products of Sine and Cosine With Different Arguments ......................................87

Integrals Involving Powers of Sine and Cosine.....................................................................................................................89


Examples of Integrals Involving Powers of Sine and Cosine.............................................................................................90

Wallis’ Formula.............................................................................................................................................................................93

Examples of Wallis’ Formula .....................................................................................................................................................94

Integrals Involving Powers of the Other Trigonometric Functions................................................................................96

Examples of Integrals Involving Powers of Other Trigonometric Functions ...............................................................98


Deriving Reduction Formulas for Secant and Cosecant ................................................................................................. 101

Module 14 Integration by Trigonometric Substitutions ...................................................................................................... 104


Why The Use of Triangles? .................................................................................................................................................... 104

Examples of Trigonometric Substitutions .......................................................................................................................... 104

Using Trigonometric Substitutions To Find General Formulas .................................................................................... 106


Module 15 Integration by Completing the Square and Partial Fractions ........................................................................ 116

Dealing With Rational Functions .......................................................................................................................................... 116


Integration by Completing the Square ................................................................................................................................ 116

Examples of Integration by Completing the Square ........................................................................................................ 117


Partial Fraction Decomposition ............................................................................................................................................ 118

Examples of Partial Fraction Decompositions .................................................................................................................. 118


Examples of Integration By Partial Fraction Decomposition ........................................................................................ 120

Using Partial Fractions To Find General Formulas .......................................................................................................... 122

Module 16 Integration by Algebraic Substitution ................................................................................................................. 125


Algebraic Substitution ............................................................................................................................................................. 125

Examples of Integration by Algebraic Substitution .......................................................................................................... 125


Module 17 Improper Integrals ................................................................................................................................................... 130

Improper Integrals .................................................................................................................................................................... 130

Convergent and Divergent Integrals .................................................................................................................................... 130

Cases Where At Least One Limit of Integration is Infinity ........................................................................................... 130

Cases Where Integrands are Discontinuous ..................................................................................................................... 130


Examples of Improper Integrals ............................................................................................................................................ 131

Module 18 Numeric Integration ................................................................................................................................................ 135


Approximating Definite Integrals ......................................................................................................................................... 135
Prepared by: Nathaniel M. Cabansay, BSCpE

The Midpoint Rule .................................................................................................................................................................... 135


The Trapezoidal Rule ............................................................................................................................................................... 135

Simpson’s Rule ........................................................................................................................................................................... 136


Series Forms of Approximation Rules ................................................................................................................................ 138

Examples of Approximating Definite Integrals ................................................................................................................. 138

Chapter 5 Applications of Integrals............................................................................................................................................... 141

Module 19 Average Function Value .......................................................................................................................................... 141

Average Value ............................................................................................................................................................................ 141


The Mean Value Theorem for Integrals .............................................................................................................................. 142

Root Mean Square Value (RMS) Value of a Function ...................................................................................................... 142


Examples of Finding the Average Function Value............................................................................................................. 142

Module 20 Area Between Curves ............................................................................................................................................. 146

Area Between Curves ............................................................................................................................................................. 146


Examples of Areas Between Curves.................................................................................................................................... 147

Deriving Area Formulas .......................................................................................................................................................... 151


Module 21 Multiple Integrals ...................................................................................................................................................... 155

Dealing with Multiple Variables ............................................................................................................................................. 155


Examples of Multiple Integrals ............................................................................................................................................... 155

Module 22 Solids of Revolution................................................................................................................................................. 158


Volumes by Revolving Part 1: Disks .................................................................................................................................... 158

Examples of Solids of Revolution: Disk Method .............................................................................................................. 158

Volumes by Revolving Part 2: Washers .............................................................................................................................. 162


Examples of Solids of Revolution: Washers Method ...................................................................................................... 162

Volumes by Revolving Part 3: Shells .................................................................................................................................... 166


Examples of Solids of Revolution: Shells Method ............................................................................................................ 166

Volumes by Revolving Part 4: Other Cross-Sectional Areas ........................................................................................ 168

Examples of Solids of Revolution: Other Cross-Sectional Areas ................................................................................ 168

Deriving Volume Formulas ..................................................................................................................................................... 169

Module 23 Arc Length ................................................................................................................................................................. 178


Arc Length .................................................................................................................................................................................. 178

Examples of Arc Length .......................................................................................................................................................... 179


Module 24 Surfaces of Revolution ............................................................................................................................................ 183
Prepared by: Nathaniel M. Cabansay, BSCpE

Surfaces by Revolving: Circles ............................................................................................................................................... 183


Examples of Surfaces of Revolution ..................................................................................................................................... 184

Deriving Lateral Surface Area and Total Surface Area Formulas ................................................................................. 185
References ................................................................................................................................................................................................ 1

Extras ......................................................................................................................................................................................................... 6

About This Set of Modules .............................................................................................................................................................. 6

Some Quotes to Live By .................................................................................................................................................................. 6


Prepared by: Nathaniel M. Cabansay, BSCpE

CHAPTER 1 INTRODUCTION TO INTEGRALS

MODULE 1 REVIEW OF DERIVATIVES AND DIFFERENTIALS

REVIEW OF DERIVATIVE RULES

Here are the derivative rules we’ve previously studied:

Basic Rules (all found in Module 3 of Derivatives)

1. = 0 (Constant Rule)
2. = (Power Rule)
a. = 1 (Derivative of x)
3. = (Constant Multiple Rule)
4. ± = ± (Sum/Difference Rule)
5. ⋅ = + (Product Rule)

6. = , ≠ 0 (Quotient Rule)

a. = , ≠ 0 (Reciprocal Rule)

7. = ⋅ (Chain Rule)
8. = ! (Derivative of an Inverse Function)

Derivatives of Trigonometric Functions (all found in Module 4 of Derivatives)

1. sin % = cos % )% (Sine Function)


2. cos % = − sin % )% (Cosine Function)
3. tan % = sec. % )% (Tangent Function)
4. csc % = − csc % cot % )% (Cotangent Function)
5. sec % = sec % tan % )% (Secant Function)
6. cot % = − csc . % )% (Cotangent Function)

Derivatives of Exponential and Logarithmic Functions (all found in Module 5 of Derivatives)

1. e/ = e/ )% (Exponential Function, base e)


2. 0/ = 0/ ln 0 )% (Exponential Function, any base)
ln % = )% (Natural Logarithm)
/
3.
4. log 3 % = / 45 3 )% (Logarithm, any base)

Derivatives of Inverse Trigonometric Functions (all found in Module 6 of Derivatives)

arcsin % = )% (Arcsine)
7 /
1.

1
Prepared by: Nathaniel M. Cabansay, BSCpE

arccos % = − )% (Arccosine)
7 /
2.

3. arctan % = 8/
)% (Arctangent)
arccsc % = − )% (Arccosecant)
|/|7/
4.

arcsec % = )% (Arcsecant)
|/|7/
5.

arccot % = − )% (Arccotangent)
8/
6.

Derivatives of Hyperbolic Functions (all found in Module 7 of Derivatives)

1. sinh % = cosh % (Hyperbolic Sine)


2. cosh % = sinh % (Hyperbolic Cosine)
3. tanh % = sech. % (Hyperbolic Tangent)
4. csch % = − csch % coth % )% (Hyperbolic Cosecant)
5. sech % = − sech % tanh % )% (Hyperbolic Secant)
6. coth % = − csch. % )% (Hyperbolic Cotangent)

Derivatives of Inverse Hyperbolic Functions (all found in Module 8 of Derivatives)

arsinh % = )% (Inverse Hyperbolic Sine)


7/ 8
1.

arcosh % = )%, % > 1 (Inverse Hyperbolic Cosine)


7/
2.

artanh % = )%, |%| < 1 (Inverse Hyperbolic Tangent)


/
3.
arcsch % = − )%, % ≠ 0 (Inverse Hyperbolic Cosecant)
|/|7/ 8
4.

arsech % = − )%, 0 < % < 1 (Inverse Hyperbolic Secant)


/7 /
5.

arcoth % = − )%, |%| > 1 (Inverse Hyperbolic Cotangent)


/
6.

Derivatives of Other Functions (first one found in Module 3 and last two found in Module 14 of Derivatives)

|%| = )%, % ≠ 0 (Absolute Value Function)


/
|/|
1.

2. %/ = % / 1 + ln % )%, % ≠ 0 (A Function Raised to Itself)


3. %= = >% = )% + % = ln % )> (A Function Raised to Another Function)

DIFFERENTIALS

Here, we will introduce a new notation of derivatives. This notation will be quite useful in the succeeding modules.

Given a function ? = , we call )? and ) differentials, and their relationship is given by:

)? = )

Note that if we are given just , the differentials are ) and ) and are computed in the same manner:
) = )

2
Prepared by: Nathaniel M. Cabansay, BSCpE

EXAMPLES OF DIFFERENTIALS

Compute the differential for ? = @


−3 .
+3 −1

No. STATEMENT REASON

1 ?= @
−3 .
+3 −1 Given
)?
=3 .
−6 +3
)
2 Get the derivatives of both sides

3 CD = EFG − HF + E CF Multiply both sides by )

=e
J
Compute the differential for

No. STATEMENT REASON

=e
J
1 Given
)
=4 e
@ J

)
2 Get the derivatives of both sides

3 C L F = MNFE OF
N P
Q CF Multiply both sides by )

APPLYING DIFFERENTIALS

If we think of Δ as the change in in Δ? = +Δ − is the change in ? corresponding to the change in


. If Δ is small, we can assume that Δ? ≈ )? . For example:

Compute )? and Δ? if ? = @
−3 .
+ 3 − 1 as changes from = 2 to = 2.04.

No. STATEMENT REASON

1 ?= @
−3 .
+3 −1 Given
? = 2.04 @ − 3 2.04 .
+ 3 2.04 − 1 = 2.04 and = 2 and get the difference
? = 1.124864
2 Substitute

?= 2 @
−3 2 +3 2 −1
?=1
Δ? = 1.124864 − 1 = 0.124864
3 )? = 3 .
−6 +3 ) Get the derivative and compute the differentials

)? = 3 2 . − 6 2 + 3 0.04 = 2 and ) = 0.04


)? = 3 0.04 = 0.12
4 Substitute

Δ? ≈ )? Since )? and Δ? are close in value



5

3
Prepared by: Nathaniel M. Cabansay, BSCpE

MODULE 2 ANTIDERIVATIVES

REVERSING THE PROCESS

Suppose we are given a derivative . Here, we want to find what function we differentiated so we could
get this derivative. This process is called antidifferentiation or integration.

WHAT IS AN ANTIDERIVATIVE

If we have a function , an antiderivative of is any function W such that W = .

If W is an antiderivative of , then the most general antiderivative is called an indefinite integral and is
denoted:

X ) =W +Y

In the above, C is the constant of integration, as the derivative of a constant is zero. The symbol ∫ is called the
integral symbol, is the integrand, is the variable of integration, W is the particular integral, and W +Y
is the indefinite integral. The differentiation notation introduced in the previous module is useful here as this
denotes that the function to be integrated is the derivative of a certain other function.

A second type of integral is the definite integral and is denoted:


[

X ) = W [
3 = W \ −W 0
3

In the above, 0 is the lower limit, and \ is the upper limit. This will be discussed in Chapter 3.

CHANGES OF VARIABLES

Note that need not be the variable of integration. It could be ], ^, _, ?, `, or other letters. If you change the
variable, however, you must change the variable written after ) . For example:

X ] )] = W ] + Y

The integral is with respect to t, and any other variables are usually treated as constants.

WHY DO WE NEED INTEGRALS?


Integrals of many types can help us find volume, area, length, distance travelled, population of a certain locale, and
many more.

ONE FINAL IMPORTANT NOTICE

Most books which discuss integrals include a table of integrals somewhere at the end of the book. We won’t be
doing that here as the methods described in this set of modules are enough to figure them out.

4
Prepared by: Nathaniel M. Cabansay, BSCpE

CHAPTER 2 INDEFINITE INTEGRALS

MODULE 3 BASIC ANTIDERIVATIVES

WHAT ARE INTEGRAL RULES?

Integral rules allow getting integrals of functions, and are mostly reverses of derivative rules.

BASIC INTEGRAL RULES AND INTEGRATION TECHNIQUES

be a constant, Y (uppercase) be the constant of integration, be any real power, , , % and > be functions
of , and W be the antiderivative of :
Let

1. ∫ a CF = b (Integral of Zero; Integral Rule 1 from this point on)


∫ Fc CF = + b, c ≠ −P (Reverse Power Rule; Integral Rule 2 from this point on)
FcdP
c8P
a. ∫ P CF = F + b (Integral of One; Integral Rule 2-bis from this point on)
2.

∫ eL F CF = e ∫ L F CF (Constant Multiple Rule; Integral Rule 3 from this point on)


a. ∫ e CF = eF + b (Integral of a Constant; Integral Rule 3-bis from this point on)
3.

4. ∫ L F ± f F CF = ∫ L F CF ± ∫ f F CF (Sum/Difference Rule for Integrals; Integral Rule 4 from this

∫ L f F f F CF = ∫ L f Cf = g f + b (Integration by Substitution)
point on)

∫ h Ci = hi − ∫ i Ch (Integration by Parts)
5.

∫L f Cf = fL f − gML f Q + b (Integral of an Inverse Function [combined with Integration


6.
P P P
7.
by Substitution])

DERIVING BASIC INTEGRAL RULES

RULE 1: INTEGRAL OF ZERO

This is a reverse of the derivative of the constant

)
No. STATEMENT REASON

0) = Y
)
1 Derivative Rule 1: Constant Rule

X a CF = b
2 Integrate both sides with respect to x

RULE 2: REVERSE POWER RULE

This is the reverse of the power rule for derivatives

)
No. STATEMENT REASON

) =
)
1 Derivative Rule 2: Power Rule

5
Prepared by: Nathaniel M. Cabansay, BSCpE

)
+1 ) = 8
)
2 Add 1 to all instances of n

) 8
+1
) = j k
) +1
3 Divide both sides by

Fc8P
X Fc CF = + b, c ≠ −P
c+P
4 Integrate both sides with respect to x and add a


constant of integration.

We then derive its special case, the integral of 1.

)
No. STATEMENT REASON

1) =
)
1 Derivative Rule 2-bis: Derivative of x

X P CF = F + b
2 Integrate both sides with respect to x and add a


constant of integration.

For the case of = −1, this will be dealt with later, as the denominator would be 0 if we used the power rule.

RULE 3: CONSTANT MULTIPLE RULE

This is the reverse of the Constant Multiple Rule for derivatives.

)
No. STATEMENT REASON

) =
)
1 Derivative Rule 3: Constant Multiple Rule

X ) =
2 Integrate both sides with respect to x

X eL F CF = e X L F CF
3 Integrate on both sides


This shows that like limits and derivatives, we can pull constants out of integrals. We then derive its special case,
the integral of constant.

)
No. STATEMENT REASON

1) =
)
1 Derivative Rule 2-bis: Derivative of x

)
) =
)
2 Multiply both sides by c

X e CF = eF + b
3 Integrate both sides with respect to x and add a


constant of integration.

RULE 4: SUM/DIFFERENCE RULE

This is the reverse of the Sum/Difference Rule for Derivatives.

6
Prepared by: Nathaniel M. Cabansay, BSCpE

)
No. STATEMENT REASON

± ) = ±
)
1 Derivative Rule 4: Sum/Difference Rule

X ± ) = ±
2 Integrate both sides with respect to x

X L F ± f F CF = X L F CF ± X f F CF
3 Integrate both functions separately


As with limits and derivatives, we can separately integrate functions separated by plus or minus signs.

DERIVING BASIC INTEGRATION TECHNIQUES

INTEGRATION BY SUBSTITUTION

This is the reverse of the chain rule.

)
No. STATEMENT REASON

l m) =
)
1 Derivative Rule 7: Chain Rule

X ) = +Y
2 Integrate both sides with respect to x

Let W be an antiderivative of . Integrate


X L f F f F CF = g f F +b
3 on both


sides

) so we can let % =
In integrating by substitution, we first check if we have a function that has its derivative with it since we are

% )% to make the integration easier.


supposed to have the given function in the form and rewrite the
function as

This technique will be combined with other integration rules starting in the next modules.

INTEGRATION BY PARTS

This is the reverse of the Product Rule for Derivatives.

) Given. Let % and > be functions of and )% and )> be


No. STATEMENT REASON

% )> + > )% ) = %>


)
1
their respective derivatives with respect to x

X % )> + > )% = %>


2 Integrate both sides with respect to x

X % )> + X > )% = %>


3 Integral Rule 4

Subtract ∫ > )% from both sides


X h Ci = hi − X i Ch
4

7
Prepared by: Nathaniel M. Cabansay, BSCpE

We can integrate by parts if we are given a function with one part that is easy to differentiate but not to integrate
[or a function where we do not know the integral] and another function unrelated to the other one (i.e., not a
constant multiple of its own derivative) easier to integrate than to differentiate [usually because the derivative is
more complicated]. Depending on the function we use or how we set up the functions, we CAN get different
results from integration by parts, but the difference will only be a constant. In short:

If = , then = +Y

We proved the above rule in Module 10 of Derivatives.

Logarithmic (ln , log. ] etc., inverse hyperbolic functions are included here), Inverse Trigonometric
A general rule of thumb for choosing which function should be differentiated is whatever comes first in LIATE:

(arcsin , arccos etc.), Algebraic ( @


, √] etc.), Trigonometric (sin , cos etc.), and Exponential (e , 10@ , etc.,
hyperbolic functions are included here).

INTEGRAL OF INVERSE FUNCTIONS

This is one of the exceptions to an integral rule not being a reverse of a derivative rule. This instead comes from
integration by substitution.

No. STATEMENT REASON

X )
1 Integration by Substitution

= ? and ) = ? )?. Then ? =


X? ? )?
? =?
2 Let and

Integrate by parts. Let % = ? and )> = ? )? . This


X? ? )? = ? ? −X ? )? + Y
means )% = )? and > = ?
3

X? ? )? = ? ? −W ? +Y
4 Integration by substitution

?= ? )? = )
XL P
f Cf = f L P
f − gML P
f Q+b
5 and


We will be seeing examples of this integration rule in Module 6

EXAMPLES OF USING BASIC INTEGRAL RULES

Find the indefinite integral of:

=4 @
+3 .
+2 +1

=4 +3 +2 +1
No. STATEMENT REASON
@ .
1 Given
2 X ) = X4 @
+3 .
+2 +1) Integrate both sides with respect to

8
Prepared by: Nathaniel M. Cabansay, BSCpE

3 X ) = X4 @
) + X3 .
) + X2 ) + X1 ) Integral Rule 4: Sum/Difference Rule

X ) = 4X @
) + 3X .
) +2X ) + X1)
4 Integral Rule 3: Constant Multiple
Rule
@8 .8 8 q8
X ) = 4j k+ 3j k+ 2j k+ +Y
3+1 2+1 1+1 0+1
5 Integral Rule 2: Reverse Power Rule

r @ .
X ) = 4j k +3j k+ 2j k+ +Y
4 3 2 1
6 Combine like terms

J s

X ) = r
+ @
+ .
+ +Y r @ .
7 Multiply by 4, by 3, by 2, and
!
simplify

Find the indefinite integral of:

] = 3] + 5 .

We can expand the function.

] = 3] + 5
No. STATEMENT REASON
.

] = 9] . + 30] + 25 Expand 3] + 5
1 Given
.
2

X ] )] = X 9] . + 30] + 25 )]
3 Integrate both sides with respect to t

X ] )] = X 9] . )] + X 30] )] + X 25 )]
4 Integral Rule 4

X ] )] = 9 X ] . )] + 30 X ] )] + 25 X 1 )]
5 Integral Rule 3

]@ ].
X ] )] = 9 j k + 30 j k + 25] + Y
3 2
6 Integral Rule 2

vs v
X ] )] = 3] @ + 15] . + 25] + Y @ .
7 Multiply by 9 and by 30

EXAMPLES OF INTEGRATION BY SUBSTITUTION

Find the indefinite integral of:

X 10 75 +3)
s
.

No. STATEMENT REASON

X 10 75 +3)
1 s . Given

9
Prepared by: Nathaniel M. Cabansay, BSCpE

2 X s√% )% Let % = 5 .
+ 3 and )% = 10 )

X %@ )%
3 Rewrite the radical as an exponent

%@
r
Integrate with respect to % using Integral Rule 2
X %@ )% = +Y
4
4

3
(Reverse Power Rule) and add a constant of integration.

r
3% @
X %@ )% = +Y
4
r
3 5 .
+3 @
X 10 75 +3) = +Y
5 Substitute back the original function
s .
4

We can also integrate ] = 3] + 5 .


by substitution.

] = 3] + 5
No. STATEMENT REASON
.
1 Given
2 X ] )] = X 3] + 5 .
)] Integrate both sides with respect to t

1 Let % = 3] + 5, )% = 3 )], so )% = )]. Use Integral


X ] )] = X %. )%
3
@
3

1 %@
Rule 3

X ] )] = j k+Y
3 3
4 Integral Rule 2

%@ /s
X ] )] = +Y
9
@ @
5 Multiply by

3] + 5 @
Substitute % = 3] + 5
X ] )] = +Y
9
6

27] @ + 135] . + 225] + 125 Expand 3] + 5 @


X ] )] = +Y
9
7

125
X ] )] = 3] @ + 15] . + 25] + +Y
9
8 Divide all terms in the numerator by 9

is a constant, and can be incorporated into Y


.x
X ] )] = 3] @ + 15] . + 25] + Y y
9

EXAMPLE OF INTEGRATION BY PARTS

Integrate:

= √ +1

We can go about this in two ways:

We can integrate by substitution…

10
Prepared by: Nathaniel M. Cabansay, BSCpE

No. STATEMENT REASON

1 = √ +1 Given

X ) = X √ +1)
2 Integrate both sides with respect to

3 X ) = X % − 1 √% )% Substitute % = + 1 and )% = )

4
X ) = X %. − %. )%
@
Multiply % − 1 by √%

@
X ) = X % . )% − X % . )%
5 Integral Rule 4 (Sum/Difference Rule)

2 x 2 @
X ) = % . − %. + Y
5 3
6 Integral Rule 2

2 x 2 @ Substitute back % = +1
X ) = +1 . − +1 . +Y
5 3
7

…or integrate by parts.

No. STATEMENT REASON

1 = √ +1 Given

X ) = X √ +1)
2 Integrate both sides with respect to

2 @ 2 @
Integrate by parts: % = and )> = √ + 1 ) . This
X ) = +1 . − X +1 . )
3 3
means )% = ) , and > = +1
3
. s

@
after integrating

2 2
the latter by substitution.
@ @
X ) = +1 . − X % . )%
3 3 integration by substitution, with % = + 1 and )% =
4 Evaluate the integral on the right-hand side using

)
2 @ 2 2 x
X ) = +1 . − z %.{ + Y
3 3 5
5 Integral Rule 2

2 @ 4 x Multiply and and substitute back % =


. .
+1
X ) = +1 . − +1 . +Y @ x
3 15
6

We used different methods, and got different answers. Again, remember that these two results will differ by no
more than a constant.

We can check the difference between these two:

2 4 2 2
No. STATEMENT REASON
@ x x @
=| +1 . − + 1 .} − | +1 . − + 1 .}
3 15 5 3
1 Subtract one of the answers from the
other

11
Prepared by: Nathaniel M. Cabansay, BSCpE

@ 2 4 2 2 +1
s
= +1 . ~j − +1 k−z + 1 − {•
3 15 5 3
2 Factor from all terms

@ 2 4 2 2 Multiply M + 1 − Q by −1
. .
= +1 .| − +1 − +1 + }
3 15 5 3 x @
3

@ 2 4 4 2 2 2 + 1 by −
r
and −
.
= +1 . | − − − − + }
3 15 15 5 5 3
x x
4 Multiply

@
5 = +1 . 0 Evaluate the expression

6 =0 Any entity multiplied by 0 is 0

Take note that the difference CAN be a nonzero constant and subtracting one of the functions from the other is
one way to check how much a certain function differs from the other.

12
Prepared by: Nathaniel M. Cabansay, BSCpE

MODULE 4 INTEGRALS OF EXPONENTIAL AND LOGARITHMIC FUNCTIONS

INTEGRALS WITH LOGARITHMIC RESULTS

Remember the power rule where we can’t deal with = −1? These functions are quite common when integrating
and this will be dealt with here.

∫ h Ch = €•|h| + b (Integral of a reciprocal, Natural logarithm result)


P
1.
∫ h €• ‚ Ch = €ƒ„ ‚ |h| + b (Logarithm to a base result)
P
2.

That first one is a very useful integral.

DERIVING INTEGRALS WITH LOGARITHMIC RESULTS

INTEGRAL OF A RECIPROCAL

1 )
No. STATEMENT REASON

)% = ln %
% )
1 Derivative of the natural logarithm

1
X )% = ln % + Y
%
2 Integrate both sides with respect to x

P If % < 0, the integral is ln −% + Y , so place absolute


X Ch = ۥ|h| + b
h value bars around % in the natural logarithm.
3


We can then rewrite the reverse power rule combined with integration by substitution as:

hc8P
X hc Ch = …c + P + b , c ≠ −P
€•|h| + b , c = −P

INTEGRAL RESULTING IN A LOGARITHM TO A BASE

We can derive this in two ways. We can start from scratch:

1 )
No. STATEMENT REASON

)% = log 3 %
% ln 0 )
1 Derivative of the logarithm

1
X )% = log 3 % + Y
% ln 0
2 Integrate both sides with respect to x

P Place absolute value bars around % in the logarithm


X Ch = €ƒ„ ‚ |h| + b
h €• ‚
3

13
Prepared by: Nathaniel M. Cabansay, BSCpE

45 3
Or we can use the integral of a reciprocal and multiply both sides by

1
No. STATEMENT REASON

X )% = ln|%| + Y
%
1 Given

1 1 ln|%|
X )% = +Y
ln 0 % ln 0
45 3
2 Multiply both sides by . The constant remains as it is.

P
X Ch = €ƒ„ ‚ |h| + b
h €• ‚
3 Use Integral Rule 3 on the left side and change-of-base


formula on the right side

EXAMPLES OF INTEGRALS WITH LOGARITHMIC RESULTS

Find the indefinite integral of:

2
=
4 +2

2
No. STATEMENT REASON

=
4 +2
1 Given

2
X ) =X )
4 +2
2 Integrate both sides with respect to

1 1 Integrate by substitution. Let % = 4 + 2, )% = 4 ) .


X ) = X )%
2 % Since the integrand has a 2 dx: )% = 2 )
3
.
1 1
X ) = ln|%| + Y X )% = ln|%| + Y
2 %
4

1 Substitute % = 4 + 2.
X ) = ln|4 + 2| + Y
2 (OPTIONAL) ? log 3 = log 3
5

and convert rational
exponents to radicals

INTEGRALS OF LOGARITHMIC AND EXPONENTIAL FUNCTIONS

Now, let’s deal with integrals of logarithmic and exponential functions.

∫ Oh Ch = Oh + b (Integral of exponential function of Euler’s number)


∫ ‚h Ch = €• ‚ + b (Integral of exponential function of any base)
1.
‚h

∫ €• h Ch = h €• h − h + b = h €• h − P + b (Integral of natural logarithm)


2.

∫ €ƒ„ ‚ h Ch = h €ƒ„ ‚ h − €• ‚ + b = €• ‚ €• h − P + b (Integral of a logarithm with arbitrary base)


h h
3.
4.

14
Prepared by: Nathaniel M. Cabansay, BSCpE

DERIVING INTEGRALS OF LOGARITHMIC AND EXPONENTIAL FUNCTIONS

INTEGRAL OF EXPONENTIAL FUNCTION OF EULER’S NUMBER

This is the reverse of the derivative rule for exponential function of Euler’s number e

No. STATEMENT REASON


) /
e/ )% = e
) e
1 Derivative of the exponential function of Euler’s number

X Oh Ch = Oh + b
2 Integrate both sides with respect to

INTEGRAL OF EXPONENTIAL FUNCTION OF ANY BASE

This is the reverse of the derivative rule for exponential function of any base

No. STATEMENT REASON


) /
0/ ln 0 )% = 0
)
1 Derivative of the exponential function of any base

X 0/ ln 0 )% = 0/ + Y
2 Integrate both sides with respect to

3 ln 0 X 0/ )% = 0/ + Y Integral Rule 3

‚h Divide both sides by ln 0


X ‚h Ch = +b
€• ‚
4

INTEGRAL OF NATURAL LOGARITHM

We use integration by parts here.

No. STATEMENT REASON

1 X ln % )% Given

1 Integrate by parts ∫ ) = − ∫ ) with respect


X ln % )% = % ln % − X % ⋅ )%
% to % , = ln % , ) = )% , so ) = / and = %
2

3 X ln % )% = % ln % − X 1 )% Divide % by %

X €• h Ch = h €• h − h + b X 1 )% = % + Y
4

X €• h Ch = h €• h − P + b OPTIONAL: Factor % in the right-hand side

15
Prepared by: Nathaniel M. Cabansay, BSCpE

INTEGRAL OF LOGARITHM TO ANY BASE

We can integrate by parts…

No. STATEMENT REASON

X log3 % )%
1 Given

1 Integrate by parts ∫ ) = − ∫ ) with respect


X log 3 % )% = % log 3 % − X % ⋅ )%
% ln 0 to % , = log 3 % , ) = )%, so ) = and = %
2
/ 45 3

1 by %
X log 3 % )% = % log 3 % − X )%
ln 0
/ 45 3
3 Multiply

1
X log 3 % )% = % log 3 % − X 1 )%
ln 0
4 Integral Rule 3

h
X €ƒ„ ‚ h Ch = h €ƒ„ ‚ h − +b X 1 )% = % + Y
€• ‚
5

…or use the integral of natural logarithm

No. STATEMENT REASON

X ln % )% = % ln % − % + Y
1 Integral of the natural logarithm

1 ln % % Divide both sides of the equation by ln 0. The constant


X ln % )% = % − +Y
ln 0 ln 0 ln 0 Y remains as is.
2

h
X €ƒ„ ‚ h Ch = h €ƒ„ ‚ h − +b
€• ‚ of base formula log 3 =
4‡ˆ‰
3 Use Integral Rule 3 on the left side, then use the change


4‡ˆ‰ 3
on both sides

EXAMPLES OF INTEGRALS OF LOGARITH MIC AND EXPONENTIAL FUNCTIONS


Find the indefinite integral of:

ℎ ? = er† + 10†

ℎ ? =e + 10
No. STATEMENT REASON
r† †

Integrate both sides with respect to ?


1 Given

X ℎ ? )? = X er† + 10† )?
2

X ℎ ? )? = X er† )? + X 10† )?
3 Integral Rule 4

1 Integrate by substitution. Let % = 4? and )% = 4 )? .


X ℎ ? )? = X e/ )% + X 10† )?
4 Thus, )% = )? and use Integral Rule 3.
4
r

16
Prepared by: Nathaniel M. Cabansay, BSCpE

e/ 10†
∫ e/ )% = e/ + Y and ∫ 0/ )% = 45 3 + Y
3‹
X ℎ ? )? = + +Y
4 ln 10
5

er† 10† Substitute % = 4?


X ℎ ? )? = + +Y
4 ln 10
6

Find the indefinite integral of:

_ = ln _ + 3 − log . _ + 3

No. STATEMENT REASON

1 _ = ln _ + 3 − log . _ + 3 Given

Integrate both sides with respect to _


X _ )_ = X ln _ + 3 − log . _ + 3 )_
2

X _ )_ = X ln _ + 3 )_ − X log . _ + 3 )_
3 Integral Rule 4

Integration by Substitution: Let % =


X _ )_ = X ln % )% − X log . % )%
_ + 3 and )% = )_
4

1 ∫ ln % )% = % ln % − % + Y and
X _ )_ = % j ln % − 1 + zlog3 % − {k + Y
ln 0 ∫ log 3 % )% = % log3 % − 45 3 + Y ,
/
5

then simplify

1 Substitute back % = _ + 3
X _ )_ = _ + 3 j ln _ + 3 − 1 + zlog 3 _ + 3 − {k + Y
ln 0
6

17
Prepared by: Nathaniel M. Cabansay, BSCpE

MODULE 5 INTEGRALS OF TRIGONOMETRIC FUNCTIONS

INTEGRALS OF THE TWO BASIC TRIGONOMETRIC FUNCTIONS

The two basic trigonometric functions we will be dealing with first are the sine and cosine functions.

∫ Œ•• h Ch = − ŽƒŒ h + b (Integral of Sine)


∫ ŽƒŒ h Ch = Œ•• h + b (Integral of Cosine)
1.
2.

DERIVING INTEGRALS OF THE TWO BASIC TRIGONOMETRIC FUNCTIONS

SINE

This is the reverse of the derivative of the cosine, as its result (derivative) is the sine function.

No. STATEMENT REASON


)
− sin % )% = cos %
)
1 Derivative of cosine

)
sin % )% = − cos %
)
2 Multiply both sides by -1

X Œ•• h Ch = − ŽƒŒ h + b
3 Integrate both sides with respect to and add a


constant of integration

COSINE

This is the reverse of the derivative of the sine, as its result (derivative) is the cosine function.

No. STATEMENT REASON


)
cos % )% = sin %
)
1 Derivative of sine

X ŽƒŒ h Ch = Œ•• h + b
2 Integrate both sides with respect to and add a


constant of integration

EXAMPLES OF INTEGRALS OF THE TWO BASIC TRIGONOMETRIC FUNCTIONS

Evaluate:

X 2 cos 2 )

No. STATEMENT REASON

X 2 cos 2 )
1 Given

18
Prepared by: Nathaniel M. Cabansay, BSCpE

2 X cos % )% Substitute % = 2 and )% = 2 )

Integrate with respect to %


X cos % )% = sin % + Y
3

X 2 cos 2 ) = sin 2 +Y
4 Substitute back the original function

Find the indefinite integral of:

= 2 sin cos

We can go about this in three ways to get three possible answers:

We can do substitution through sin

No. STATEMENT REASON

1 = 2 sin cos Given

X ) = X 2 sin cos )
2 Integrate both sides with respect to

Integral Rule 3 then integrate by substitution: Let % =


X ) = 2 X % )%
sin and )% = cos
3

X ) = %. + Y
4 Integral Rule 2 and simplify

Substitute back % = sin


X ) = sin. + Y
5

We can also do substitution through cos

No. STATEMENT REASON

1 = 2 sin cos Given

X ) = X 2 sin cos )
2 Integrate both sides with respect to

Integral Rule 3 then integrate by substitution: Let % =


X ) = −2 X % )%
cos and )% = − sin , or −)% = sin
3

X ) = −% . + Y
4 Integral Rule 2 and simplify

Substitute back % = cos


X ) = − cos . + Y
5

We can also use the trigonometric identity 2 sin cos = sin 2

No. STATEMENT REASON

1 = 2 sin cos Given

19
Prepared by: Nathaniel M. Cabansay, BSCpE

2 = sin 2 Use the identity 2 sin cos = sin 2

X ) = X sin 2 )
3 Integrate both sides with respect to

1 Integrate by substitution: Let % = 2 , so )% = 2 ) or


X ) = X sin % )%
2 )% = )
4
.

1
X ) = − cos % + Y X sin % )% = − cos % )%
2
5

cos 2 Substitute back % = 2


X ) =− +Y
2
6

We’ll see how much these three differ:

Let’s define three constants , ., and @

No. STATEMENT REASON


= sin. − − cos .
cos 2
1 Subtract one of the answers from another

=− − − cos .
.
2
cos 2
=− − sin.
@
2
= sin. + cos . . multiply − cos . by −1.
cos − sin For . and @ use the identity cos 2 =
2 For and
. .
=− + cos .
.
2 cos . − sin.
cos . − sin.
=− − sin.
@
2
= sin. + cos . For . and @ , multiply the numerator by −1
sin − cos
3
. .
= + cos .
2
(as there was a negative sign in front)
.

sin. − cos .
= − sin.
@
2
= sin. + cos . For . and @ , combine into a single fraction.
sin
+ cos
4
. . Factor -1 from the numerator for @
=
.
2
− sin − cos .
.
=
@
2
sin. + cos .
=−
@
2
=1
identity sin. + cos . = 1
5 For all constants, use the Pythagorean

1
=
.
2
1
=−
@
2

20
Prepared by: Nathaniel M. Cabansay, BSCpE

They all differed only by a constant in this case, so our results are still correct, and these differences can already be
accounted for in the constants of integration of those three functions.

INTEGRALS WITH TRIGONOMETRIC RESULTS

These are the reverses of certain trigonometric functions.

∫ ŒOŽ G h Ch = ••• h + b (Tangent Result)


∫ ŽŒŽ h Žƒ• h Ch = − ŽŒŽ h + b (Cosecant Result)
1.

∫ ŒOŽ h ••• h Ch = ŒOŽ h + b (Secant Result)


2.

∫ ŽŒŽ G h Ch = − Žƒ• h + b (Cotangent Result)


3.
4.

DERIVING INTEGRALS WITH TRIGONOMETRIC RESULTS

TANGENT RESULT

This is the reverse of the derivative of the tangent

)
No. STATEMENT REASON

sec . % )% = tan %
)
1 Derivative of tangent

X ŒOŽ G h Ch = ••• h + b
2 Integrate both sides with respect to and add a


constant of integration

COSECANT RESULT

This is the reverse of the derivative of the cosecant

No. STATEMENT REASON


)
− csc % cot % )% = csc %
)
1 Derivative of cosecant

)
csc % cot % )% = − csc %
)
2 Multiply both sides by -1

X ŽŒŽ h Žƒ• h Ch = − ŽŒŽ h + b


3 Integrate both sides with respect to and add a


constant of integration

SECANT RESULT

This is the reverse of the derivative of the secant

No. STATEMENT REASON


)
sec % tan % )% = sec %
)
1 Derivative of secant

21
Prepared by: Nathaniel M. Cabansay, BSCpE

X ŒOŽ h ••• h Ch = ŒOŽ h + b


2 Integrate both sides with respect to and add a


constant of integration

COTANGENT RESULT

This is the reverse of the derivative of the cotangent

No. STATEMENT REASON


)
− csc . % )% = cot %
)
1 Derivative of cotangent

)
csc . % )% = − cot %
)
2 Multiply both sides by -1

X ŽŒŽG h Ch = − Žƒ• h + b
3 Integrate both sides with respect to and add a


constant of integration

EXAMPLES OF INTEGRALS WITH TRIGONOMETRIC RESULTS

Find the indefinite integral of:

= sec . − csc .

No. STATEMENT REASON

1 = sec . − csc . Given

X ) = X sec . − csc . )
2 Integrate both sides with respect to

X ) = X sec . ) − X csc . )
3 Integral Rule 4

4
X ) = tan − − cot +Y ∫ sec . ) = tan + Y and ∫ csc . ) = − cot + Y

Multiply − cot by −1
X ) = tan + cot + Y
5

INTEGRALS OF REMAINING FOUR TRIGONOMETRIC FUNCTIONS

Here are the integrals of the remaining four trigonometric functions.:

∫ ••• h Ch = − €•|ŽƒŒ h| + b = €•|ŒOŽ h| + b (Integral of Tangent)


∫ ŽŒŽ h Ch = €•|ŽŒŽ h − Žƒ• h| + b = − €•|ŽŒŽ h + Žƒ• h| + b = €• ‘••• MGQ‘ (Integral of Cosecant)
1.
h
2.

∫ ŒOŽ h Ch = €•|ŒOŽ h + ••• h| + b = − €•|ŒOŽ h − ••• h| + b = €• ‘••• MG + N Q‘ (Integral of Secant)


h ’

∫ Žƒ• h Ch = €•|Œ•• h| + b = − €•|ŽŒŽ h| + b (Integral of Cotangent)


3.
4.

22
Prepared by: Nathaniel M. Cabansay, BSCpE

TANGENT HALF-ANGLE SUBSTITUTIONS

We will first define a few substitutions that we will be using to derive certain rules. We will be discussing these
further in Chapter 4.

For trigonometric functions, we derive these two:

sin 2 =
. “”5
8“”5
1.

No. STATEMENT REASON

1 sin 2 = sin 2 Given

2 sin 2 = 2 sin cos Double Angle Formula for Sine


sin 2 = 2 tan cos . •‡– –—5
= tan
•‡– •‡–
3 Multiply the right side by , then

2 tan 1
sin 2 = cos =
sec . sec
4

2 tan Pythagorean identity: 1 + tan. = sec .


sin 2 =
1 + tan.
5

cos 2 =
“”5
8“”5
2.

No. STATEMENT REASON

1 cos 2 = cos 2 Given

2 cos 2 = cos . − sin. Double Angle Formula for Cosine

cos 2 = 1 − tan. cos . •‡– –—5


= tan
•‡– •‡–
3 Multiply the right side by , then

1 − tan. 1
cos 2 = cos =
sec . sec
4

1 − tan. Pythagorean identity: 1 + tan. = sec .


cos 2 =
1 + tan.
5


We will also use the sum/difference identity, double angle identity, and half angle identity for tangent:

tan ˜ + ™ = , tan 2 = , tan = œ


“”5 š8“”5 › . “”5 •‡–
“”5 š “”5 › “”5 . 8•‡–

DERIVING INTEGRALS OF REMAINING FOUR TRIGONOMETRIC FUNCTIONS

INTEGRAL OF TANGENT

No. STATEMENT REASON

X tan % )%
1 Given

23
Prepared by: Nathaniel M. Cabansay, BSCpE

sin % Use the ratio identity tan =


–—5
X tan % )% = X )%
cos %
•‡–
2

1 Integrate by substitution: Let > = cos % and )> =


X tan % )% = − X )>
> − sin % )%
3

1
X tan % )% = − ln|>| + Y X )% = ln|%| + Y
%
4

5 X tan % )% = − ln|cos %| + Y Substitute back > = cos %

X ••• h Ch = €•|ŒOŽ h| + b ? log 3 = log 3 †


= sec
•‡–
6 and

INTEGRAL OF COSECANT

We can derive this in two ways:

We can use csc % − cot % …

No. STATEMENT REASON

X csc % )%
1 Given

csc . % − csc % cot % Multiply csc % by


•–• / •‡“ /
X csc % )% = X )%
csc % − cot %
•–• / •‡“ /
2

1 Integrate by substitution: Let > = csc % − cot % and


X csc % )% = X )>
> )> = − csc % cot % + csc . % = csc. % − csc % cot %
3

1
X csc % )% = ln|>| + Y X )% = ln|%| + Y
%
4

Substitute back > = csc % − cot %


X ŽŒŽ h Ch = €•|ŽŒŽ h − Žƒ• h| + b
5


…or we can use csc % + cot %

No. STATEMENT REASON

X csc % )%
1 Given

csc . % + csc % cot % Multiply csc % by


•–• /8•‡“ /
X csc % )% = X )%
csc % + cot %
•–• /8•‡“ /
2

1 Integrate by substitution: Let > = csc % + cot % and


X csc % )% = − X )>
> )> = − csc % cot % − csc . %, so −)> = csc % cot % +
3

csc . %
1
X csc % )% = − ln|>| + Y X )% = ln|%| + Y
%
4

24
Prepared by: Nathaniel M. Cabansay, BSCpE

5 X ŽŒŽ h Ch = − €•|ŽŒŽ h + Žƒ• h| + b Substitute back > = csc % − cot %


We can prove − ln|csc % + cot %| = ln|csc % − cot %|

No. STATEMENT REASON

− ln|csc % + cot %| = ln|csc % − cot %|


1
1 Given
? log 3 = log 3 †
ln • • = ln|csc % − cot %|
csc % + cot %
2

1 Function property of equality: If ln = ln ?, then =?


= csc % − cot %
csc % + cot %
3

csc % − cot % •–• / •‡“ /


= csc % − cot %
csc . % − cot . % •–• / •‡“ /
4 Multiply the left-hand side by

csc % − cot % = csc % − cot % Use the Pythagorean identity 1 + cot . = csc .

5

From the second form, we can derive a third form.

No. STATEMENT REASON

X csc % )% = − ln|csc % + cot %| + Y


csc %
1 Cosecant plus cotangent form of integral of

1 ? log 3 = log 3 †
X csc % )% = ln • •+Y
csc % + cot %
2

1
X csc % )% = ln ž ž+Y
1 cos %
3 Reciprocal and ratio identities for cosecant and

+
sin % sin %
cotangent

sin % Move sin % to the numerator to simplify the


X csc % )% = − ln • •+Y
1 + cos %
4
expression.

sin. %
X csc % )% = − ln žŸ ž+Y
5 Rewrite the inside expression as a square root
1 + cos % .

1 − cos . % Pythagorean Identity sin. + cos . =1


X csc % )% = − ln žŸ ž+Y
6
1 + cos % .

1 − cos % 1 + cos %
X csc % )% = − ln žŸ ž+Y
7 Difference of two squares
1 + cos % .

Cancel a 1 + cos % from the numerator and


1 − cos %
X csc % )% = − ln žŸ ž+Y
8
1 + cos %
denominator

25
Prepared by: Nathaniel M. Cabansay, BSCpE

h
X ŽŒŽ h Ch = − €• ‘••• M Q‘ + b % 1 − cos %
G tan =Ÿ
9
2 1 + cos %

INTEGRAL OF SECANT

We can derive this in two ways:

We can use sec % + tan % …

No. STATEMENT REASON

X sec % )%
1 Given

sec . % + sec % tan % Multiply sec % by


– • /8“”5 /
X sec % )% = X )%
sec % + tan %
– • /8“”5 /
2

1 Integrate by substitution: Let > = sec % + tan % and


X sec % )% = X )>
> )> = sec % tan % + sec . % = sec . % + sec % tan %
3

1
X sec % )% = ln|>| + Y X )% = ln|%| + Y
%
4

Substitute back > = sec % + tan %


X ŒOŽ h Ch = €•|ŒOŽ h + ••• h| + b
5


…or we can use sec % − tan %

No. STATEMENT REASON

X sec % )%
1 Given

sec . % − sec % tan % Multiply sec % by


– • / “”5 /
X sec % )% = X )%
sec % − tan %
– • / “”5 /
2

1 Integrate by substitution: Let > = sec % − tan % and


X sec % )% = − X )>
> )> = sec % tan % − sec . % , so −)> = sec . % −
3

sec % tan %
1
X sec % )% = − ln|>| + Y X )% = ln|%| + Y
%
4

Substitute back > = sec % − tan %


X ŒOŽ h Ch = − €•|ŒOŽ h − ••• h| + b
5

We can prove − ln|sec % − tan %| = ln|sec % + tan %|

No. STATEMENT REASON


− ln|sec % − tan %| = ln|sec % + tan %|
1
1 Given

? log 3 = log 3 †
ln • • = ln|sec % + tan %|
sec % − tan %
2

26
Prepared by: Nathaniel M. Cabansay, BSCpE

1 Function property of equality: If ln = ln ?, then =?


= sec % + tan %
sec % − tan %
3

sec % + tan % – • /8“”5 /


= sec % + tan %
sec. % − tan. %
4 Multiply the left-hand side by – • /8“”5 /

sec % + tan % = sec % + tan % Use the Pythagorean identity tan. % + 1 = sec . %

5

From the first form, we can derive a third form:

No. STATEMENT REASON

X sec % )% = ln|sec % + tan %| + Y


1 Given

1
X sec % )% = ln • + tan %• + Y
cos %
2 Reciprocal identity of secant

% % /
sin. + cos .
X sec % )% = ln ž 2 2 + tan %ž + Y .
% rewrite % as 2
/
3 Pythagorean Identity using the angle , then

cos 2
2
.

% %
sin. 2 + cos . 2
X sec % )% = ln ž % % + tan %ž + Y
4 Double angle identity of cosine

cos . − sin.
2 2
% Divide the denominator and numerator by cos .
tan. + 1
X sec % )% = ln ž 2
% + tan %ž + Y
.
5

1 − tan.
2
% %
tan. + 1 2 tan
X sec % )% = ln ž 2 2 ž+Y
%+ %
6 Double angle formula for tangent

1 − tan. 1 − tan.
2 2
% %
tan. + 2 tan + 1
X sec % )% = ln ž 2 2 ž+Y
%
7 Add both terms together

1 − tan.
2
% .
Mtan 2 + 1Q
X sec % )% = ln ž ž+Y
Factor the perfect square trinomial numerator
% %
8

M1 − tan 2Q Mtan 2 + 1Q
and difference of two squares denominator.

%
Addition is commutative.

Mtan 2 + 1Q Cancel a Mtan . + 1Q from both the numerator


/

X sec % )% = ln ž % ž+Y
9

M1 − tan 2Q and denominator

% ¡ ¡
tan + tan tan =1
X sec % )% = ln ž 2 4 ž+Y 4
% ¡
10

1 − tan tan
2 4
h ’ tan ˜ + tan ™
X ŒOŽ h Ch = €• ‘••• M + Q‘ + b tan ˜ + ™ =
G N 1 − tan ˜ tan ™
11

27
Prepared by: Nathaniel M. Cabansay, BSCpE

INTEGRAL OF COTANGENT

No. STATEMENT REASON

X cot % )%
1 Given

cos % Use the ratio identity cot =


•‡–
X cot % )% = X )%
sin %
–—5
2

1 Integrate by substitution: Let > = sin % and )> =


X cot % )% = X )>
> cos % )%
3

1
X cot % )% = ln|>| + Y X )% = ln|%| + Y
%
4

X Žƒ• h Ch = €•|Œ•• h| + b Substitute back > = sin %


OPTIONAL: ? log 3 = log3
5

= − €•|ŽŒŽ h| + b

EXAMPLES OF INTEGRALS OF TRIGONOMETRIC FUNCTIONS

¡
Find the integral of:

¢ = tan M − Q
2
We can go about this in two ways:
We can use the function itself…

¡
No. STATEMENT REASON

¢ = tan M − Q
2
1 Given

¡
X¢ ) = X tan M − Q )
2
2 Integrate both sides with respect to

Substitute % = . − and )% = −) or −)% = )


£
X¢ ) = − X tan % )%
3

X¢ ) = − ln|sec %| + Y X tan % )% = ln|sec %| + Y


4

? log 3 = log 3 †
X¢ ) = ln|cos %| + Y
5 and the reciprocal of secant is cosine

¡ Substitute back % = −
£
X¢ ) = ln ‘cos M − Q‘ + Y
2
.
6

Cofunction identity cos M − Q = sin


£
X¢ ) = ln|sin | + Y .
7

28
Prepared by: Nathaniel M. Cabansay, BSCpE

…or use the cofunction identity cot = tan M − Q


£
.

¡
No. STATEMENT REASON

¢ = tan M − Q
2
1 Given

2 ¢ = cot Cofunction identity tan M . − Q = cot


£

X¢ ) = X cot )
3 Integrate both sides with respect to

X¢ ) = ln|sin | + Y X cot % )% = ln|sin %| + Y


4

In some other integrals, we CAN get the same answer even though we used different methods.

29
Prepared by: Nathaniel M. Cabansay, BSCpE

MODULE 6 INTEGRALS OF INVERSE TRIGONOMETRIC FUNCTIONS

INTEGRALS WITH INVERSE TRIGONOMETRIC RESULTS

These are just reverses of the derivatives of trigonometric functions.

∫7 Ch = •¤ŽŒ•• M Q + b = − •¤ŽŽƒŒ M Q + Y (Arcsine/Arccosine Result)


P h h
‚G hG ‚ ‚
1.

∫ hG 8‚G Ch = ‚ •¤Ž••• M‚Q + b = − ‚ •¤ŽŽƒ• M‚Q + b (Arctangent/Arccotangent Result)


P P h P h
2.

∫ Ch = •¤ŽŒOŽ M Q + b = − •¤ŽŽŒŽ M Q + b (Arcsecant/Arccosecant Result)


P P h P h
h7hG ‚G ‚ ‚ ‚ ‚
3.

DERIVING INTEGRALS WITH INVERSE TRIGONOMETRIC RESULTS

ARCSINE/ARCCOSINE RESULT

This is the reverse of the derivatives of the arcsine and arccosine

No. STATEMENT REASON


1 )
)% = arcsin %
√1 − %. )
1 Derivatives of arcsine and arccosine.

1 )
− )% = arccos %
√1 − % . )
1 ) % Divide % on both sides by 0. Since 0 is a constant, use
)% = arcsin M Q
% . ) 0 Derivative Rule 3. For arccos M Q, multiply both sides by
/
2

0 œ1 − M0 Q 3

1 ) %
)% = M− arccos M QQ
-1.

% ) 0
0œ1 − M Q
.

0
1 ) % Bring the 0 inside the radical
)% = arcsin M Q
√0. − %. ) 0
3

1 ) %
)% = M− arccos M QQ
√0. − %. ) 0
P h
X Ch = •¤ŽŒ•• M Q + b
√‚G − hG ‚
4 Integrate both sides with respect to

P h
X Ch = − •¤ŽŽƒŒ M Q + b
√‚G − hG ‚

ARCTANGENT/ARCCOTANGENT RESULT

This is the reverse of the derivatives of the arctangent and arccotangent

30
Prepared by: Nathaniel M. Cabansay, BSCpE

No. STATEMENT REASON


1 )
)% = arctan %
1+% . )
1 Derivatives of arctangent and arccotangent

1 )
− )% = arccot %
1+% . )
1 ) % Divide the % on both sides by 0. Since 0 is a constant,
)% = arctan M Q
% . ) 0 use Derivative Rule 3. For arccot M3 Q, multiply both
/
0 z1 + M Q {
2

0
1 ) %
)% = M− arccot M QQ
sides by -1

% . ) 0
0 z1 + M Q {
0
1 ) 1 %
)% = z arctan M Q{
0. + % . ) 0 0
3
3 Multiply both sides by and simplify the left-hand side

1 ) 1 %
)% = z− arccot M Q{
0 +%
. . ) 0 0
P P h
X Ch = •¤Ž••• M Q + b
‚G + hG ‚ ‚
4 Integrate both sides with respect to

P P h
X G Ch = − •¤ŽŽƒ• M Q + b
‚ + hG ‚ ‚

ARCSECANT/ARCCOSECANT RESULT

This is the reverse of the derivative of the arcsecant and arccosecant

No. STATEMENT REASON


1 )
)% = arcsec %
%√%. − 1 )
1 Derivatives of arcsecant and arccosecant

1 )
− )% = arccsc %
%√% − 1
. )
1 ) % Divide the % on both sides by 0. The 0 cancels out with
)% = arcsec M Q
% ) 0 /
in the denominator. For arccsc M Q, multiply both
/
2

% œM Q − 1
.

0
3 3
the

1 ) %
)% = M− arccsc M QQ
sides by -1.

% . ) 0
% œM Q − 1
0
1 ) 1 %
)% = z arcsec M Q{
) 0 0
Multiply both sides by 3
%
3

0% œM Q − 1
.

0
1 ) 1 %
)% = z− arccsc M Q{
% . ) 0 0
0% œM Q − 1
0
1 ) 1 % Bring the 0 inside the radical
)% = z arcsec M Q{
%√%. − 0. ) 0 0
4

31
Prepared by: Nathaniel M. Cabansay, BSCpE

1 ) 1 %
)% = z− arccsc M Q{
%√%. − 0. ) 0 0
P P h
X Ch = •¤ŽŒOŽ M Q + b
h√hG − ‚G ‚ ‚
5 Integrate both sides with respect to

P P h
X Ch = − •¤ŽŽŒŽ M Q + b
h√hG − ‚G ‚ ‚

EXAMPLES OF INTEGRALS WITH INVERSE TRIGONOMETRIC RESULTS

Find the indefinite integral of:

1
=
√16 − .

1
No. STATEMENT REASON

=
√16 −
1 Given
.

1
X ) =X )
√16 −
2 Integrate both sides with respect to
.

∫7 )% = arcsin M Q + Y . 0 in this case is 4.


/
X ) = arcsin M Q + Y
4
3 / 3
3

Find the indefinite integral of:

1
? =
?. +9

1
No. STATEMENT REASON

? =
?. +9
1 Given

1 Integrate both sides with respect to ?


X ? )? = X )?
?. +9
2

1 ? ∫3 )% = arctan M Q + Y . 0 in this case is 3.


/
X ? )? = arctan M Q + Y
3 3
8/ 3 3
3

Find the indefinite integral of:

1
ℎ ] =
]√] . − 25

1
No. STATEMENT REASON

ℎ ] =
]√] . − 25
1 Given

32
Prepared by: Nathaniel M. Cabansay, BSCpE

1 Integrate both sides with respect to ]


X ℎ ] )] = X )]
]√] . − 25
2

1 ] ∫ )% = arcsec M Q + Y . 0 in this case is 5.


/
X ℎ ] )] = arcsec z { + Y
5 5
/ 7/ 3 3 3
3

INTEGRALS OF INVERSE TRIGONOMETRIC FUNCTIONS

These use either the integral of an inverse function ∫ ) = −W + Y or integration by


parts.

∫ •¤ŽŒ•• M‚Q Ch = h •¤ŽŒ•• ‚ + √‚G − hG + b (Integral of Arcsine)


h h
1.

∫ •¤ŽŽƒŒ M‚Q Ch = h •¤ŽŽƒŒ ‚ − √‚G − hG + b (Integral of Arccosine)


h h
2.

∫ •¤Ž••• M‚Q Ch = h •¤Ž••• ‚ − G €•|‚G + hG | + b (Integral of Arctangent)


h h ‚
3.

∫ •¤ŽŽŒŽ M‚Q Ch = h •¤ŽŽŒŽ ‚ − ‚ €•¥h − √hG − ‚G ¥ + b = h •¤ŽŽŒŽ ‚ + ‚ €•¥h + √hG − ‚G ¥ + b


h h h
4.

∫ •¤ŽŒOŽ M‚Q Ch = h •¤ŽŒOŽ ‚ − ‚ €•¥h + √hG − ‚G ¥ + b = h •¤ŽŒOŽ ‚ + ‚ €•¥h − √hG − ‚G ¥ + b


(Integral of Arccosecant)
h h h
5.

∫ •¤ŽŽƒ• M‚Q Ch = h •¤ŽŽƒ• ‚ + G €•|‚G + hG | + b (Integral of Arccotangent)


(Integral of Arcsecant)
h h P
6.

DERIVING INTEGRALS OF INVERSE TRIGONOMETRIC FUNCTIONS

INTEGRAL OF ARCSINE

INTEGRAL OF AN INVERSE FUNCTION

%
No. STATEMENT REASON

X arcsin M Q )%
0
1 Given

X ) = −W +Y
2 Integral of an inverse function

1 % % % % % = arcsin %, = , and ) = )%,


/
X arcsin )% = arcsin + cos Marcsin Q + Y
0 0 0 0 0
3 3
= sin % and W = − cos %
3 Let

% % %
this means

Multiply both sides by 0


X arcsin )% = % arcsin + 0 cos Marcsin Q + Y
0 0 0
4

% % % .
X arcsin )% = % arcsin + 0Ÿ1 − M Q + Y
5 Use this triangle and the trigonometric ratio CAH

0 0 0
1

71 − .

33
Prepared by: Nathaniel M. Cabansay, BSCpE

h h Bring the 0 inside the radical


X •¤ŽŒ•• M Q Ch = h •¤ŽŒ•• + 7‚G − hG + b
‚ ‚
6

INTEGRATION BY PARTS

%
No. STATEMENT REASON

X arcsin M Q )%
0
1 Given

% % %
= arcsin M Q and ) =
/
X arcsin M Q )% = % arcsin M Q − X )%
0 0 √0 − % .
. 3
2
)%. This gives us ) = )% and =%
Integrate by parts: Let

+Y 73 /

% % 1 1 Integrate by substitution: Let > = 0. − % . so


X arcsin M Q )% = % arcsin M Q + X )> + Y
0 0 2 √> )> = −2% )% or − . )> = % )%
3

% %
X arcsin )% = % arcsin + √> + Y
0 0
4 Integral Rule 2: Power Rule

h h Substitute back > = 0. − % .


X •¤ŽŒ•• M Q Ch = h •¤ŽŒ•• + 7‚G − hG + b
‚ ‚
5

INTEGRAL OF ARCCOSINE

INTEGRAL OF AN INVERSE FUNCTION

%
No. STATEMENT REASON

X arccos M Q )%
0
1 Given

X ) = −W +Y
2 Integral of an inverse function

1 % % % % % = arccos %, = 3 , and ) = 3 )% ,
/
X arccos )% = arccos − sin Marccos Q + Y
0 0 0 0 0 = cos % and W = sin %
3 Let

% % %
this means

Multiply both sides by 0


X arccos )% = % arccos − 0 sin Marccos Q + Y
0 0 0
4

% % .
X arccos )% = % arccos % − 0 Ÿ1 − M Q + Y
5 Use this triangle and the trigonometric ratio

0 0
SOH


1
71 − .

h Bring the 0 inside the radical


X •¤ŽŽƒŒ Ch = h •¤ŽŽƒŒ h − 7‚G − hG + b

6

34
Prepared by: Nathaniel M. Cabansay, BSCpE

INTEGRATION BY PARTS

%
No. STATEMENT REASON

X arccos M Q )%
0
1 Given

% % %
= arccos M Q and ) =
/
X arccos M Q )% = % arccos M Q + X )%
0 0 √0 − % .
. 3
2
)%. This gives us ) = − )% and =%
Integrate by parts: Let

+Y 73 /

% % 1 1 Integrate by substitution: Let > = 0. − % . so


X arccos M Q )% = % arccos M Q − X )> + Y
0 0 2 √> )> = −2% )% or − )> = % )%
3
.

% %
X arccos )% = % arccos − √> + Y
0 0
4 Integral Rule 2: Power Rule

h h Substitute back > = 0. − % .


X •¤ŽŽƒŒ M Q Ch = h •¤ŽŽƒŒ − 7‚G − hG + b
‚ ‚
5

INTEGRAL OF ARCTANGENT

INTEGRAL OF AN INVERSE FUNCTION

No. STATEMENT REASON

X arctan % )%
1 Given

2 X ) = −W +Y Integral of an inverse function

1 % % % % % = arctan % , = , and ) =
/
X arctan )% = arctan − ln ‘sec Marctan Q‘ + Y
0 0 0 0 0
3
)% , this means = tan % and W = ln|sec %|
3 Let

% % % Multiply both sides by 0


X arctan )% = % arctan − 0 ln ‘sec Marctan Q‘ + Y
0 0 0
4

% % % .
X arctan )% = % arctan − 0 ln žŸ1 + M Q ž + Y
5 Use this triangle and the trigonometric ratio

0 0 0
SHA

71 + .

1
% % 1
X arctan )% = % arctan − 0 ln • 70. + %. • + Y
0 0 0
3
6 Factor out a from the radical

% %
X arctan )% = % arctan − 0 ln ‘70. + %. ‘ + Y log 3 M Q = log 3 − log 3 ?, then − ln 0 is a
0 0 †
7
constant, so this becomes part of Y

35
Prepared by: Nathaniel M. Cabansay, BSCpE

h h ‚ log 3 † = ? log3 . Square root represents


X •¤Ž••• M Q Ch = h •¤Ž••• − €•|‚G + hG | + b
‚ ‚ G
8


.
an exponent of

INTEGRATION BY PARTS

%
No. STATEMENT REASON

X arctan M Q )%
0
1 Given

% % %
= arccos M Q and
/
X arctan M Q )% = % arctan − 0 X . )% + Y
0 0 0 + %. 3
2
) = )% . This gives us ) = 3 )% and
3
Integrate by parts: Let

8/
=%
% % 0 1 Integrate by substitution: Let > = 0. + % . so
X arctan M Q )% = % arctan − X )> + Y
0 0 2 > )> = 2% )% or )> = % )%
3
.

% % 0 1
X arctan )% = % arctan − ln|>| + Y X )% = ln|%| + Y
0 0 2 %
4

h h ‚ Substitute back > = 0. + % .


X •¤Ž••• M Q Ch = h •¤Ž••• − €•|‚G + hG | + b
‚ ‚ G
5

INTEGRAL OF ARCCOSECANT

%
No. STATEMENT REASON

X arccsc )%
0
1 Given

2 X ) = −W +Y Integral of an inverse function

1 % % % % % % = arccsc % , = ,
/
X arccsc )% = arccsc − ln ‘csc Marccsc Q − cot Marccsc Q‘
0 0 0 0 0 0
3
and ) = )%, this means =
3 Let

+Y 3
csc % and W = ln|csc % −
1 % % % % %
X arccsc )% = arccsc + ln ‘csc Marccsc Q + cot Marccsc Q‘ cot %| = − ln|csc % + cot %|
0 0 0 0 0 0
+Y
% % % % Multiply both sides by 0
X arccsc )% = % arccsc − 0 ln ‘csc Marccsc Q − cot Marccsc Q‘
0 0 0 0
4

+Y
% % % %
X arccsc )% = % arccsc + 0 ln ‘csc Marccsc Q + cot Marccsc Q‘
0 0 0 0
+Y

36
Prepared by: Nathaniel M. Cabansay, BSCpE

% % % .
X arccsc )% = % arccsc % − 0 ln ž − ŸM Q − 1ž + Y
5 Use this triangle and the

0 0 0
trigonometric ratio CAO

% % .
= % arccsc % + 0 ln ž + ŸM Q − 1ž + Y
0 0 1

7 . −1

% % % 1
X arccsc )% = % arccsc − 0 ln • − 7% . − 0. • + Y
0 0 0 0
3
6 Factor a from the radical

% % 1
= % arccsc + 0 ln • + 7% . − 0. • + Y
0 0 0

% % % − √% . − 0.
X arccsc )% = % arccsc − 0 ln ¦ ¦+Y
0 0 0
7 Combine into one fraction

% % + √% . − 0.
= % arccsc + 0 ln ¦ ¦+Y
0 0

h h
X •¤ŽŽŒŽ M Q Ch = h •¤ŽŽŒŽ − ‚ €• ‘h − 7hG − ‚G ‘ + b log 3 M Q = log 3 − log 3 ?,
‚ ‚ †
8
h then − ln 0 is a constant, so this
= h •¤ŽŽŒŽ + ‚ €• ‘h + 7hG − ‚G ‘ + b
‚ becomes part of Y

INTEGRAL OF ARCSECANT

INTEGRAL OF AN INVERSE FUNCTION

%
No. STATEMENT REASON

X arcsec M Q )%
0
1 Given

X ) = −W +Y
2 Integral of an inverse function

1 % % % % % % = arcsec % , =
X arcsec )% = arcsec − ln ‘sec Marcsec Q + tan Marcsec Q‘
0 0 0 0 0 0 /
, and ) = 3 )%, this means
3 Let

+Y 3
= sec % and W = ln|sec % +
1 % % % % % tan %| = − ln|sec % − tan %|
X arcsec )% = arcsec + ln ‘sec Marcsec Q − tan Marcsec Q‘
0 0 0 0 0 0
+Y
% % % % Multiply both sides by 0
X arcsec )% = % arcsec − 0 ln ‘sec Marcsec Q + tan Marcsec Q‘
0 0 0 0
4

+Y
% % % %
X arcsec )% = % arcsec + 0 ln ‘sec Marcsec Q − tan Marcsec Q‘
0 0 0 0
+Y

37

1
Prepared by: Nathaniel M. Cabansay, BSCpE

% % % .
X arcsec % )% = % arcsec − 0 ln ž + ŸM Q − 1ž + Y
5 Use this triangle and the

0 0 0
trigonometric ratio TOA

% % % .
= % arcsec + 0 ln ž − ŸM Q − 1ž + Y
0 0 0 7 . −1

% % 1
X arcsec % )% = % arcsec − 0 ln • + 7%. − 0. • + Y
0 0 0
3
6 Factor a from the radical

% % 1
= % arcsec + 0 ln • − 7%. − 0. • + Y
0 0 0

% % + √%. − 0.
X arcsec % )% = % arcsec − 0 ln ¦ ¦+Y
0 0
7 Combine into one fraction

% % − √%. − 0.
= % arcsec + 0 ln ¦ ¦+Y
0 0

h
X •¤ŽŒOŽ h Ch = h •¤ŽŒOŽ − ‚ €• ‘h + 7hG − ‚G ‘ + b log 3 M Q = log 3 − log 3 ?,
‚ †
8
h then − ln 0 is a constant, so
= h •¤ŽŒOŽ + ‚ €• ‘h − 7hG − ‚G ‘ + b
‚ this becomes part of Y

INTEGRAL OF ARCCOTANGENT

INTEGRAL OF AN INVERSE FUNCTION

%
No. STATEMENT REASON

X arccot M Q )%
0
1 Given

X ) = −W +Y
2 Integral of an inverse function

1 % % % % % = arccot % , = , and ) =
/
X arccot )% = arccot + ln ‘csc Marccot Q‘ + Y
0 0 0 0 0
3
)% , this means = cot % and W =
3 Let

3
− ln|csc %|
% % % Multiply both sides by 0
X arccot )% = % arccot + 0 ln ‘csc Marccot Q‘ + Y
0 0 0
4

% % % .
X arccot )% = % arccot + 0 ln žŸ1 + M Q ž + Y
5 Use this triangle and the trigonometric

0 0 0
ratio CHO

38
Prepared by: Nathaniel M. Cabansay, BSCpE

71 + .
1

% % 1
X arccot )% = % arccot + 0 ln • 70. + % . • + Y
0 0 0
3
6 Factor out a from the radical

% %
X arccot )% = % arccot + 0 ln ‘70. + % . ‘ + Y log 3 M Q = log 3 − log 3 ?, then − ln 0 is
0 0 †
7
a constant so it becomes part of Y
h h ‚ log 3 †
= ? log3
X •¤ŽŽƒ• M Q Ch = h •¤ŽŽƒ• + €•|‚G + hG | + b
‚ ‚ G
8

INTEGRATION BY PARTS

%
No. STATEMENT REASON

X arccot M Q )%
0
1 Given

% % %
= arccot M Q and
/
X arccot M Q )% = % arccot + 0 X . )% + Y
0 0 0 + %. 3
2
) = )% . This gives us ) = − 3 )%
3
Integrate by parts: Let

8/
=%
% % 0 1
and

Integrate by substitution: Let > = 0. + % .


X arccot M Q )% = % arccot + X )> + Y
0 0 2 > so )> = 2% )% or )> = % )%
3
.

% % 0 1
X arccot )% = % arccot + ln|>| + Y X )% = ln|%| + Y
0 0 2 %
4

h h ‚ Substitute back > = 0. + % .


X •¤ŽŽƒ• M Q Ch = h •¤ŽŽƒ• + €•|‚G + hG | + b
‚ ‚ G
5

EXAMPLES OF INTEGRALS OF INVERSE TRIGONOMETRIC FUNCTIONS

Find the indefinite integral of:

1
^ = arcsin z {
^

We can use the identity: arccsc = arcsin M Q

1
No. STATEMENT REASON

^ = arcsin z {
^
1 Given

39
Prepared by: Nathaniel M. Cabansay, BSCpE

^ = arccsc ^ 1
arccsc = arcsin z {
2

% %
X ^ )^ = ^ arccsc ^ − ln ‘^ − 7^ . − 1‘ + Y X arccsc )% = % arccsc − 0 ln ‘% − 7%. − 0. ‘
0 0
3

+Y
X ^ )^ = ^ arccsc ^ + ln ‘^ + 7^ . − 1‘ + Y %
= % arccsc + 0 ln ‘% + 7%. − 0. ‘
0
+Y
Find the indefinite integral of:

3
ℎ ] = arcsec z {
]

3
No. STATEMENT REASON

ℎ ] = arcsec z {
]
1 Given

] 1
ℎ ] = arccos z { arcsec = arccos z {
3
2

] % %
X ℎ ] )] = ] arccsc z { − 79 − ] . + Y X arccos )% = % arccos − 70. − %. + Y
3 0 0
3

40
Prepared by: Nathaniel M. Cabansay, BSCpE

MODULE 7 INTEGRALS OF H YPERBOLIC FUNCTIONS

INTEGRALS OF THE FIRST TWO HYPERBOLIC FUNCTIONS

Let us define the first two hyperbolic functions:

ev − e v
ev + e v
sinh ] = , cosh ] =
2 2

The integrals of these functions are as follows:

∫ Œ••§ h Ch = ŽƒŒ§ h + b (Integral of Hyperbolic Sine)


∫ ŽƒŒ§ h Ch = Œ••§ h + b (Integral of Hyperbolic Cosine)
1.
2.

DERIVING INTEGRALS OF THE FIRST TWO HYPERBOLIC FUNCTIONS

INTEGRAL OF HYPERBOLIC SINE

We can derive this in two ways:

REVERSE OF THE DERIVATIVE OF HYPERBOLIC COSINE

No. STATEMENT REASON


)
sinh % )% = cosh %
)
1 Derivative of hyperbolic cosine

X Œ••§ h Ch = ŽƒŒ§ h + b
2 Integrate both sides with respect to

EXPONENTIAL FORMS

No. STATEMENT REASON

X sinh % )%
1 Given

e/ − e /
¨v − ¨ v
X sinh % )% = X )% sinh ] =
2 2
2

1
X sinh % )% = X e/ − e /
)%
2
3 Integral Rule 3

1
X sinh % )% = zX e/ )% − X e /
)%{
2
4 Integral Rule 4

1 /
X sinh % )% = e +e /
+Y X e/ )% = e/ + Y
2
5

e/ + e /
X Œ••§ h Ch = ŽƒŒ§ h + b = cosh %
2
6

41
Prepared by: Nathaniel M. Cabansay, BSCpE

INTEGRAL OF HYPERBOLIC COSINE

We can also derive this in two ways:

REVERSE OF THE DERIVATIVE OF HYPERBOLIC SINE

No. STATEMENT REASON


)
cosh % )% = sinh %
)
1 Derivative of hyperbolic sine

X ŽƒŒ§ h Ch = Œ••§ h + b
2 Integrate both sides with respect to

EXPONENTIAL FORMS

No. STATEMENT REASON

X cosh % )%
1 Given

e/ + e /
¨v + ¨ v
X cosh % )% = X )% sinh ] =
2 2
2

1
X cosh % )% = X e/ + e /
)%
2
3 Integral Rule 3

1
X cosh % )% = zX e/ )% + X e /
)%{
2
4 Integral Rule 4

1 /
X cosh % )% = e −e /
+Y X e/ )% = e/ + Y
2
5

e/ − e /
X ŽƒŒ§ h Ch = Œ••§ h + b = sinh %
2
6

EXAMPLES OF INTEGRALS OF THE FIRST TWO HYPERBOLIC FUNCTIONS

Find the indefinite integral of:

= 2 sinh cosh

We can do this in three ways:

We can do substitution with sinh

No. STATEMENT REASON

1 = 2 sinh cosh Given

X ) = X 2 sinh cosh )
2 Integrate both sides with respect to

42
Prepared by: Nathaniel M. Cabansay, BSCpE

3 X ) = 2 X sinh cosh ) Integral Rule 3

Integrate by substitution: Let % = sinh and )% =


X ) = 2 X % )%
cosh )
4

5 X ) = %. + Y Integral Rule 2

Substitute back % = sinh


X ) = sinh. +Y
6

We can do substitution with cosh

No. STATEMENT REASON

1 = 2 sinh cosh Given

X ) = X 2 sinh cosh )
2 Integrate both sides with respect to

X ) = 2 X sinh cosh )
3 Integral Rule 3

Integrate by substitution: Let % = cosh and )% =


X ) = 2 X % )%
sinh )
4

X ) = %. + Y
5 Integral Rule 2

Substitute back % = sinh


X ) = cosh. +Y
6

We can use the identity 2 sinh cosh = sinh 2

No. STATEMENT REASON

1 = 2 sinh cosh Given

2 = sinh 2 2 sinh cosh = sinh 2

X ) = X sinh 2 )
3 Integrate both sides with respect to

1 Integrate by substitution: Let % = 2 and )% = 2 ) , so


X ) = X sinh % )%
2 ) = )%
4
.

cosh %
X ) = +Y X sinh % )% = cosh % + Y
2
5

cosh 2 Substitute back % = 2


X ) = +Y
2
6

43
Prepared by: Nathaniel M. Cabansay, BSCpE

We can now check to see how much these functions differ:

No. STATEMENT REASON

= cosh. − sinh.
cosh 2
1 Subtract one of the answers from another

= − cosh.
.
2
cos 2
=− − sinh.
@
2
= cosh. − sinh. For . and @ use the identity cosh 2 =
cosh + sinh cosh. + sinh.
2
. .
= − cosh.
.
2
cosh. + sinh.
= − sinh.
@
2
= cosh. − sinh. . and @ , combine into a single fraction.
− cosh − sinh
3 For
. . . factor a -1 from the numerator
=
2
For ,
.

cosh. − sinh.
=
@
2
=1
identity cosh. − sinh. = 1
5 For all constants, use the Pythagorean

1
=−
.
2
1
=
@
2

INTEGRALS WITH HYPERBOLIC RESULTS

These are the reverses of derivatives of hyperbolic functions.

∫ sech. % )% = tanh % + Y (Hyperbolic Tangent Result)


∫ csch % coth % )% = − csch % + Y (Hyperbolic Cosecant Result)
1.

∫ sech % tanh % )% = − sech % + Y (Hyperbolic Secant Result)


2.

∫ csch. % )% = − coth % + Y (Hyperbolic Cotangent Result)


3.
4.

DERIVING INTEGRALS WITH HYPERBOLIC RESULTS

HYPERBOLIC TANGENT RESULT

REVERSE OF THE DERIVATIVE OF HYPERBOLIC TANGENT

No. STATEMENT REASON


)
sech. % )% = tanh %
)
1 Given

44
Prepared by: Nathaniel M. Cabansay, BSCpE

2 X ŒOŽ§G h Ch = •••§ h + b Integrate both sides with respect to

HYPERBOLIC COSECANT RESULT

REVERSE OF THE DERIVATIVE OF HYPERBOLIC COSECANT

No. STATEMENT REASON


)
− csch % coth % )% = csch %
)
1 Given

)
csch % coth % )% = − csch %
)
2 Multiply both sides by -1

X ŽŒŽ§ h Žƒ•§ h Ch = − ŽŒŽ§ h + b


3 Integrate both sides with respect to

HYPERBOLIC SECANT RESULT

REVERSE OF THE DERIVATIVE OF THE HYPERBOLIC SECANT

No. STATEMENT REASON


)
− sech % tanh % )% = sech %
)
1 Given

)
sech % tanh % )% = − sech %
)
2 Multiply both sides by -1

X ŒOŽ§ h •••§ h Ch = − ŒOŽ§ h + b


3 Integrate both sides with respect to

HYPERBOLIC COTANGENT RESULT

REVERSE OF THE DERIVATIVE OF THE HYPERBOLIC COTANGENT

No. STATEMENT REASON


)
− csch. % )% = coth %
)
1 Given

)
csch. % )% = − coth %
)
2 Multiply both sides by -1

X ŽŒŽ§G h Ch = − Žƒ•§ h + b
3 Integrate both sides with respect to

45
Prepared by: Nathaniel M. Cabansay, BSCpE

EXAMPLES OF INTEGRALS WITH HYPERBOLIC RESULTS

Find the integral of:

= sech. − csch.

No. STATEMENT REASON

1 = sech. − csch. Given

2 X ) = X sech. − csch. ) Integrate both sides with respect to

X ) = X sech. ) − X csch. )
3 Integral Rule 4

X ) = tanh − − coth +Y ∫ sech. % )% = tanh % + Y and ∫ csch. % )% =


− coth % + Y
4

Multiply − coth
X ) = tanh + coth + Y
5 by -1

INTEGRALS OF REMAINING FOUR HYPERBOLIC FUNCTIONS

Here are the integrals of the remaining four hyperbolic functions:

∫ •••§ h Ch = €•|ŽƒŒ§ h| + b (Integral of Hyperbolic Tangent)


∫ ŽŒŽ§ h Ch = €• ‘•••§ MGQ‘ + b = − €•|ŽŒŽ§ F + Žƒ•§ F| + b = €•|Žƒ•§ F − ŽŒŽ§ F| + b =
1.
h

−G •¤Žƒ•§ Oh + b = − •¤Žƒ•§ ŽƒŒ§ h + b (Integral of Hyperbolic Cosecant)


2.

∫ ŒOŽ§ h Ch = •¤ŽŒ•• •••§ F + b = •¤Ž••• Œ••§ F + b = G •¤Ž••• Oh + b =


G •¤Ž••• M•••§ Q + b (Integral of Hyperbolic Secant)
3.
h
G
4. ∫ Žƒ•§ h Ch = €•|Œ••§ h| + b (Integral of Hyperbolic Cotangent)

Some of the forms for the integral of the hyperbolic cosecant have inverse hyperbolic functions, and will be
discussed in Module 8.

HYPERBOLIC TANGENT HALF-ANGLE SUBSTITUTIONS

In Module 5, we derived the tangent half-angle substitutions we can use to derive certain integrals. For hyperbolic
functions, they are done a similar way.

sinh 2 =
. “”5©
“”5©
1.

No. STATEMENT REASON

1 sinh 2 = sinh 2 Given

2 sinh 2 = 2 sinh cosh Double Angle Formula for Hyperbolic Sine

sinh 2 = 2 tanh cosh. •‡–© –—5©


= tanh
•‡–© •‡–©
3 Multiply the right side by , then

46
Prepared by: Nathaniel M. Cabansay, BSCpE

2 tanh 1
sinh 2 = cosh =
sech. sech
4

2 tanh Hyperbolic Pythagorean Identity: 1 − tanh. = sech.


sinh 2 =
1 − tanh.
5

cosh 2 =
8“”5©
“”5©
2.

No. STATEMENT REASON

1 cosh 2 = cosh 2 Given

2 cosh 2 = cosh. + sinh. Double Angle Formula for Hyperbolic Cosine

cosh 2 = 1 + tanh. cosh. •‡–© –—5©


= tanh
•‡–© •‡–©
3 Multiply the right side by , then

1 + tanh. 1
cosh 2 = cosh =
sech. sech
4

1 + tanh. Hyperbolic Pythagorean identity: 1 − tanh. = sech.


cosh 2 =
1 − tanh.
5


We also have this derivative we can use for such substitutions:

No. STATEMENT REASON

% = tanh
2
1 Substitution

2 = 2 artanh % Isolate by inversing the hyperbolic tangent and


multiplying both sides by 2
) 2 Differentiate both sides with respect to %
=
)% 1 − % .
3

DERIVING INTEGRALS OF REMAINING FOUR HYPERBOLIC FUNCTIONS

INTEGRAL OF HYPERBOLIC TANGENT

No. STATEMENT REASON

X tanh % )%
1 Given

sinh % sinh %
X tanh % )% = X )% tanh % =
cosh % cosh %
2

1 Integrate by substitution: Let > = cosh % and )> =


X tanh % )% = X )>
> sinh % )%
3

1
X tanh % )% = ln|>| + Y X )% = ln|%| + Y
%
4

47
Prepared by: Nathaniel M. Cabansay, BSCpE

5 X •••§ h Ch = €•|ŽƒŒ§ h| + b Substitute back > = cosh %

INTEGRAL OF HYPERBOLIC COSECANT

LOGARITHM FORMS

We can use csch % + coth %…

No. STATEMENT REASON

X csch % )%
1 Given

csch. % + csch % coth % •–•© /8•‡“© /


X csch % )% = X )%
csch % + coth %
•–•© /8•‡“© /
2 Multiply by

1 Integrate by substitution: Let > = csch % + coth % and


X csch % )% = − X )>
> )> = − csch % coth % − csch. % , so −)> = csch. % +
3

csch % coth %
1
X csch % )% = − ln|>| + Y X )% = ln|%| + Y
%
4

Substitute back > = csch % + coth %


X ŽŒŽ§ h Ch = − €•|ŽŒŽ§ h + Žƒ•§ h| + b
5


…or we can use coth % − csch %

No. STATEMENT REASON

X csch % )%
1 Given

csch % coth % − csch. % •‡“© / •–•© /


X csch % )% = X )%
coth % − csch %
2 Multiply by •‡“© / •–•© /

1 Integrate by substitution: Let > = coth % − csch % and


X csch % )% = X )>
> )> = csch % coth % − csch. %
3

1
X csch % )% = ln|>| + Y X )% = ln|%| + Y
%
4

Substitute back > = coth % − csch %


X ŽŒŽ§ h Ch = €•|Žƒ•§ h − ŽŒŽ§ h| + b
5

HYPERBOLIC TANGENT FORM

No. STATEMENT REASON

X csch % )%
1 Given

48
Prepared by: Nathaniel M. Cabansay, BSCpE

1 1
X csch % )% = X )% csch =
sinh % sinh
2

1 − > . 2 )> Let > = tanh and use the hyperbolic tangent half-angle
/
X csch % )% = X ⋅ .
2> 1 − >.
3
.
=
substitution so the derivative is

1
X csch % )% = X )>
>
4 Simplify the right-hand integral

1
X csch % )% = ln|>| + Y X )% = ln|%| + Y
%
5

h Substitute back > = tanh


/
X ŽŒŽ§ h Ch = €• ‘•••§ ‘ + b
G
.
6


Two other forms for the integral of the hyperbolic cosecant will be derived in Module 8.

INTEGRAL OF HYPERBOLIC SECANT

ARCSINE FORM

No. STATEMENT REASON

X sech % )%
1 Given

sech. % – •© /
X sech % )% = X )%
sech %
– •© /
2 Multiply the right-hand side by

sech. % Hyperbolic Pythagorean Identity: 1 − tanh. = sech. .


X sech % )% = X )%
√1 − tanh. %
3
Get its square root
1 Integrate by substitution: Let > = tanh % and )> =
X sech % )% = X )>
√1 − >. sech. % )%
4

1
X sech % )% = arcsin > + Y X )% = arcsin % + Y
√1 − % .
5

Substitute back > = tanh %


X ŒOŽ§ h Ch = •¤ŽŒ•• •••§ h + b
6

ARCTANGENT OF HYPERBOLIC SINE FORM

No. STATEMENT REASON

X sech % )%
1 Given

1 1
X sech % )% = X )% sech =
cosh % cosh
2

cosh % •‡–© /
X sech % )% = X )%
cosh. %
•‡–© /
3 Multiply the right-hand side by

49
Prepared by: Nathaniel M. Cabansay, BSCpE

cosh % Hyperbolic Pythagorean Identity: cosh. − sinh. =1


X sech % )% = X )%
1 + sinh. %
4

1 Integrate by substitution: Let > = sinh % and )> =


X sech % )% = X )>
1 + >. cosh % )%
5

1
X sech % )% = arctan > + Y X )% = arctan % + Y
1 + %.
6

Substitute back > = sinh %


X ŒOŽ§ h Ch = •¤Ž••• Œ••§ h + b
7

ARCTANGENT OF EXPONENTIAL FORM

No. STATEMENT REASON

X sech % )%
1 Given

2
X sech % )% = X )%
e/ + e /
2 Exponential form

2e/ ‹
X sech % )% = X )%
e./ +1

3 Multiply the right-hand integral by

1 Integrate by substitution: Let > = e/ and )> = e/ )%


X sech % )% = 2 X )>
>. +1
4

1
X sech % )% = 2 arctan > + Y X )% = arctan % + Y
1 + %.
5

6 X ŒOŽ§ h Ch = G •¤Ž••• Oh + b Substitute back > = e/

ARCTANGENT OF HALF HYPERBOLIC TANGENT FORM

No. STATEMENT REASON

X sech % )%
1 Given

1 1
X sech % )% = X )% sech =
cosh % cosh
2

1 − > . 2 )> Let > = tanh and use the hyperbolic tangent half-angle
/
X sech % )% = X ⋅ .
1 + >. 1 − > .
3
.
=
substitution so the derivative is

1
X sech % )% = 2 X )>
1 + >.
4 Simplify

1
X sech % )% = 2 arctan > + Y X )% = arctan % + Y
1 + %.
5

h
Substitute back > = tanh M Q
/
X ŒOŽ§ h Ch = G •¤Ž••• M•••§ Q + b
G .
6

50
Prepared by: Nathaniel M. Cabansay, BSCpE

INTEGRAL OF HYPERBOLIC COTANGENT

No. STATEMENT REASON

X coth % )%
1 Given

cosh % cosh %
X coth % )% = X )% coth % =
sinh % sinh %
2

1 Integrate by substitution: Let > = sinh % and )> =


X coth % )% = X )>
> cosh % )%
3

1
X coth % )% = ln|>| + Y X )% = ln|%| + Y
%
4

Substitute back > = sinh %


X Žƒ•§ h Ch = €•|Œ••§ h| + b
5

EXAMPLES OF INTEGRALS OF HYPERBOLIC FUNCTIONS

Find the integral of:

ℎ = sech. coth

No. STATEMENT REASON

1 ℎ = sech. coth Given

Xℎ ) = X sech. coth )
2 Integrate both sides with respect to

sech. 1
Xℎ ) =X ) coth =
tanh tanh
3

1 Integrate by substitution: Let % = tanh and )% =


Xℎ ) =X )%
% sech. )
4

1
Xℎ ) = ln|%| + Y X )% = ln|%| + Y
%
5

Substitute back % = tanh


Xℎ ) = ln|tanh | + Y
6

51
Prepared by: Nathaniel M. Cabansay, BSCpE

MODULE 8 INTEGRALS OF INVERSE HYPERBOLIC FUNCTIONS

INTEGRALS WITH INVERSE HYPERBOLIC RESULTS

These are just the reverses of some of the derivatives of inverse hyperbolic functions

∫7 Ch = •¤Œ••§ M Q + b = €• h + √hG + ‚G + b (Inverse Hyperbolic Sine Result)


P h
hG 8‚G ‚
1.

∫7 Ch = •¤ŽƒŒ§ M Q + b = €• h + √hG − ‚G + b (Inverse Hyperbolic Cosine Result)


P h
hG ‚G ‚
2.

∫ ‚G Ch = •¤•••§ M Q + b = €• M Q + b, hG < ‚G (Inverse Hyperbolic Tangent Result)


P P h P ‚8h
hG ‚ ‚ G‚ ‚ h
3.

∫ Ch = − •¤ŽŒŽ§ M Q + b = − €• z { + b (Inverse Hyperbolic Cosecant Result)


P P h P ‚87hG 8‚G
h7hG 8‚G ‚ ‚ ‚ h
4.

∫ Ch = − •¤ŒOŽ§ M Q + b = €• z { + b (Inverse Hyperbolic Secant Result)


P P h P ‚87‚G hG
h7‚G hG ‚ ‚ ‚ h
5.

∫ hG Ch = − •¤Žƒ•§ M Q + b = €• M Q + b, hG > ‚G (Inverse Hyperbolic Cotangent Result)


P P h P h ‚
‚G ‚ ‚ G‚ h8‚
6.

DERIVING INTEGRALS WITH INVERSE HYPERBOLIC RESULTS

INVERSE HYPERBOLIC SINE RESULT


First, we’ll reverse the inverse hyperbolic sine derivative…

No. STATEMENT REASON


1 )
)% = arsinh %
√%. + 1 )
1 Derivative of inverse hyperbolic sine

1 ) % Divide the % on both sides by 0. Since 0 is a constant,


)% = arsinh M Q
% . ) 0
2

0 œM Q + 1
0
use Derivative Rule 3

1 ) % Bring the 0 inside the radical to simplify it


)% = arsinh M Q
√% . + 0. ) 0
3

P h
X Ch = •¤Œ••§ M Q + b
√hG + ‚G ‚
4 Integrate both sides with respect to


…then, we’ll convert to logarithmic form.

1 %
No. STATEMENT REASON

X )% = arsinh M Q + Y
√% . + 0. 0
1 Inverse hyperbolic sine result

1 % % .
X )% = ln ª + ŸM Q + 1« + Y
2 Logarithmic form of inverse hyperbolic sine

√%. + 0. 0 0

1 % 1
X )% = ln z + 7% . + 0. { + Y
√% . + 0. 0 0
3
3 Pull a outside the radical

52
Prepared by: Nathaniel M. Cabansay, BSCpE

1 % + √% . + 0.
X )% = ln j k+Y
√%. + 0. 0
4 Combine into one fraction

1
X )% = ln M% + 7% . + 0. Q − ln 0 + Y log 3 z { = log 3 − log 3 ?
√%. + 0. ?
5

P ln 0 is a constant
X Ch = €• Mh + 7hG + ‚G Q + b
√hG + ‚G
6

INVERSE HYPERBOLIC COSINE RESULT


First, we’ll reverse the inverse hyperbolic cosine derivative…

No. STATEMENT REASON


1 )
)% = arcosh %
√% . − 1 )
1 Derivative of inverse hyperbolic sine

1 ) % Divide the % on both sides by 0. Since 0 is a constant,


)% = arcosh M Q
% . ) 0
2

0 œM Q − 1
0
use Derivative Rule 3

1 ) % Bring the 0 inside the radical to simplify it


)% = arcosh M Q
√%. − 0. ) 0
3

P h
X Ch = •¤ŽƒŒ§ M Q + b
√hG − ‚G ‚
4 Integrate both sides with respect to


…then, we’ll convert to logarithmic form.

1 %
No. STATEMENT REASON

X )% = arcosh M Q + Y
√% . − 0. 0
1 Inverse hyperbolic cosine result

1 % % .
X )% = ln ª + ŸM Q − 1« + Y
2 Logarithmic form of inverse hyperbolic cosine

√%. − 0. 0 0

1 % 1
X )% = ln z + 7% . − 0. { + Y
− 0. 0 0
3
√% .
3 Pull a outside the radical

1 % + √% . − 0.
X )% = ln j k+Y
√%. − 0. 0
4 Combine into one fraction

1
X )% = ln M% + 7% . − 0. Q − ln 0 + Y log 3 z { = log 3 − log 3 ?
√%. − 0. ?
5

P ln 0 is a constant
X Ch = €• Mh + 7hG − ‚G Q + b
√hG − ‚G
6

53
Prepared by: Nathaniel M. Cabansay, BSCpE

INVERSE HYPERBOLIC TANGENT RESULT

First, we’ll reverse the inverse hyperbolic tangent derivative…

No. STATEMENT REASON


1 )
)% = artanh %
1 − %. )
1 Given

1 ) % Divide the % on both sides by 0. Since 0 is a constant,


)% = artanh M Q
% .
) 0
0 z1 − M Q {
2

0
use Derivative Rule 3

1 ) 1 %
)% = artanh M Q
% . ) 0 0
3
0. z1 − M Q {
3 Divide both sides by

0
1 ) 1 %
)% = artanh M Q
0. −% . ) 0 0
4 Simplify the left-hand side

P P h
X Ch = •¤•••§ M Q + b
‚G −h G ‚ ‚
5 Integrate both sides with respect to


…then convert to logarithmic forms

1 1 %
No. STATEMENT REASON

X )% = artanh M Q + Y
0. − % . 0 0
1 Inverse hyperbolic tangent result

%
1 1 1+
X . )% = ln ª 0
%« + Y
2 Logarithmic form of inverse hyperbolic tangent
0 − %. 20 1−0

P P ‚+h 3
X Ch = ۥ z {+b
‚G − hG G‚ ‚−h
3
3 Multiply the expression inside the logarithm by

INVERSE HYPERBOLIC COSECANT RESULT


First, we’ll reverse the inverse hyperbolic cosecant derivative…

No. STATEMENT REASON


1 )
)% = arcsch %
%√%. +1 )
1 Given

1 ) % Divide the % on both sides by 0. Since 0 is a constant,


)% = arcsch M Q
% ) 0 use derivative rule 3, and the 0 cancels out with the
/
2
.
% œM Q + 1
0
3

1 ) 1 % Divide both sides by 0


)% = z arcsch M Q{
% ) 0 0
3
.
0% œM Q + 1
0
1 ) 1 % Bring the 0 inside the radical on the left side
)% = z arcsch M Q{
%√% . + 0. ) 0 0
4

54
Prepared by: Nathaniel M. Cabansay, BSCpE

P P h
X Ch = •¤ŽŒŽ§ M Q + b
h√hG + ‚G ‚ ‚
5 Integrate both sides with respect to

…then convert to logarithmic forms.

1 1 %
No. STATEMENT REASON

X )% = arcsch M Q + Y
%√% . + 0. 0 0
1 Given

1 1 ⎛ 1 1 ⎞
2 Logarithmic form of inverse hyperbolic

X )% = ln + + 1⎟ + Y
%√% . + 0. 0 ⎜ M% Q ¯ % .
cosecant

0 M0 Q
⎝ ⎠

1 1 0 0.
X )% = ln ª + Ÿ . + 1« + Y
3 Rewrite as reciprocals

%√%. + 0. 0 % %

1 1 0 1
X )% = ln z + 70. + %. { + Y
%√%. + 0. 0 % %
/
4 Pull out a from the radical

P P ‚ + √‚G + hG
X Ch = ۥ j k+b
h√hG + ‚G ‚ h
5 Combine into one fraction

INVERSE HYPERBOLIC SECANT RESULT

First, we’ll reverse the inverse hyperbolic secant derivative…

No. STATEMENT REASON


1 )
)% = arsech %
%√%. −1 )
1 Given

1 ) % Divide the % on both sides by 0. Since 0 is a constant,


)% = arsech M Q
% ) 0 use derivative rule 3, and the 0 cancels out with the
/
2

% œM 0 Q − 1
.
3

1 ) 1 % Divide both sides by 0


)% = z arsech M Q{
% ) 0 0
3
.
0% œM Q − 1
0
1 ) 1 % Bring the 0 inside the radical on the left side
)% = z arsech M Q{
%√% . − 0. ) 0 0
4

P P h
X Ch = •¤ŒOŽ§ M Q + b
h√hG − ‚G ‚ ‚
5 Integrate both sides with respect to


…then convert to logarithmic forms.

55
Prepared by: Nathaniel M. Cabansay, BSCpE

1 1 %
No. STATEMENT REASON

X )% = arsech M Q + Y
%√% . − 0. 0 0
1 Given

1 1 ⎛ 1 1 ⎞
2 Logarithmic form of inverse hyperbolic

X )% = ln ⎜ % + ¯ . − 1⎟ + Y
%√% − 0 0 %
secant
. .
M0 Q M0 Q
⎝ ⎠

1 1 0 0.
X )% = ln ª + Ÿ . − 1« + Y
3 Rewrite as reciprocals

%√%. − 0. 0 % %

1 1 0 1
X )% = ln z + 70. − %. { + Y
%√%. − 0. 0 % %
/
4 Pull out a from the radical

P P ‚ + √‚G − hG
X Ch = ۥ j k+b
h√hG − ‚G ‚ h
5 Combine into one fraction

INVERSE HYPERBOLIC COTANGENT RESULT

First, we’ll reverse the inverse hyperbolic cotangent derivative…

No. STATEMENT REASON


1 )
)% = coth %
1 − %. )
1 Given

1 )
)% = − coth %
%. −1 )
2 Multiply both sides by -1

1 ) % Divide the % on both sides by 0. Since 0 is a constant,


)% = M− coth M QQ
% .
) 0
0 zM Q − 1{
3

0
use Derivative Rule 3

1 ) 1 %
)% = z− coth M Q{
% ) 0 0
Divide both sides by 3
.
0. zM Q − 1{
4

0
1 ) 1 %
)% = z− coth M Q{
%. −0 . ) 0 0
5 Simplify the left-hand side

P P h
X Ch = − Žƒ•§ M Q + b
hG − ‚G ‚ ‚
6 Integrate both sides with respect to

…then convert to logarithmic forms

1 1 %
No. STATEMENT REASON

X )% = − coth M Q + Y
%. −0 . 0 0
1 Inverse hyperbolic cotangent result

56
Prepared by: Nathaniel M. Cabansay, BSCpE

%
1 1 +1
X . 0
)% = − ln ª% «+Y
2 Logarithmic form of inverse hyperbolic cotangent
% −0 . 20 −1
0
1 1 %+0 3
X )% = − ln z {+Y
%. − 0. 20 %−0
3
3 Multiply the expression inside the logarithm by

P P h−‚ ? log 3 = log 3 †


X Ch = ۥ M Q+b
hG −‚ G G‚ h+‚
4

EXAMPLES OF INTEGRALS WITH INVERSE HYPERBOLIC RESULTS

1
Find the indefinite integral of:

=
√ . − 16

1
No. STATEMENT REASON

=
√ − 16
1 Given
.

1
X ) =X )
√ − 16
2 Integrate both sides with respect to
.

∫7 )% = arcosh M Q + Y . 0 in this case is 4.


/
X ) = arcosh M Q + Y
4
/ 3 3
3

Find another two forms of the indefinite integral of csch %

Wait… this was supposed to be back in Module 7, isn’t it? Well, the methods to be used here are from this
module, so we included it in this module instead. Here’s one of these forms:

No. STATEMENT REASON


csch %
2
1 Given

csch % =
e/ − e /
2 Exponential form

2 Integrate both sides with respect to %


X csch % )% = X )%
e/ − e ³
3

2e/ ‹
X csch % )% = X )%
e./ −1

4 Multiply the right-hand side by

1 Integrate by substitution: Let > = e/ so )> = e/ )% and


X csch % )% = 2 X )>
>. −1
5
use Integral Rule 3

∫/ )% = − arcoth M Q + Y , where 0 = 1
/
X csch % )% = −2 arcoth > + Y 3 3 3
6

Substitute back > = e/


X csch % )% = −2 arcoth e/ + Y
7

57
Prepared by: Nathaniel M. Cabansay, BSCpE

…and here’s another one:

No. STATEMENT REASON

csch %
1
1 Given

csch % =
sinh %
2 Reciprocal form

1 Integrate both sides with respect to %


X csch % )% = X )%
sinh %
3

sinh % –—5© /
X csch % )% = X )%
sinh. %
–—5© /
4 Multiply the right-hand side by

sinh % cosh. % − 1 = sinh. %


X csch % )% = X )%
cosh. % − 1
5

1 Integrate by substitution: Let > = cosh % so )> =


X csch % )% = X )>
>. −1 sinh % )%
6

∫/ )% = − arcoth M Q + Y , where 0 = 1
/
X csch % )% = − arcoth > + Y 3 3 3
7

8 X csch % )% = − arcoth cosh % + Y Substitute back > = cosh %

INTEGRALS OF INVERSE HYPERBOLIC FUNCTIONS


These use integration by parts:

∫ •¤Œ••§ M‚Q Ch = h •¤Œ••§ M‚Q − √hG + ‚G + b (Integral of Inverse Hyperbolic Sine)


h h
1.

∫ •¤ŽƒŒ§ M‚Q Ch = h •¤ŽƒŒ§ M‚Q − √hG − ‚G + b (Integral of Inverse Hyperbolic Cosine)


h h
2.

∫ •¤•••§ M‚Q Ch = h •¤•••§ M‚Q + G €•|‚G − hG | + b (Integral of Inverse Hyperbolic Tangent)


h h ‚
3.

h •¤ŽŒŽ§ M Q + ‚ •¤Œ••§ M Q + b h>a


h h

∫ •¤ŽŒŽ§ M‚Q Ch = …
h ‚ ‚

h •¤ŽŒŽ§ M Q − ‚ •¤Œ••§ M Q + b h<a


h h
4. (Integral of Inverse Hyperbolic
‚ ‚

∫ •¤ŒOŽ§ M‚Q Ch = h •¤ŒOŽ§ M‚Q + ‚ •¤ŽŒ•• M‚Q + b (Integral of Inverse Hyperbolic Secant)
Cosecant)
h h h
5.

∫ •¤Žƒ•§ M‚Q Ch = h •¤Žƒ•§ M‚Q + G €•|hG − ‚G | + b (Integral of Inverse Hyperbolic Cotangent)


h h ‚
6.

DERIVING INTEGRALS OF INVERSE HYPERBOLIC FUNCTIONS

INTEGRAL OF INVERSE HYPERBOLIC SINE

%
No. STATEMENT REASON

X arsinh M Q )%
0
1 Given

58
Prepared by: Nathaniel M. Cabansay, BSCpE

% % %
= arsinh M Q and
/
X arsinh M Q )% = % arsinh M Q − X )% + Y
0 0 √% + 0.
. 3
2
) = )% , so ) = )% and =%
Integrate by parts: Let

7/ 83

% % > Integrate by substitution: Let > . = % . +


X arsinh M Q )% = % arsinh M Q − X )> + Y
0 0 > 0. , so 2> )> = 2% )% or > )> = % )% and
3

> = √%. + 0.
% % Divide > by >
X arsinh M Q )% = % arsinh M Q − X 1 )> + Y
0 0
4

% %
X arsinh M Q )% = % arsinh M Q − > + Y
0 0
5 Integral Rule 2-bis: Integral of 1

h h Substitute back > = √% . + 0.


X •¤Œ••§ M Q Ch = h •¤Œ••§ M Q − 7hG + ‚G + b
‚ ‚
6

INTEGRAL OF INVERSE HYPERBOLIC COSINE

%
No. STATEMENT REASON

X arcosh M Q )%
0
1 Given

% % %
= arcosh M Q and
/
X arcosh M Q )% = % arcosh M Q − X )% + Y
0 0 √% − 0.
. 3
2
) = )% , so ) = )% and =%
Integrate by parts: Let

7/ 3

% % > Integrate by substitution: Let > . = %. − 0. ,


X arcosh M Q )% = % arcosh M Q − X )> + Y
0 0 > so 2> )> = 2% )% or > )> = % )% and > =
3

√% . − 0.
% % Divide > by >
X arcosh M Q )% = % arcosh M Q − X 1 )> + Y
0 0
4

% %
X arcosh M Q )% = % arcosh M Q − > + Y
0 0
5 Integral Rule 2-bis: Integral of 1

h h Substitute back > = √%. − 0.


X •¤ŽƒŒ§ M Q Ch = h •¤ŽƒŒ§ M Q − 7hG − ‚G + b
‚ ‚
6

INTEGRAL OF INVERSE HYPERBOLIC TANGENT

%
No. STATEMENT REASON

X artanh M Q )%
0
1 Given

% % %
= artanh M Q and
/
X artanh M Q )% = % artanh M Q − 0 X . )% + Y
0 0 0 − %. 3
2
) = )% , so ) = )% and =%
3
Integrate by parts: Let

3 /

% % 1 Integrate by substitution: Let > = 0. − % . ,


X artanh M Q )% = % artanh M Q − 0 X )> + Y
0 0 −2> so )> = −2% )%
3

59
Prepared by: Nathaniel M. Cabansay, BSCpE

% % 0 1
X artanh M Q )% = % artanh M Q + X )> + Y
0 0 2 >
4 Integral Rule 3

% % 0 1
X artanh M Q )% = % artanh M Q + ln|>| + Y X )% = ln|%| + Y
0 0 2 %
5

h h ‚ Substitute back > = 0. − %.


X •¤•••§ M Q Ch = h •¤•••§ M Q + €•|‚G − hG | + b
‚ ‚ G
6

INTEGRAL OF INVERSE HYPERBOLIC COSECANT

%
No. STATEMENT REASON

X arcsch M Q )%
0
1 Given

% % % =
X arcsch M Q )% = % arcsch M Q + 0 X )% + Y
0 0 |%|√0. + % . arcsch M Q and ) = )%, so ) =
/
2 Integrate by parts: Let

3
)% and =%
3
/ 73 /

% Divide % by % and take the


X arcsch M Q )%
0
3

% 1
⎧% arcsch M Q + 0 X )% + Y %>0
definition of the absolute value.

0 √0 . + % .
=
⎨% arcsch M%Q − 0 X 1
)% + Y %<0
⎩ 0 √0 + % .
.

h h 1 %
h h •¤ŽŒŽ§ M Q + ‚ •¤Œ••§ M Q + b h > a X )% = arsinh M Q + Y
X •¤ŽŒŽ§ M Q Ch = · ‚ ‚ √0. + %. 0
4

‚ h h
h •¤ŽŒŽ§ M Q − ‚ •¤Œ••§ M Q + b h < a
‚ ‚

INTEGRAL OF INVERSE HYPERBOLIC SECANT

%
No. STATEMENT REASON

X arsech M Q )%
0
1 Given

% % %
= arsech M Q and
/
X arsech M Q )% = % arsech M Q + 0 X )%
0 0 %√0 − % .
. 3
2
) = )% , so ) = )% and =%
3
Integrate by parts: Let

+Y / 73 /

% % 1 Divide % by %
X arsech M Q )% = % arsech M Q + 0 X )%
0 0 √0. − %.
3

+Y
h h h 1 %
X •¤ŒOŽ§ M Q Ch = h •¤ŒOŽ§ M Q + ‚ •¤ŽŒ•• M Q + b X )% = arcsin M Q + Y
‚ ‚ ‚ √0. − %. 0
4

60
Prepared by: Nathaniel M. Cabansay, BSCpE

INTEGRAL OF INVERSE HYPERBOLIC COTANGENT

%
No. STATEMENT REASON

X arcoth M Q )%
0
1 Given

% % %
= arcoth M Q and
/
X arcoth M Q )% = % arcoth M Q + 0 X . )% + Y
0 0 % − 0. 3
2
) = )% , so ) = − )% and =%
3
Integrate by parts: Let

/ 3

% % 1 Integrate by substitution: Let > = % . − 0. ,


X arcoth M Q )% = % arcoth M Q + 0 X )> + Y
0 0 2> so )> = 2% )%
3

% % 0 1
X arcoth M Q )% = % arcoth M Q + X )> + Y
0 0 2 >
4 Integral Rule 3

% % 0 1
X arcoth M Q )% = % arcoth M Q + ln|>| + Y X )% = ln|%| + Y
0 0 2 %
5

h h ‚ Substitute back > = % . − 0.


X •¤Žƒ•§ M Q Ch = h •¤Žƒ•§ M Q + €•|hG − ‚G | + b
‚ ‚ G
6

EXAMPLES OF INTEGRALS OF INVERSE HYPERBOLIC FUNCTIONS


Find the indefinite integral of:

¸ = arsinh z {
3


No. STATEMENT REASON

¸ = arsinh z {
3
1 Given

2¸ Integrate both sides with respect to ¸


X ¸ )¸ = X arsinh z { )¸
3
2

∫ arsinh M3 Q )% = % arsinh M3 Q − √%. + 0. + Y ,


/ /
2¸ 3 .
X ¸ )¸ = ¸ arsinh z { − Ÿ¸ . + z { + Y
3
where 0 =
@
3 2 .

61
Prepared by: Nathaniel M. Cabansay, BSCpE

CHAPTER 3 DEFINITE INTEGRALS

MODULE 9 RIEMANN SUMS

REVIEWING SUMMATION NOTATION


“Wait a minute, I thought this was going to be about definite integrals?”, many of you might ask. We’ll come to
that soon enough since this will be quite important later on.

Now, summation notation or sigma notation is a more compact way of writing a sum of a number of terms. It is
written as follows:

¹ 0º = 0 + 0 . + 0 @ + ⋯ + 0
º»

Here, ½ is the index of summation, 0º is the ½th term of the sum, and is the upper bound of the summation. Take

Also, both the upper and lower bounds must be constant with respect to ½.
note that the lower bound does not always need to be 1. As long as it is less than the upper bound, we’re fine.

For example: ∑rº» ½ = 1 + 2 + 3 + 4 = 10, ∑º» 2½ − 1 = 1 + 3 + 5 + 7 + ⋯ = .

THE AREA PROBLEM

is a continuous, non-negative function defined on an interval 0, \ , we want to


The area problem basically asks us to approximate the area under a curve. Suppose

= = 1 and = 3,
determine the area of the region between and the x-axis. For example, we have

.
the function: and we are asked to find the area between
as we see on the right. We can’t do this exactly yet, but we can get estimates if we
divide the area into rectangles of size:

\−0
Δ =

In each interval, we can form a rectangle with the height given by the value of the
function at a point in the interval. We can then find the area of each rectangle and add
them up to estimate the area.

Suppose we divide into 4 rectangles so we get a width of 0.5, then use the right
endpoint of each interval to define the height of the rectangle, as seen in the figure on

1 1 1 1
the right, this gives us:

¿= 1.5 + 2 + 2.5 + 3 = 0.5625 + 1 + 1.5625 + 2.25


2 2 2 2
= 5.375

This will overestimate the area, as some of that area is going to be above the curve.
This summation is known as the right Riemann sum.

62
Prepared by: Nathaniel M. Cabansay, BSCpE

Now suppose we use the left endpoints of each interval to define the height of the
rectangle, as seen in the figure on the right, this gives us:

1 1 1 1
¿= 1 + 1.5 + 2 + 2.5 = 0.25 + 0.5625 + 1 + 1.5625
2 2 2 2
= 3.375

This will underestimate the area, as some of the area below the curve is not going to be
covered. This summation is known as the left Riemann sum.

Now suppose we use the midpoints of each interval to define the height of the rectangle,
as seen in the figure of the right, this gives us:

1 1 1 1
¿= 1.25 + 1.75 + 2.25 + 2.75 = 4.3125
2 2 2 2

We are starting to get a better estimate of the area, since some area will be above and
some area will be below the curve. This summation is known as the midpoint Riemann
sum.

We will get more accurate answers if we increase the number of sub-intervals. Increasing the sub-intervals to 10,
for example, will produce more accurate answers than having just 4 sub-intervals.

À 0 on 0, \ and we divide the interval into sub-intervals of length Δ =


[ 3

= 0, = 0+Δ , . = 0 + 2Δ , … , º = 0 + ½Δ ,… =0+ −1 Δ , =
So, if we have , the endpoints
q
0 + Δ = \. We then choose a point , ., … , º , …
of each subinterval are at:
∗ ∗ ∗ ∗
in those sub-intervals
to define the height of the
rectangle. These points are not necessarily the same point in each subinterval, but are usually the left endpoints,
the right endpoints, or the midpoints of each interval.

The area under the curve on the interval is then approximated by:

¿≈ ∗
Δ + ∗
. Δ + ⋯+ ∗
º Δ +⋯+ ∗
Δ

We can then use summation notation to rewrite it as:

¿≈¹ ∗
º Δ
º»

To get a better estimation, we increase without bound. If we let go to infinity, we get the exact area. Thus:

¿ = lim ¹ ∗
º Δ
→Å
º»

63
Prepared by: Nathaniel M. Cabansay, BSCpE

NEGATIVE AND NET AREAS

Note that in the example above, we evaluated an area under the curve which is above the x-axis. Suppose we
evaluate an area between the x-axis and a curve or function below it, we are evaluating an area over the curve
below the x-axis, so we will get a negative area in this case. The negative sign for the area only implies that we are
doing things upside down, or getting the area over a curve rather than under it.

If the curve crosses the x-axis, we have both a positive and a negative area. In the figure below, we see a negative
area (in orange), and a positive area (in blue)

The orange area is a negative area since this is the area between the x-axis and the curve below it. Since we were
asked to look for the area below a curve, and we are getting above the curve, we inverse the sign as we are doing
the inverse process.

Now this curve crosses the x-axis, so we get the net area by adding the area in blue and the area in orange, which
is known as the net area.

EXAMPLES OF RIEMANN SUMS

Estimate the net area of the region between = @


−2 .
+ 4 on 0,1 using = 10, using:

a. The right Riemann sum


b. The left Riemann sum
c. The midpoint Riemann sum

64
Prepared by: Nathaniel M. Cabansay, BSCpE

a. Right Riemann sum

No. STATEMENT REASON

1 = @
−2 .
+ 4, = 10 Given
\−0
Δ =
2 Widths of rectangles

1−0
Δ =
10
Δ = 0.1
¿= Δ
¿= 0.1 ⋅ 0.1 = 3.981 ⋅ 0.1 = 0.3981
3 Heights of rectangles

¿= 0.2 ⋅ 0.1 = 3.928 ⋅ 0.1 = 0.3928


¿= 0.3 ⋅ 0.1 = 3.847 ⋅ 0.1 = 0.3847
¿= 0.4 ⋅ 0.1 = 3.744 ⋅ 0.1 = 0.3744
¿= 0.5 ⋅ 0.1 = 3.625 ⋅ 0.1 = 0.3625
¿= 0.6 ⋅ 0.1 = 3.496 ⋅ 0.1 = 0.3496
¿= 0.7 ⋅ 0.1 = 3.363 ⋅ 0.1 = 0.3363
¿= 0.8 ⋅ 0.1 = 3.232 ⋅ 0.1 = 0.3232
¿= 0.9 ⋅ 0.1 = 3.109 ⋅ 0.1 = 0.3109
¿= 1 ⋅ 0.1 = 3 ⋅ 0.1 = 0.3
4 ¿ = 0.3981 + 0.3928 + ⋯ + 0.3 = 3.5325 Add all areas

b. Left Riemann sum

No. STATEMENT REASON

1 = @
−2 .
+ 4, = 10 Given
\−0
Δ =
2 Widths of rectangles

1−0
Δ =
10
Δ = 0.1
¿= Δ
¿= 0 ⋅ 0.1 = 4 ⋅ 0.1 = 0.4
3 Heights of rectangles

¿= 0.1 ⋅ 0.1 = 3.981 ⋅ 0.1 = 0.3981


¿= 0.2 ⋅ 0.1 = 3.928 ⋅ 0.1 = 0.3928
¿= 0.3 ⋅ 0.1 = 3.847 ⋅ 0.1 = 0.3847
¿= 0.4 ⋅ 0.1 = 3.744 ⋅ 0.1 = 0.3744
¿= 0.5 ⋅ 0.1 = 3.625 ⋅ 0.1 = 0.3625
¿= 0.6 ⋅ 0.1 = 3.496 ⋅ 0.1 = 0.3496
¿= 0.7 ⋅ 0.1 = 3.363 ⋅ 0.1 = 0.3363
¿= 0.8 ⋅ 0.1 = 3.232 ⋅ 0.1 = 0.3232

65
Prepared by: Nathaniel M. Cabansay, BSCpE

¿= 0.9 ⋅ 0.1 = 3.109 ⋅ 0.1 = 0.3109

4 ¿ = 0.4 + 0.3981 + ⋯ + 0.3109 = 3.6325 Add all areas

c. Midpoint Riemann sum

No. STATEMENT REASON

1 = @
−2 .
+ 4, = 10 Given
\−0
Δ =
2 Widths of rectangles

1−0
Δ =
10
Δ = 0.1
¿= Δ
¿= 0.05 ⋅ 0.1 = 3.995125 ⋅ 0.1 = 0.3995125
3 Heights of rectangles

¿= 0.15 ⋅ 0.1 = 3.958375 ⋅ 0.1 = 0.3958375


¿= 0.25 ⋅ 0.1 = 3.890625 ⋅ 0.1 = 0.3890625
¿= 0.35 ⋅ 0.1 = 3.797875 ⋅ 0.1 = 0.3797875
¿= 0.45 ⋅ 0.1 = 3.686125 ⋅ 0.1 = 0.3686125
¿= 0.55 ⋅ 0.1 = 3.561375 ⋅ 0.1 = 0.3561375
¿= 0.65 ⋅ 0.1 = 3.429625 ⋅ 0.1 = 0.3429625
¿= 0.75 ⋅ 0.1 = 3.296875 ⋅ 0.1 = 0.3296875
¿= 0.85 ⋅ 0.1 = 3.169125 ⋅ 0.1 = 0.3169125
¿= 0.95 ⋅ 0.1 = 3.052375 ⋅ 0.1 = 0.3052375
4 ¿ = 0.3995125 + 0.3958375 + ⋯ + 0.3052375 = 3.58375 Add all areas

66
Prepared by: Nathaniel M. Cabansay, BSCpE

MODULE 10 DEFINITE INTEGRALS AND THE FUNDAMENTAL THEOREM OF CALCULUS

WHAT ARE DEFINITE INTEGRALS

Definite integrals are basically integrals with limits. As we saw back in Chapter 2, they are written as follows:

[
X ) = W [
3 =W \ −W 0
3

This is also the sum of the areas of all rectangles subdividing the interval 0, \ to find the area under a curve:

[
\−0
X ) = lim ¹ ∗
º z {
3 →Å
º»

PROPERTIES OF SUMMATIONS AND SOME CONVENIENT FORMULAS

Before we derive the properties of definite integrals, we need the following properties and formulas:

Properties

1. We can factor constants out of a summation

¹ 0º = ¹ 0º
º»ºÆ º»ºÆ

2. We can break up a summation across a sum or difference

¹ 0º ± \º = ¹ 0º ± ¹ \º
º»ºÆ º»ºÆ º»ºÆ

Formulas

1. Repeated Addition is Multiplication

¹ =
º»

2. Adding the first n integers results in Triangular Numbers

+1
¹½ =
2
º»

3. Adding the first n odd integers results in the square of n.

¹ 2½ − 1 = .

º»

67
Prepared by: Nathaniel M. Cabansay, BSCpE

4. Adding the first n squares results in the nth square pyramidal number.

+1 2 +1
¹ ½. =
6
º»

5. Adding the first n cubes results in the square of the nth triangular number.

.
+1
.
¹ ½ = ǹ ½È = j
@
k
2
º» º»

PROPERTIES OF DEFINITE INTEGRALS


Here are some properties of definite integrals. Some properties are unique to definite integrals, while some others
are similar to those in indefinite integrals.

∫3 ) = − ∫[ ) (Reversal of Limits)
[ 3

∫3 ) = 0 (Single Limit)
1.
3

) = ∫3 ) (Constant Multiple Rule for Definite Integrals)


2.
∫3
[ [

± ) = ) ± ∫3 ) (Sum/Difference Rule for Definite Integrals)


3.
∫3 ∫3
[ [ [

) = ) + ) (Sum of Integrals on Adjacent Intervals for Integrable Functions)


4.
∫3 ∫3 ∫É
[ É [

) = ] )] (Change of Variables)
5.
∫3 ∫3
[ [

) = \ − 0 (Definite Integral of a Constant)


6.
∫3
[

À 0 for 0 ≤ ≤ \, then ∫3 ) À 0 (Sign of Function Matches Sign of Definite Integral)


7.
[

À for 0 ≤ ≤ \, then ) À ∫3 ) (Preservation of Inequality)


8. If
∫3
[ [

10. If Ë ≤ ≤ Ì for 0 ≤ ≤ \, then Ë \ − 0 ≤ ) ≤ Ì \ − 0 (Upper and Lower Bounds)


9. If
∫3
[

11. ‘∫3 ) ‘ ≤ ∫3 | | ) (Absolute Value of a Definite Integral)


[ [

is an even function, then ∫ ) = 2 ∫q ) . (Integral of an Even Function)


3 3
3
∫3 ) = 0 (Integral of an Odd Function)
3
12. If
13. If is an odd function, then

DERIVING TH E PROPERTIES OF INTEGRALS

REVERSAL OF LIMITS

No. STATEMENT REASON


[
\−0
X ) = lim ¹ ∗
z {
1 Definition of the definite integral
º
→Å
3 º»

[
− 0−\ \−0 = − 0 −\
X ) = lim ¹ ∗
j k
2
º
→Å
3 º»

[
0−\
X ) = − lim ¹ ∗
z { ¹ 0º = ¹ 0º
3
º
→Å
3 º» º»ºÆ º»ºÆ

68
Prepared by: Nathaniel M. Cabansay, BSCpE

[ 3
X ) = −X )
4 Definition of the definite integral
3 [

SINGLE LIMIT

No. STATEMENT REASON


[
\−0
X ) = lim ¹ ∗
z {
1 Definition of the definite integral
º
3 →Å
º»

3
0−0 Set \ = 0
X ) = lim ¹ ∗
M Q
2
º
3 →Å
º»

Subtract 0 from 0 and anything multiplied by 0 is 0


3
X ) = lim 0
3
3 →Å

Limit Theorem 1: lim =



X L F CF = a →3
4

CONSTANT MULTIPLE RULE FOR DEFINITE INTEGRALS

No. STATEMENT REASON


[
\−0
X ) = lim ¹ ∗
z {
1 Definition of the definite integral
º
3 →Å
º»

[
\−0
X ) = lim ¹ ∗
z { ¹ 0º = ¹ 0º
2
º
3 →Å
º» º»ºÆ º»ºÆ

Limit Theorem 3: lim = lim


→3 →3
Í Í
X eL F CF = e X L F CF
3 Definition of the definite integral
‚ ‚

SUM/DIFFERENCE RULE FOR DEFINITE INTEGRALS

No. STATEMENT REASON


[
\−0
X ± ) = lim ¹ ∗
± ∗
z {
1 Definition of the definite integral
º º
3 →Å
º»

[
\−0 \−0
X ± ) = Ç lim ¹ z { ± lim ¹ z {È ¹ 0º ± \º = ¹ 0º ± ¹ \º
∗ ∗
2
→Å º →Å º
3 º» º» º»ºÆ º»ºÆ º»ºÆ

Limit Theorem 4: lim ±


→3
= lim ± lim
→3 →3

69
Prepared by: Nathaniel M. Cabansay, BSCpE

Í Í Í
X L F ±f F CF = X L F CF ± X f F CF
3 Definition of the definite integral
‚ ‚ ‚

SUM OF INTEGRALS ON ADJACENT INTERVALS FOR INTEGRABLE FUNCTIONS

No. STATEMENT REASON

1 Graphical
[
=X )
representation and

3
definition of a
definite integral

2 (Graphical
É [
=X ) +X )
Representation)
3 É
Split the interval at

0, \
a point within

3 Notice that the two


smaller areas
defined in
Statement 2 are just
equal to the larger
area defined in
Statement 1.

Í e Í
X L F CF = X L F CF + X L F CF
‚ ‚ e

CHANGE OF VARIABLES

No. STATEMENT REASON


[
X )
1 Given
3
[
= ] and ) = )]
X ] )]
2 Let
3
Í Í
= ] and ) = )]
X L F CF = X L Î CÎ
3 Since
‚ ‚

70
Prepared by: Nathaniel M. Cabansay, BSCpE

DEFINITE INTEGRAL OF A CONSTANT

No. STATEMENT REASON


[
\−0
X ) = lim ¹ ∗
z {
1 Definition of a definite integral
º
3 →Å
º»

[
\−0 = . Since = , ∗
=
X ) = lim ¹ z {
2 Let º as well
3 →Å
º»
[
\−0
X ) = z { ¹ =
3
3
º»

Limit Theorem 1
Í [ 3
X e CF = e Í − ‚
4 Multiply by

SIGN OF FUNCTION MATCHES SIGN OF DEFINITE INTEGRAL

No. STATEMENT REASON


[
\−0
X ) = lim ¹ ∗
z {
1 Definition of the definite integral
º
→Å
3 º»

2 If À 0 for 0 ≤ ≤ \, then: À 0 and


[ 3
À0
\−0
Since

¹ ∗
º z {À0
º»

3 If À 0 for 0 ≤ ≤ \, then: Get the limits of both sides

\−0
lim ¹ ∗
º z { À lim 0 = 0
→Å →Å
º»

4 If L F À a for ‚ ≤ F ≤ Í, then: Definition of the definite integral

Í−‚
c
Í
X L F CF = €•Ï ¹ L F∗Ð z {Àa
‚ c→Å c
лP

PRESERVATION OF INEQUALITY

No. STATEMENT REASON

À for 0 ≤ ≤ \, then: À for 0 ≤ ≤ \, − À 0 for


0 ≤ ≤ \. If À 0 for 0 ≤ ≤ \, then:
1 If Since
[
X − ) À0 [
3 X ) À0
3

2 If À for 0 ≤ ≤ \, then: Difference Rule for Definite Integrals

71
Prepared by: Nathaniel M. Cabansay, BSCpE

[ [
X ) −X ) À0
3 3

3 If L F À f F for ‚ ≤ F ≤ Í, then: Add ∫3


[
) to both sides.
Í Í
X L F CF À X f F CF
‚ ‚

UPPER AND LOWER BOUNDS

No. STATEMENT REASON

If Ë ≤ ≤ Ì for 0 ≤ ≤ \, then: Since Ë ≤ ≤ Ì for 0 ≤ ≤ \. If À


0 ≤ ≤ \, then:
1 for
[ [ [
X Ë) ≤X ) ≤X Ì) [ [
3 3 3
X ) ÀX )
3 3

If Ñ ≤ L F ≤ Ò for ‚ ≤ F ≤ Í, then: [
X ) = \−0
2
Í 3
Ñ Í − ‚ ≤ X L F CF ≤ Ò Í − ‚

ABSOLUTE VALUE OF A DEFINITE INTEGRAL

No. STATEMENT REASON

1 −| |≤ ≤| | The actual value of a function is always between its


additive inverse and itself.
[ [ [
À for 0 ≤ ≤ \, then:
− X| |) ≤ X ) ≤ X| |) [ [
2 If

X ) ÀX )
3 3 3
3 3
Í Í
If −0 ≤ ≤ 0, then | | ≤ 0
žX L F CFž ≤ X|L F | CF
3

‚ ‚

DEFINITE INTEGRAL OF AN EVEN FUNCTION

No. STATEMENT REASON


3 q 3
be an even function defined on −0, 0 , then
X ) =X ) +X )
1 Let
3 3 q
use the property
[ É [
X ) =X ) +X )
3 3 É

− =
3 3 3
X ) =X ) +X )
2 For an even function,
3 q q

72
Prepared by: Nathaniel M. Cabansay, BSCpE

Add ∫q ) to ∫q )
‚ ‚ 3 3
X L F CF = G X L F CF
3
‚ a

DEFINITE INTEGRAL OF AN ODD FUNCTION

No. STATEMENT REASON


3 q 3
be an odd function defined on −0, 0 , then
X ) =X ) +X )
1 Let
3 3 q
use the property
[ É [
X ) =X ) +X )
3 3 É

− =−
3 3 3
X ) = −X ) +X )
2 For an odd function,
3 q q

Add − ∫q ) to ∫q )
‚ 3 3
X L F CF = a
3

EXAMPLES OF USING PROPERTIES OF DEFINITE INTEGRALS

Evaluate the definite integral:


£
X sin )
£

No. STATEMENT REASON


£
X sin )
1 Given
£

£ Note that sin


X sin ) =0
2 is an odd function, so we use the
£
property:
3

X ) =0
3

Evaluate the definite integral:


£
r
X sec. )
£
r

No. STATEMENT REASON


£
r
X sec . )
1 Given
£
r

73
Prepared by: Nathaniel M. Cabansay, BSCpE

Pythagorean Identity: sin. + cos . =1


£
r
X sec . )
2
£
r

Note that sec


£ £
r r
X sec .
) = 2 X sec . )
3 is an even function, and the product of
£
r q
even functions results in an even function so we use the
property:
3 3
X ) = 2X )
3 3

FUNDAMENTAL THEOREM OF CALCULUS

The fundamental theorem of calculus states that differentiation and integration are inverse operations.

The first part of this theorem is written as:

If is continuous on 0, \ , then:

=X ] )]
3

and this function is continuous on 0, \ and is differentiable on 0, \ and that =

The second part of this theorem is written as:

[
X ) =W \ −W 0
3

The first part of the theorem simply states the connection between the integral and the derivative, while the
second part of the theorem states that the definite integral can be computed with the use of antiderivatives or
indefinite integrals.

PROVING THE FUNDAMENTAL THEOREM OF CALCULUS

FIRST PART

No. STATEMENT REASON

For a given ] continuous on 0, \ ,


=X ] )]
1
3
on 0, \ and differentiable on 0, \
define the function also continuous


+ ℎ in
+ℎ − =X ] )] − X ] )]
0, \ , we have
2 For any two numbers and
3 3

=X ] )]
3

and

74
Prepared by: Nathaniel M. Cabansay, BSCpE


+ℎ =X ] )]
3
8Ó [ É [
+ℎ − =X ] )] + X ] )] − X ] )] X ) =X ) +X )
3
3 3 3 3 É

+ℎ − =X ] )]
4 Simplify the right-hand side

+ℎ − 1 8Ó
Divide both sides by ℎ, assuming ℎ ≠ 0
= X ] )]
ℎ ℎ
5

8Ó 8Ó 8Ó
Assume ℎ > 0 and assume + ℎ is still
X Ë )] ≤ X ] )] ≤ X Ì )]
in 0, \ and since we know that
6

continuous on , + ℎ , the Extreme


is

Ë +ℎ− ≤X ] )] ≤ Ì +ℎ−
numbers and ) in , + ℎ such that
Value Theorem tells us that there are

= Ë is the absolute minimum of



Ëℎ ≤ X ] )] ≤ Ìℎ
in , + ℎ and ) = Ì is the
8Ó in , + ℎ .
ℎ≤X ] )] ≤ ) ℎ
absolute maximum of
From that, use this property:
[
Ë \−0 ≤X ) ≤Ì \−0
3

1 8Ó
Divide all parts of the inequality by ℎ
≤ X ] )] ≤ )

7

+ℎ −
≤ ≤ )
ℎ assumed ℎ < 0, we will still arrive at the
8 From statement 5. Note that if we

same inequality
+ℎ −
lim ≤ lim ≤ lim )
ℎ as ℎ approaches 0
9 Get the limits of all parts of the inequality
Ó→q Ó→q Ó→q

+ℎ − As ℎ approaches 0, and ) will approach


≤ lim ≤
ℎ as both and ) are in , + ℎ
10
Ó→q

≤ ≤
=
11 Definition of the derivative and Limit
Theorem 10: Squeeze Theorem

SECOND PART

No. STATEMENT REASON

=X ] )]
1 First part of the Fundamental Theorem of Calculus
3

=
2 =W Let W be any anti-derivative of on 0, \

3 W = +Y If W = , then W = +

75
Prepared by: Nathaniel M. Cabansay, BSCpE

W \ −W 0 = \ +Y − 0 +Y and W are continuous on 0, \ , if we


= \ − 0 take the limits as approaches 0 from the right and \
4 Because

from the left, we can see that this is still true if = 0


and = \. So, for 0 ≤ ≤ \, we know that W
+ Y.
and

[ 3
W \ −W 0 =X ] )] − X ] )]
5 From the first part of the Fundamental Theorem of
3 3
Calculus
[ 3
W \ −W 0 =X ] )] X ) =0
6
3 3

Í [ [
X L F CF = g Í − g ‚ X ) =X ] )]
7
‚ 3 3

USING TH E FUNDAMENTAL THEOREM OF CALCULUS

The Fundamental Theorem of Calculus, especially the second part, is especially useful when computing definite
integrals, as this allows us to use the indefinite integrals and simply substitute the limits in to find the area under a
curve.

Example:

Evaluate the definite integral:


£
r
X sec. )
£
r

We can go about this in two ways:

We can use the Fundamental Theorem of Calculus directly…

No. STATEMENT REASON


£
r
X sec . )
1 Given
£
r
£
r ¡ ¡
X sec . ) = tan M Q − tan M− Q
4 4
2 Fundamental Theorem of Calculus and

X sec . % )% = tan % + Y
£
r

£
r ¡ ¡ tan − = − tan
X sec . ) = tan M Q + tan M Q
4 4
3
£
r
£ ¡
r tan M Q = 1
X sec .
) = 1+1 4
4
£
r

76
Prepared by: Nathaniel M. Cabansay, BSCpE

£
r
X sec . ) =2
5 Add 1 to 1
£
r

…or we can use this property:


3 3
X ) = 2X )
3 q

No. STATEMENT REASON


£
r
X sec . )
1 Given
£
r

Since sec is an even function, and the product of two


£ £
r r
X sec .
) = 2 X sec . )
2
£
r q
even functions is still an even function
3 3
X ) = 2X )
3 q
£
r ¡
X sec . ) = 2 Mtan M Q − tan 0 Q
4
3 Fundamental Theorem of Calculus
£
r
£
tan M Q = 1 and tan 0 = 0
£
r
X sec .
) =2 1 r
4
£
r

£
r
X sec . ) =2
5 Multiply 1 by 2
£
r

Evaluate the definite integral:

.
X 3 .
)

No. STATEMENT REASON


.
X 3 .
)
1 Given

.
X 3 .
) = 2 @
− 1 @
2 Fundamental Theorem of Calculus and
8
X ) = + Y, ≠ −1
+1
.
2@ = 8 and 1@ = 1
X 3 .
) =8−1
3

.
X 3 .
) =7
4 Subtract 1 from 8

77
Prepared by: Nathaniel M. Cabansay, BSCpE

CHAPTER 4 INTEGRATION TECHNIQUES

MODULE 11 MORE ON INTEGRATION BY SUBSTITUTION

SUBSTITUTION RULE FOR INDEFINITE INTEGRALS

have to do is to introduce a variable % equal to that function and )% corresponding to the differential of that
Sometimes, we are given functions that are not in one of the forms of the rules seen in Modules 3 to 8. All we

function.

In essence, this is just the reverse of the Chain Rule for Derivatives. We derived the integral formula for this one
back in Module 3. Here is the rule:

X ) =W +Y

) so we can let % =
In integrating by substitution, we first check if we have a function that has its derivative with it since we are

% )% to make the integration easier.


supposed to have the given function in the form and rewrite the
function as

MORE EXAMPLES OF INTEGRATION BY SUBSTITUTION FOR INDEFINITE INTEGRALS


Evaluate:

1
X z1 − { sec . − ln )

1
No. STATEMENT REASON

X z1 − { sec . − ln )
1 Given

Integrate by substitution: Let % = − ln so )% = 1 −


X sec . % )%
)
2

tan % + Y
X sec . % )% = tan % + Y
3

4 tan − ln +Y Substitute back % = − ln

Evaluate:

e. − 1
X )
e. + 1

We have two options to go about this one but this is more of a branched solution since the first step is common

No. STATEMENT REASON


e. − 1
X )
e. + 1
1 Given

78
Prepared by: Nathaniel M. Cabansay, BSCpE

e −e Multiply the numerator and denominator by e


X )
e +e
2

1
X )%
u % = e + e and )% = e − e )
3 (OPTION A) Integrate by substitution directly by letting

X tanh )
= tanh )
Õ Õ
(OPTION B) Õ8 Õ

e +e (OPTION A) ∫ / )% = ln|%| + Y then substitute back


ln • •+Y
2 % =e +e
4

ln|e + e | − ln|2| + Y
(OPTION B) ∫ tanh ) = ln|cosh | + Y
ln|e + e | + Y
log 3 M Q = log 3 − log 3 ?, then − ln|2| is a constant,

so this gets assimilated into the +Y

Evaluate:

cos √
X )

No. STATEMENT REASON

cos √
X )

1 Given

2 X cos % )% Integrate by substitution: Let % = √ so )% =


.√
2
2 )% =
or

2 sin % + Y
3 X cos % )% = sin % + Y

4 2 sin √ + Y Substitute back % = √

SUBSTITUTION RULE FOR DEFINITE INTEGRALS

changed as the function is substituted. If we let % =


For definite integrals, integration by substitution follows a similar process. The only difference is that the limits are
we get this rule:

[ [
X ) =X % )%
3 3

We can derive the rule from the Fundamental Theorem of Calculus and the substitution rule for indefinite
integrals.

No. STATEMENT REASON


[
X ) =W \ −W 0
1 Fundamental Theorem of Calculus
3
[
X ) =W \ −W 0
2 Combine with integration by substitution
3

79
Prepared by: Nathaniel M. Cabansay, BSCpE

[ [ Let % =
X ) =X % )%
3 then apply the Fundamental Theorem of
3 3
Calculus.

EXAMPLES OF SUBSTITUTION RULE FOR DEFINITE INTEGRALS

Evaluate:
£
. 6 sin ]
X )]
q 74 − 3 cos ]
No. STATEMENT REASON
£
. 6 sin ]
X )]
1 Given
q 74 − 3 cos ]

Integrate by substitution: % = 4 − 3 cos ] , so )% =


£
. 6 sin ] r
1
X )] = 2 X )%
3 sin ] or 2 )% = 6 sin ] )]. We then have % M Q =
£
√%
2
q 74 − 3 cos ] @ .
4 − 3 cos M Q = 4 − 0 = 4 and % 0 = 4 − 3 cos 0 =
£
.
4−1 =3
£
. 6 sin ]
X )] = 2l2√%m@
r
3 Use the Fundamental Theorem of Calculus on the right-

q 74 − 3 cos ]
hand side
£
. 6 sin ]
X )] = 2 4 − 2√3
q 74 − 3 cos ]
£
. 6 sin ]
X )] = 8 − 4√3
4 Simplify the right-hand side

q 74 − 3 cos ]

Evaluate:
q
X ]73 + ] . )]
.

No. STATEMENT REASON


q
X ]73 + ] . )]
1 Given
.
q
1 @ Integrate by substitution: % = 3 + ] . , so )% = 2] )] or
X ]73 + ] . )] = X √% )%
2 Ö )% = ] )]. We then have % 0 = 3 + 0. = 3 and
2
. .
% −2 = 3 + −2 . = 3 + 4 = 7
q
1 Ö
X ]73 + ] . )] = − X √% )%
2 @
3 Use the Reversal of Limits property of definite integrals
.
q
1 2 @ Ö
X ]73 + ] . )] = − | %. }
2 3
4 Use the Fundamental Theorem of Calculus on the right-
. @
hand side

80
Prepared by: Nathaniel M. Cabansay, BSCpE

q
1 14
X ]73 + ] . )] = − | √7 − 2√3}
. 2 3
q
7√7
X ]73 + ] . )] = √3 −
3
5 Simplify the right-hand side

81
Prepared by: Nathaniel M. Cabansay, BSCpE

MODULE 12 MORE ON INTEGRATION BY PARTS

INTEGRATION BY PARTS

While many integrals can already be solved using integration by substitution, many still cannot be solved using
integration by substitution. We have a technique based on the product rule for differentiation that allows us to
exchange one integral for another.

In essence, this is just the reverse of the Product Rule for Derivatives. We derived the integral formula for this
one back in Module 3. Here is the rule:

X % )> = %> − X > )%

The main problem of integration by parts lies in selecting the proper % and )> to be used. To select our % and )>,
we use the mnemonic LIATE to determine which should be our %:

Logarithmic Functions (including Inverse Hyperbolic Functions) like ln , log @ ], arsinh


Trigonometric Functions like arcsin , arctan M Q [These will usually not appear together in an
and Inverse
v

Algebraic Functions like . , √] r + 7


integration by parts problem]

Trigonometric Functions like sin and Exponential Functions (including Hyperbolic Functions) like e ,

0x , sinh ] [IF

priority for )> instead.


A similar mnemonic is DETAIL, which is simply the reverse of the LIATE mnemonic and is used to determine the

EXAMPLES OF INTEGRATION BY PARTS

Evaluate:

X sec . )

No. STATEMENT REASON

X sec . )
1 Given

Integrate by parts. From LIATE: Let % = and )> =


X sec . ) = tan − X tan )
sec . ) so we get )% = ) and > = tan
2

X sec . ) = tan − ln|sec | + Y X tan ) = ln|sec | + Y


3

Evaluate:

X sec @ )

82
Prepared by: Nathaniel M. Cabansay, BSCpE

No. STATEMENT REASON

X sec @ )
1 Given

This is quite tricky, but if we separate sec @


X sec @ ) = X sec sec . )
into sec and sec. we can have a form
2

where we can use integration by parts on.

Let % = sec and )> = sec. ) so we get


X sec @ ) = sec tan − X sec tan. )
)% = sec tan ) and > = tan
3

Rewrite tan. = sec . −1


X sec @ ) = sec tan − X sec sec . −1 )
4

Multiply sec . − 1 by sec


X sec @ ) = sec tan − X sec @ ) + X sec )
5 and use Integral
Rule 4

Add ∫ sec @ ) to both sides


2 X sec @ ) = sec tan + X sec )
6

2 X sec@ ) = sec tan + ln|sec + tan | + Y X sec ) = ln|sec + tan | + Y


7

1 1
X sec @ ) = sec tan + ln|sec + tan | + Y
2 2
8 Divide both sides by 2

TABULAR METHOD OF INTEGRATION BY PARTS

A more organized and quicker method of integration by parts is using a table, which can also be called the DI
Method (Chow, 2016). We would need a form of this table:

+ % )>
D I

− )% >
+ ).%
X > )>

− )@%
X zX > )>{

This has three stopping conditions:

A. If a 0 appears in the derivatives column


B. If the product of a row is already integrable
C. If the product of a row is a multiple of the original integrand

In this table, rows represent integrals, and diagonals will represent products. It might be helpful to place % =,
)> =, )% =, > = in the table so you remember to multiply diagonally and use an integral on a row.

83
Prepared by: Nathaniel M. Cabansay, BSCpE

EXAMPLES OF USING TABULAR METHOD OF INTEGRATION BY PARTS

Evaluate:

X5 .
cos 5 − 2 )

No. STATEMENT REASON

X5 .
cos 5 − 2 )
1 Given

5 5 5
X5 .
cos 5 − 2 ) = .
sin 5 + 2 + cos 5 + 2 + sin 5 + 2 +Y
2 2 4
2 Integrate by parts
(Table shown on
the left)
D I

+ %=5 .
)> = cos 5 + 2

− )% = 10 ) 1
>= sin 5 + 2
2
+ ) . % = 10 ) 1
X > )> = − cos 5 + 2
4
− )@ % = 0 1
X zX > )>{ = sin 5 + 2
8

Evaluate:
.
Xj − + 1k e )
2

No. STATEMENT REASON


.
Xj − + 1k e )
2
1 Given

. .
Xj − + 1k e ) = −j − + 1k e − −1 e −e +Y
2 2
2 Integrate by parts (Table
shown on the left)
D I

+ .
)> = e
%=j − + 1k
2

− )% = −1 > = −e

+ ) %=1
.
X > )> = e

− )@ % = 0
X zX > )>{ = −e

. . Factor −e
Xj − + 1k e ) = −e Çj − + 1k + − 1 + 1È + Y
2 2
3 from the right-
hand side

. .
Xj − + 1k e ) = −e j + 1k + Y
2 2
4 Combine like terms

84
Prepared by: Nathaniel M. Cabansay, BSCpE

Evaluate:

X4 @
ln 2 )

No. STATEMENT REASON

X4 @
ln 2 )
1 Given

X4 @
ln 2 ) = r
ln 2 −X @
)
2 Integrate by parts (Table shown on the left)

D I

+ % = ln 2 )> = 4 @

− 1 >= r
)% =

r
X4 @
ln 2 ) = r
ln 2 − +Y
4
3 Integral Rule 2

Evaluate:

X e. cos 2 )

No. STATEMENT REASON

1 X e. cos 2 ) Given

e. e.
X e. cos 2 ) = cos + sin − X e. cos 2 ) +Y
2 2 on the left, either choice of % or
2 Integrate by parts (Table shown

)> will yield a similar answer)


+ % = cos 2 )> = e.
D I

% = e. )> = cos 2
− )% = −2 sin 2 e.
>=
)% = 2e. 2
sin 2
>=
2
+ ) . % = −4 cos 2 e.
X > )> =
) . % = 4e. 4
cos 2
X > )> = −
4

e. e. Add ∫ e. cos 2 ) to both


2 X e. cos 2 ) = cos + sin +Y
2 2
3
sides

e.
X e. cos 2 ) = cos + sin +Y
4
4 Divide both sides by 2
Õ

r
(OPTIONAL) Factor from
the right-hand side

85
Prepared by: Nathaniel M. Cabansay, BSCpE

MODULE 13 TRIGONOMETRIC TRANSFORMATIONS

IMPORTANT TRIGONOMETRIC IDENTITIES

Before we tackle integrating trigonometric integrands, we’ll need a few trigonometric identities here:

a. sin. + cos . = 1
A. Pythagorean Identities

b. 1 + tan. = sec.
c. cot . + 1 = cot .

a. sin ˜ ± ™ = sin ˜ cos ™ ± cos ˜ sin ™


B. Sum/Difference of Angles

b. cos ˜ ± ™ = cos ˜ cos ™ ∓ sin ˜ sin ™

a. sin − = − sin
C. Negative Angle Identities

b. cos − = cos

a. sin 2 = 2 sin cos


D. Double Angle Identities

b. cos 2 = cos .
− sin. = 2 cos . − 1 = 1 − 2 sin.

sin. = 1 − cos 2
E. Power-Reducing Formulas for Sine and Cosine:

.
cos . = 1 + cos 2
a.

.
sin cos = sin 2
b.

.
c.

sin 0 sin \ = . cos 0 − \ − cos 0 + \


F. Product-to-Sum Formulas

cos 0 cos \ = cos 0 − \ + cos 0 + \


a.

.
sin 0 cos \ = sin 0 − \ + sin 0 + \
b.

.
c.

INTEGRALS INVOLVING PRODUCTS OF SINE AND COSINE WITH DIFFERENT ARGUMENTS

For dealing with integrands involving products of sine and cosine with different arguments, we use the product-to-
sum formulas:

sin 0 sin \ = cos 0 − \ − cos 0 + \


.
a.
cos 0 cos \ = cos 0 − \ + cos 0 + \
.
sin 0 cos \ = sin 0 − \ + sin 0 + \
b.

.
c.

We will also need the two negative argument identities:

sin − = − sin
cos − = cos
a.
b.

86
Prepared by: Nathaniel M. Cabansay, BSCpE

In this case, we have:

1
X sin % sin > ) = X cos % − > − cos % + > )
2

1
X cos % cos > ) = X cos % − > + cos % + > )
2

1
X sin % cos > ) = X sin % − > + sin % + > )
2

EXAMPLES OF INTEGRALS INVOLVING PRODUCTS OF SINE AND COSINE WITH DIFFERENT


ARGUMENTS

Evaluate:

X sin 7 sin 5 )

No. STATEMENT REASON

X sin 7 sin 5 )
1 Given

1 1
X sin 7 sin 5 ) =X cos 2 − cos 12 ) sin 0 sin \ = cos 0 − \ − cos 0 + \
2 2
2

1
X sin 7 sin 5 ) = X cos 2 − cos 12 )
2
3 Integral Rule 3

1 1
X sin 7 sin 5 ) = X cos 2 ) − X cos 12 )
2 2
4 Integral Rule 4

1 1 Let % = 2 in the left integral and %. =


X sin 7 sin 5 ) = X cos % )% − X cos %. )%.
4 24 12 in the right integral. This gives )% =
5

2 ) or )% = ) in the left integral and


.
)%. = 12 ) or )%. = ) in the right
.

1 1
integral.

X sin 7 sin 5 ) = sin % − sin %. + Y X cos % )% = sin % + Y


4 24
6

1 1 Substitute back % = 2 and %. = 12


X sin 7 sin 5 ) = sin 2 − sin 12 +Y
4 24
7

Evaluate:

X cos 3 cos 6 )

No. STATEMENT REASON

X cos 3 cos 6 )
1 Given

87
Prepared by: Nathaniel M. Cabansay, BSCpE

1
X cos 6 cos 3 ) =X cos 3 + cos 9 )
2 multiplication and cos 0 cos \ =
2 Use the commutative property of

cos 0 − \ + cos 0 + \
.

1
X cos 6 cos 3 ) = X cos 3 + cos 9 )
2
3 Integral Rule 3

1 1
X cos 6 cos 3 ) = X cos 3 ) + X cos 9 )
2 2
4 Integral Rule 4

1 1 Let % = 3 in the left integral and


X cos 6 cos 3 ) = X cos %1 )%1 + X cos %2 )%2
6 18 %. = 9 in the right integral. This gives
5

)% = 3 ) or @ )% = ) in the left
integral and )%. = 9 ) or )%. = )
y

1 1
in the right integral.

X cos 6 cos 3 ) = sin % + sin %. + Y X cos % )% = sin % + Y


6 18
6

1 1 Substitute back % = 2 and %. = 12


X cos 6 cos 3 ) = sin 3 + sin 9 +Y
6 18
7

Evaluate:

X sin 4 + 3 cos 2 − 1 )

No. STATEMENT REASON

X sin 4 + 3 cos 2 − 1 )
1 Given

1 1
X sin 4 + 3 cos 2 − 1 ) = X sin 2 + 4 + sin 6 + 2 ) sin 0 cos \ = sin 0 − \
2 2
2
+ sin 0 + \
1
X sin 4 + 3 cos 2 − 1 ) = X sin 2 + 4 + sin 6 + 2 )
2
3 Integral Rule 3

1 1
X sin 4 + 3 cos 2 − 1 ) = X sin 2 + 4 ) + X sin 6 + 2 )
2 2
4 Integral Rule 4

1 1 Let % = 2 + 4 and %. = 6 + 2.
X sin 4 + 3 cos 2 − 1 ) = X sin % )% + X sin %. )%
4 12 This gives us )% = 2 ) or
5

)% = ) and )%. = 6 ) or
.
)% = )
Ù

1 1
X sin 4 + 3 cos 2 − 1 ) = − cos % − cos %. + Y X sin % )% = − cos % + Y
4 12
6

1 1 Substitute back % = 2 + 4 and


X sin 4 + 3 cos 2 − 1 ) = − cos 2 + 4 − cos 6 + 2 + Y
4 12 %. = 6 + 2
7

88
Prepared by: Nathaniel M. Cabansay, BSCpE

INTEGRALS INVOLVING POWERS OF SINE AND COSINE

For dealing with integrals involving powers of sine and cosine we need two variants of its Pythagorean identity,
two power-reducing formulas, and a rearrangement of the double angle identity for sine:

sin. + cos . = 1
a. sin. = 1 − cos .
1.

b. cos . = 1 − sin.
sin. = 1 − cos 2
.
2.
cos . = 1 + cos 2
.
sin cos = sin 2
3.

.
4.

Then we have these derivatives:

1. sin % = cos % )%
2. cos % = − sin % )%

Then we have these integrals:

∫ cos % )% = sin % + Y
∫ sin % )% = − cos % + Y
1.
2.

of the differential sinÚ cos ) , where Ë and


Once we have the needed identities, derivatives, and integrals, we are ready to deal with integrals involving a form
are non-negative integers. We will be dealing with four
possible cases:

NOTE: For all cases, 0 is considered an even exponent.

A. If Ñ (the exponent on the sine) is even and c (the exponent on the cosine) is odd:

We strip out one cosine, and get this form:

X sinÚ cos ) = X sinÚ cos cos )

We then rewrite the remaining cosines, which now have an even exponent, using cos . = 1 − sin.
multiply the sines into it, then use the substitution % = sin and )% = cos )
so we can

B. If Ñ (the exponent on the sine) is odd and c (the exponent on the cosine) is even:

We strip out one sine, and get this form:

X sinÚ cos ) = X sinÚ cos sin )


We then rewrite the remaining sines, which now have an even exponent, using sin. = 1 − cos .
multiply the cosines into it, then use the substitution % = cos and )% = − sin )
so we can

C. If both Ñ and c are odd:

You can strip out either one sine or one cosine, but it will be more convenient to strip out a sine or cosine from
whichever between them has a smaller exponent.

89
Prepared by: Nathaniel M. Cabansay, BSCpE

D. If both Ñ and c are even:

We can use the power-reducing formulas or the modified double angle formula for sine:

sin. = 1 − cos 2 , cos . = 1 + cos 2 , and sin cos = sin 2


. . .

EXAMPLES OF INTEGRALS INVOLVING POWERS OF SINE AND COSINE

Evaluate:

X sinr cos Ö )

No. STATEMENT REASON

X sinr cos Ö )
1 Given

X sinr cos Ö ) = X sinr cos Ù cos )


2 Cosine is the one with the odd
exponent, so strip out one
cosine.

X sinr cos Ö ) = X sinr 1 − sin. @


cos )
3 Rewrite the remaining cosines

cos . = 1 − sin.
into sines using this formula:

4 X sinr cosÖ ) = X sinr 1 − 3 sin. + 3 sinr − sinÙ cos ) Binomial expansion

X sinr cosÖ ) = X sinr − 3 sinÙ + 3 sin× − sin q cos )


expanded binomial by sinr
5 Multiply all terms of the

X sinr cos Ö ) = X %r − 3% Ù + 3% × − % q
)%
% = sin and )% = cos )
6 Integrate by substitution: Let

% x 3% Ö % y %
X sinr cos Ö ) = − + − +Y
5 7 3 11
7 Integral Rules 2 and 3

sinx 3 sinÖ siny sin Substitute back % = sin


X sinr cos Ö ) = − + − +Y
5 7 3 11
8

Evaluate:

X sinx cos r )

No. STATEMENT REASON

X sinx cos r )
1 Given

X sinx cos r ) = X sinr cos r sin )


2 Sine is the one with the odd
power, so strip out one sine

90
Prepared by: Nathaniel M. Cabansay, BSCpE

X sinx cos r ) = X 1 − cos . .


cos r sin )
3 Rewrite the remaining sines

sin. = 1 − cos .
into cosines using this formula:

X sinx cos r ) = X 1 − 2 cos . + cos r cos r sin )


4 Binomial expansion

X sinx cos r ) = X cos r − 2 cos Ù + cos × sin )


expanded binomial by cos r
5 Multiply all terms of the

X sinx cos r ) = − X % r − 2%Ù + %× )%


% = cos and )% = − sin )
6 Integrate by substitution: Let

so −)% = sin )

% x 2% Ö % y
X sinx cos r ) = − + − +Y
5 7 9
7 Integral Rules 2 and 3

cos x 2 cos Ö cos y Substitute back % = cos


X sinx cos r ) = − + − +Y
5 7 9
8

Evaluate:

X 40 sinÖ cos @ )

No. STATEMENT REASON

X 40 sinÖ cos @ )
1 Given

X sinÖ cos @ ) = 40 X sinÖ cos . cos )


2 Cosine has the smaller odd
power, so strip out one cosine.
Also, use Integral Rule 3.

X sinÖ cos @ ) = 40 X sinÖ 1 − sin. cos )


3 Rewrite the remaining cosines

cos . = 1 − sin.
into sines using this formula:

Multiply 1 − sin. by sinÖ


X sinÖ cos @ ) = 40 X sinÖ − siny cos )
4

X sinÖ cos @ ) = 40 X % Ö − % y )%
% = sin and )% = cos )
5 Integrate by substitution: Let

X sinÖ cos @ ) = 5% × − 4% q
+Y
6 Integral Rule 2, then multiply
the resulting polynomial by 40

Substitute back % = sin


X sinÖ cos @ ) = 5 sin× − 4 sin q
+Y
7

Evaluate:

X sinr cos r )

We can go about this in two ways:

91
Prepared by: Nathaniel M. Cabansay, BSCpE

We can use the power reducing formulas:

No. STATEMENT REASON

X sinr cos r )
1 Given

1 .
1 .
X sinr cos r ) = X z 1 − cos 2 { z 1 + cos 2 { )
2 2 formulas sin. = 1 −
2 Use the power reducing
.
cos 2 and cos . = 1+
.
cos 2
1
X sinr cos r ) = X 1 − cos 2 .
1 + cos 2 .
)
16
.
3 Square and then use Integral
Rule 3
1
X sinr cos r ) = X 1 − cos . 2 .
)
16
4 Multiply both binomials

1 1 1 .
X sinr cos r ) = X z1 − − cos 4 { )
16 2 2 formula cos . = . 1 +
5 Use the power reducing

cos 2

1 1 1 .
X sinr cos r ) = X z − cos 4 { )
16 2 2
6 Combine like terms

1
X sinr cos r ) = X 1 − cos 4 .
)
64
r
7 Factor out

1
X sinr cos r ) = X 1 − 2 cos 4 + cos . 4 )
64
8 Square the binomial

1 1 1
X sinr cos r ) = X 1 − 2 cos 4 + + cos 8 )
64 2 2 formula cos . = 1 +
9 Use the power reducing
.
cos 2
1 3 1
X sinr cos r ) = X − 2 cos 4 + cos 8 )
64 2 2
10 Combine like terms

3 1 1
X sinr cosr ) = X 1 ) − X cos 4 ) + X cos 8 )
128 32 128
11 Integral Rules 3 and 4

3 1 1 Let % = 4 so )% = 4 )
X sinr cosr ) = X1 ) − X cos % )% + X cos %. )%.
128 128 1024 or )% = ) and %. = 8
12
r
so )%. = 8 ) or × )%. = )
3 1 1 ∫ cos % )% = sin % + Y and
X sinr cosr ) = − sin % + sin %.
128 128 1024 ∫1) = + Y
13

3 1 1 Substitute back % = 4 and


X sinr cosr ) = − sin 4 + sin 8 +Y
128 128 1024 %. = 8
14

92
Prepared by: Nathaniel M. Cabansay, BSCpE

Or we can use the modified double angle identity for sine

No. STATEMENT REASON

X sinr cos r )
1 Given

?
Ú Ú
= ? Ú
X sinr cos r ) = X sin cos r
)
2

1 r 1
X sinr cos r ) = X z sin 2 { ) sin cos = sin 2
2 2
3

1 Raise sin 2
X sinr cos r ) = X sinr 2 )
16
.
4 to the 4th

1
power

X sinr cos r ) = X sinr 2 )


16
5 Integral Rule 3

1 1 .
X sinr cos r ) = X z 1 − cos 4 { )
16 2 sin. = 1 − cos 2
6 Use the reduction formula
.

1
X sinr cos r ) = X 1 − 2 cos 4 + cos . 4 )
64
7 Use integral rule 3 then

1 1 1
square the binomial

X sinr cos r ) = X z1 − 2 cos 4 + + cos 8 { )


64 2 2 cos . = 1 + cos 2
8 Use the reduction formula
.

1 3 1
X sinr cos r ) = X − 2 cos 4 + cos 8 )
64 2 2
9 Combine like terms

3 1 1
X sinr cosr ) = X 1 ) − X cos 4 ) + X cos 8 )
128 32 128
10 Integral Rules 3 and 4

3 1 1 Let % = 4 so )% = 4 )
X sinr cosr ) = X1 ) − X cos % )% + X cos %. )%.
128 128 1024 or )% = ) and %. = 8
11
r
so )%. = 8 ) or )%. = )
×

3 1 1 ∫ cos % )% = sin % + Y and


X sinr cosr ) = − sin % + sin %.
128 128 1024 ∫1) = + Y
12

3 1 1 Substitute back % = 4 and


X sinr cosr ) = − sin 4 + sin 8 +Y
128 128 1024 %. = 8
13

WALLIS’ FORMULA
£
.
Wallis’ formula is useful for definite integrals of trigonometric functions with limits from 0 to


G Ñ−P ‼ c−P ‼
X Œ••Ñ h ŽƒŒ c h Ch = ⋅Ü
a Ñ+c ‼

−2 … 5 3 1 > 0 Þ))
Where ‼ is the double factorial function and is defined as ‼ = Ý −2 … 6 4 2 > 0 ¨>¨
1 = −1, 0

93
Prepared by: Nathaniel M. Cabansay, BSCpE

Ë−1 Ë−3 … 5 3 1 Ë > 1 ¨>¨ −1 −3 … 5 3 1 > 1 ¨>¨


So, we have Ë − 1 ‼ = Ý Ë − 1 Ë − 3 … 6 4 2 Ë > 1 Þ)) −1 ‼=Ý −1 −3 … 6 4 2 > 1 Þ))
1 Ë = 1, 0 1 Ë = 1, 0
, and

Ë+ Ë+ −2 … 5 3 1 Ë + > 0 Þ))
Ë+ ‼=Ý Ë+ Ë+ −2 … 6 4 2 Ë + > 0 ¨>¨
1 = −1, 0

And ˜ = if both Ë and are even, and 1 if at least one of Ë or


£
.
is odd.

EXAMPLES OF WALLIS’ FORMULA

Evaluate:
£
.
X sinr cos x )
q

No. STATEMENT REASON


£
.
X sinr cos x )
1 Given

q
£
. 4 − 1 !! 5 − 1 ‼
X sinr cos x ) = so we use Wallis’ formula with Ë = 4 and = 5. Since
4+5 ‼
2 The conditions for Wallis’ formula to be used are met,

is odd, ˜ = 1
q

£
. 4 2 3 1
X sinr cos x ) =
9 7 5 3 1
3 Definition of the double factorial
q

£
. 8
X sinr cos x ) =
4 Divide out the common factors 3 and 1, then multiply 4

q 63
by 2 and 9 by 7 then 5

Evaluate:
£
.
X sinÖ cos . )
q

No. STATEMENT REASON


£
.
X sinÖ cos . )
1 Given

q
£
. 7 − 1 !! 2 − 1 ‼
X sinÖ cos . ) = so we use Wallis’ formula with Ë = 7 and = 2. Since
7+2 ‼
2 The conditions for Wallis’ formula to be used are met,

Ë is odd, ˜ = 1
q

£
. 6 4 2 1
X sinÖ cos . ) =
9 7 5 3 1
3 Definition of the double factorial
q

£
. 16
X sinr cos x ) =
4 Divide out the common factors 3, and 1, then multiply 4

q 315
by 2 and 9 by 7

94
Prepared by: Nathaniel M. Cabansay, BSCpE

Evaluate:
£
Ù
X sinÖ 3 cos @ 3 )
q

No. STATEMENT REASON


£
Ù
X sinÖ 3 cos @ 3 )
1 Given

q
£ £
Ù 1 .
X sin 3 Ö
cos 3@
) = X sinÖ % cos @ % )% are not yet met, but we have 3 inside the
3 q
2 The conditions for Wallis’ formula to be used

functions, so let % = 3 and )% = 3 ) , so


q

)% = ) , then use Integral Rule 3 and


@
change the limits according to
/ [
X % )%
/ 3

Since % = 3 , the upper limit is now 3 M Q =


£
Ù
and the lower limit is now 3 0 = 0
£
.
£
Ù 1 7−1 ‼ 3−1 ‼
X sinÖ 3 cos @ 3 ) =
3 7+3 ‼
3 The conditions for Wallis’ formula to be used

Ë = 7 and = 3. Since Ë and are both


q
are now met, so we use Wallis’ formula with

odd, ˜ = 1
£
Ù 1 6 4 2 2
X sinÖ 3 cos @ 3 ) =
3 10 8 6 4 2
4 Definition of the double factorial
q

£
Ù 1
X sinÖ 3 cos @ 3 ) =
5 Divide out the common factors 6, 4, 2, and 2,

q 120
then multiply 10 by 4 and then multiply the
rq @
resulting by

Evaluate:
£
r
X sinr 2 cos Ù 2 )
q

No. STATEMENT REASON


£
r
X sinr 2 cos Ù 2 )
1 Given
q
£ £
r 1 .
X sin 2 r
cos 2Ù
) = X sinr % cos Ù % )% are not yet met, but we have 2 inside the
2 q
2 The conditions for Wallis’ formula to be used

functions, so let % = 2 and )% = 2 ) , so


q

)% = ) , then use Integral Rule 3 and


.
change the limits according to
/ [
X % )%
/ 3

95
Prepared by: Nathaniel M. Cabansay, BSCpE

Since % = 2 , the upper limit is now 2 M Q =


£
r
and the lower limit is now 2 0 = 0
£
.
£
r ¡ 4−1 ‼ 6−1 ‼
X sinr 2 cos Ù 2 ) =
4 4+6 ‼
3 The conditions for Wallis’ formula to be used

Ë = 4 and = 6. Since Ë and are both


q
are now met, so we use Wallis’ formula with

even, ˜ =
£
.
£
r ¡ 3 1 5 3 1
X sinr 2 cos Ù 2 ) =
4 10 8 6 4 2
4 Definition of the double factorial

£
r 3¡
X sinr 2 cos Ù 2 ) =
5 Divide out the common factors 5 and 3, then

q 1024 @ £
multiply 8 by 2, 2, 2 and 4 and 3 by 1 and 1
and then multiply the resulting .xÙ by r

INTEGRALS INVOLVING POWERS OF THE OTHER TRIGONOMETRIC FUNCTIONS

For integrals dealing with powers of tangent, secant, cotangent, and cosecant, we use variants of the Pythagorean
identities:

1 + tan. = sec.
sec . − 1 = tan.
1.

cot . + 1 = csc .
2.

csc . − 1 = cot .
3.
4.

Then we have these derivatives:

1. tan % = sec . % )%
2. csc % = − csc % cot % )%
3. sec % = sec % tan %
4. cot % = − csc . %

Then we have these integrals:

∫ sec . % )% = tan % + Y
∫ csc . % )% = − cot % + Y
1.

∫ sec % tan % )% = sec % + Y


2.

∫ csc % cot % )% = − csc % + Y


3.

∫ tan % )% = ln|sec % | + Y
4.

∫ cot % )% = ln|sin % | + Y
5.

∫ sec % )% = ln|sec % + tan % | + Y


6.

∫ csc % )% = − ln|csc % + cot % | + Y = ln|csc % − cot % | + Y


7.
8.

Once we have these, we are ready to deal with integrals involving a form of the differentials tanÚ ) , cot Ú ) ,
sec ) , csc ) , tanÚ sec ) , and cot Ú csc ) where Ë and are non-negative integers. We will
be dealing with six possible cases:

96
Prepared by: Nathaniel M. Cabansay, BSCpE

NOTE: For all of these cases, 0 is considered an even number.

A. •••Ñ F and Žƒ• Ñ F, Ñ À G

cosecants, respectively. Repeat this process until we get Ë − 2 = 1 or Ë − 2 = 0


We strip out two tangents or cotangents each time we integrate and convert those two tangents to secants or

We get these forms:

X tanÚ ) = X tanÚ .
tan. ) = X tanÚ .
sec . − 1 )

X cot Ú ) = X cot Ú .
cot . ) = X cot Ú .
csc . − 1 )

We then use the substitutions and derivatives: % = tan , )% = sec . ) or % = cot , )% = − csc . ) .

B. ŒOŽ c F and ŽŒŽ c F where c > G and is even

We strip out two secants or cosecants each time we integrate and convert the remaining secants or cosecants to
tangents or cotangents, respectively.

We get these forms:


.
X sec ) = X sec .
sec . ) = X 1 + tan. . sec . )

.
X csc ) = X csc .
csc . ) = X 1 + cot . . csc . )

We then use the substitutions and derivatives: % = tan , )% = sec . ) or % = cot , )% = − csc . ) .

C. ŒOŽ c F and ŽŒŽ c F where c > P and is odd


We integrate by parts, stripping out two secants or cosecants each time. When we integrate by parts, we let % =
sec .
or csc .
and )> = sec . ) or csc . ) . We get these forms:

X sec ) = sec . tan − − 2 X tan. sec . ) = sec . tan − − 2 X sec − sec . )

X csc ) = − csc .
cot − − 2 X cot . csc .
) = csc .
cot − − 2 X csc − csc .
)

D. •••Ñ F ŒOŽ c F and Žƒ• Ñ F ŽŒŽ c F where c is even and Ñ is any number

We strip out two secants or cosecants and get these forms:

X tanÚ sec ) = X tanÚ sec .


sec. )

X cot Ú csc ) = X cot Ú csc .


csc . )

We then rewrite the remaining secants or cosecants using the identities: 1 + tan. = sec . and 1 + cot . =
csc . then expand, multiply, and proceed as per Case B above.

E. •••Ñ F ŒOŽ c F and Žƒ• Ñ F ŽŒŽ c F where Ñ and c are both odd

We strip out one tangent and secant each or one cotangent and cosecant each and get these forms:

X tanÚ sec ) = X tanÚ sec sec tan )

97
Prepared by: Nathaniel M. Cabansay, BSCpE

X cot Ú csc ) = X cot Ú csc csc cot )

We then rewrite the remaining tangents or cotangents using the identities: sec . − 1 = tan. and csc . − 1 =
cot . then expand, multiply, and use the substitutions and derivatives: % = sec , )% = sec tan ) or % =
csc , )% = − csc cot )

F. •••Ñ F ŒOŽ c F and Žƒ• Ñ F ŽŒŽ c F where Ñ is even and c is odd

We rewrite the tangents using the identities sec . − 1 = tan. and csc . − 1 = cot . . We get these forms:
Ú
X tanÚ sec ) = X sec . −1 . sec )

Ú
X cot Ú csc ) = X csc . − 1 . csc )

We then expand, multiply, and proceed as per Case C above.

EXAMPLES OF INTEGRALS INVOLVING POWERS OF OTHER TRIGONOMETRIC FUNCTIONS

Evaluate:

X tan@ )

No. STATEMENT REASON

X tan@ )
1 Given

X tan@ ) = X tan tan. )


2 Strip out two tangents

Rewrite tan. = sec . −1


X tan@ ) = X tan sec . −1 )
3

Multiply sec . − 1 by tan


X tan@ ) = X tan sec . ) − X tan )
4 and use Integral
Rule 4

X tan@ ) = X % )% − ln|sec | + Y
% = tan and )% = sec . ) . As for the right
5 Integrate the left integral by substitution: Let

integral, use this

X tan ) = ln|sec | + Y

%.
X tan@ ) = − ln|sec | + Y
2
6 Integral Rule 2

tan. Substitute back % = tan


X tan@ ) = − ln|sec | + Y
2
7

Evaluate:

X sec r )

98
Prepared by: Nathaniel M. Cabansay, BSCpE

No. STATEMENT REASON

X sec r )
1 Given

X sec r ) = X sec . sec . )


2 Strip out two secants

Rewrite sec . = 1 + tan.


X sec r ) = X 1 + tan. sec. )
3

Integrate by Substitution: Let % = tan


X sec r ) = X 1 + % . )%
)% = sec . )
4 and

%@
X sec r ) = % + +Y
3
5 Integral Rules 2 and 4

tan@ Substitute back % = tan


X sec r ) = tan + +Y
3
6

Evaluate:

X csc @ )

No. STATEMENT REASON

X csc @ )
1 Given

X csc @ ) = X csc csc . )


2 Strip out two cosecants

Integrate by parts: Let % = csc


X csc @ ) = − csc cot − X csc cot . )
)> = csc . so we get )% =
3 and

− csc cot and > = − cot

Rewrite cot . = csc . − 1, multiply


X csc @ ) = − csc cot − X csc @ ) + X csc )
csc . − 1 by csc and use Integral
4

Rule 4

Add ∫ csc @ to both sides and use


2 X csc @ ) = − csc cot − ln|csc + cot |
∫ csc ) = − ln|csc + cot |
5

1 1
X csc @ ) = − csc cot − ln|csc + cot |
2 2
6 Divide both sides by 2

Evaluate:

X sec x )

No. STATEMENT REASON

X sec x )
1 Given

99
Prepared by: Nathaniel M. Cabansay, BSCpE

2 X sec x ) = X sec@ sec . ) Strip out two secants

Integrate by parts: Let % = sec @


X sec x ) = sec @ tan − 3 X sec @ tan. )
and )> = sec . so we get )% =
3

3 sec @ tan and > = tan

Rewrite cot . = csc . − 1,


X sec x ) = sec @ tan − 3 X sec x ) + 3 X sec @ )
multiply csc . − 1 by csc and use
4

Integral Rule 4
3 3 Add 3 ∫ sec x ) to both sides and
4 X sec x ) = sec @ tan + sec tan + ln|sec + tan | + Y
2 2
sec tan
5

X sec @ ) =
2
1
+ ln|sec
2
+ tan | + Y
1 3 3
X sec5 ) = sec3 tan + sec tan + ln|sec + tan | + Y
4 8 8
6 Divide both sides by 4

Evaluate:

X cot @ csc Ù )

No. STATEMENT REASON

X cot @ csc Ù )
1 Given

X cot @ csc Ù ) = X cot @ csc r csc . )


2 Strip out two cosecants

Rewrite csc r = csc . .


= 1+
X cot @ csc Ù ) = X cot @ 1 + cot . .
csc . )
cot . .
3

X cot @ csc Ù ) = X cot @ 1 + 2 cot . + cot r csc. )


4 Binomial expansion

Multiply 1 + 2 cot . + cot r


X cot @ csc Ù ) = X cot @ + 2 cot x + cot Ö csc. )
cot @
5 by

Integrate by substitution: Let % =


X cot @ csc Ù ) = − X % @ + 2% x + % Ö )%
cot and )% = − csc . )
6

%r %Ù %×
X cot @ csc Ù ) = − − − +Y
4 3 8
7 Integral Rules 2 and 4

cot r cot Ù cot × Substitute back % = cot


X cot @ csc Ù ) = − − − +Y
4 3 8
8

Evaluate:

X tan@ sec x )

100
Prepared by: Nathaniel M. Cabansay, BSCpE

No. STATEMENT REASON

X tan@ sec x )
1 Given

X tan@ sec x ) = X tan. sec r sec tan )


2 Strip out one tangent and one
secant

Rewrite tan. = sec . − 1 and


X tan@ sec x ) = X sec Ù − sec r sec tan )
multiply the binomial by secr
3

Integrate by substitution: Let % =


X tan@ secx ) = X % Ù − % r )%
sec and )% = sec tan
4

%Ö %x
X tan@ sec x ) = − +Y
7 5
5 Integral Rules 2 and 4

sec Ö sec x Substitute back % = sec


X tan@ sec x ) = − +Y
7 5
6

Evaluate:

X tanr sec )

No. STATEMENT REASON

X tanr sec )
1 Given

Rewrite tanr = tan. .


=
X tanr sec ) = X sec . −1 .
sec )
sec . − 1 .
2

X tanr sec ) = X secr − 2 sec. + 1 sec )


3 Expand the binomial

4 X tanr sec ) = X secx − 2 sec@ + sec ) Multiply the trinomial by sec

X tanr sec ) = X sec x ) − 2 X sec@ ) + X sec )


5 Integral Rule 4

1 3 3 ∫ sec x ) = r sec @ tan +


X tanr sec ) = | sec @ tan + sec tan + ln|sec + tan |}
4 8 8
sec tan + ln|sec + tan | +
6
@ @
− sec tan + ln|sec + tan | + ln|sec + tan | × ×
+Y Y , ∫ sec @ ) = sec tan +
.
ln|sec + tan | + Y, and
.
∫ sec ) = ln|sec + tan | +
Y
1 5 3
X tanr sec ) = sec @ tan − sec tan + ln|sec + tan | + Y
4 8 8
7 Combine like terms

DERIVING REDUCTION FORMULAS FOR SECANT AND COSECANT

Derive a reduction formula for

101
Prepared by: Nathaniel M. Cabansay, BSCpE

X sec ) , >1

No. STATEMENT REASON

X sec )
1 Given

X sec ) = X sec .
sec. )
2 Strip out two secants

Integrate by parts: Let % =


X sec ) = sec .
tan − − 2 X sec .
tan. )
sec . and )> =
3

sec . ) . We get )% =
− 2 sec . tan )
and > = tan

4 X sec ) = sec . tan − − 2 X sec ) + − 2 X sec . ) Rewrite tan. = sec . − 1,


multiply and use Integral
Rule 4

− 2 ∫ sec ) to
− 1 X sec ) = sec .
tan + − 2 X sec .
)
5 Add

1 −2
both sides

−1
X sec ) = sec .
tan + X sec .
)
−1 −1
6 Divide both sides by


Derive a reduction formula for

X csc ) , >1

No. STATEMENT REASON

X csc )
1 Given

2 X csc ) = X csc .
csc . ) Strip out two cosecants

Integrate by parts: Let % =


X csc ) = − csc .
cot − − 2 X csc .
cot . )
csc . and )> =
3

csc . ) . We get )% =
− − 2 csc . cot and
> = − cot

4 X csc ) = − csc . cot − − 2 X csc ) + − 2 X csc . ) Rewrite cot . = csc . − 1,


multiply, and use Integral
Rule 4

5 − 1 X csc ) = − csc . cot + − 2 X csc . ) Add − 2 ∫ csc )


from both sides
1 −2 −1
X csc ) =− csc . cot + X csc . )
−1 −1
6 Divide both sides by

102
Prepared by: Nathaniel M. Cabansay, BSCpE

103
Prepared by: Nathaniel M. Cabansay, BSCpE

MODULE 14 INTEGRATION BY TRIGONOMETRIC SUBSTITUTIONS

WHY THE USE OF TRIANGLES?

Some integrals containing the forms √0. − √0. + . (or √ . + 0. as addition is commutative) and √ . − 0.
.,

cannot be integrated using the methods we’ve already known, like integration by simple substitution (as seen
throughout Modules 3 to 8 and 14), integrals resulting in inverse trigonometric or hyperbolic, or integration by
parts. The method of trigonometric substitution will come in handy in this case. Specifically, these three triangles
will come in handy and provide the needed substitutions.

In using trigonometric substitution, we look at the three cases from the triangles above:

F à CF
0 \ 0
Form in Integrand

7‚G − ÍG FG = sin ` ` = arcsin z { ) = cos ` )`


\ 0 \
0 \ 0
7ÍG FG + ‚G = tan ` ` = arctan z { ) = sec . ` )`
\ 0 \
0 \ 0
7ÍG FG − ‚G = sec ` ` = arcsec z { ) = sec ` tan ` )`
\ 0 \

EXAMPLES OF TRIGONOMETRIC SUBSTITUTIONS

Evaluate:

X 725 − 4 . )

No. STATEMENT REASON

X 725 − 4 . )
1 Given

=
25 . 5
X Ÿ25 − 4 z sin `{ cos ` )` x
sin ` , so we get ) = cos ` )`
x
2 Integrate by trigonometric substitution: Let

4 2 . .

104
Prepared by: Nathaniel M. Cabansay, BSCpE

25 Simplify the entire integrand and use 1 − sin. ` =


X 7cos . ` cos ` )`
2 cos . `
3

25
X cos . ` )`
2
4 Simplify the integrand further

25 Use the power reducing identity cos . ` = 1 + cos 2`


X 1 + cos 2` )`
4
.
5

25 25
X 1 )` + X cos 2` )`
4 4
6 Use Integral Rule 4

25 25 Integrate by substitution: Let % = 2` so )% = 2 )` or


X 1 )` + X cos % )%
4 8 )% = )`
7
.

25 25 Use Integral Rule 2-bis and ∫ cos % )% = sin % + Y


`+ sin % + Y
4 8
8

25 25 Substitute back % = 2`
` + sin 2` + Y
4 8
9

25 25 Use the identity sin 2` = 2 sin ` cos `


`+ cos ` sin ` + Y
4 4
10

25 2 25 √25 − 4 .2
Substitute back ` = arcsin M Q so we get sin ` =
. .
arcsin z { + +Y x x
4 5 4 5 5
11

and cos ` =
7.x r
x

25 2
725 − 4 . + arcsin z { + Y
2 4 5
12 Simplify the entire polynomial

Evaluate:

X2 7 r +1)

No. STATEMENT REASON

X2 7 r +1)
1 Given

Integrate by substitution: Let % = .


so )% = 2 )
X 7%. + 1 )%
2

Integrate by trigonometric substitution: Substitute % =


X 7tan. ` + 1 sec . ` )%
tan ` so )% = sec . ` )`
3

Use the identity 1 + tan. ` = sec . ` then simplify the


X sec @ ` )`
4

1 1
integrand

sec ` tan ` + ln|sec ` + tan `| + Y


2 2 1 −2
5 Use the reduction formula:
X sec ) = sec .
tan + X sec .
)
−1 −1

% 1 Substitute back ` = arctan % , so we get tan ` = % and


7% . + 1 + ln ‘% + 7% . + 1‘ + Y
2 2 sec ` = √%. + 1
6

% 1 (OPTIONAL) ln % + √% . + 1 = arsinh %
7% . + 1 + arsinh % + Y
2 2

105
Prepared by: Nathaniel M. Cabansay, BSCpE

.
1 Substitute back % = .
7 r + 1 + ln ‘ .
+7 r + 1‘ + Y
2 2
7

.
1
7 r + 1 + arsinh .
+Y
2 2
Evaluate:

X2 7 r −1)

No. STATEMENT REASON

X2 7 r −1)
1 Given

2 X 7%. − 1 )% Integrate by substitution: Let % = .


so )% = 2 )

Integrate by trigonometric substitution: Substitute % =


X 7sec . ` − 1 sec ` tan ` )`
sec ` so )% = sec ` tan ` )`
3

Use the identity tan. ` = sec . ` − 1 then simplify the


X sec ` tan. ` )`
4
integrand

Use the identity tan. ` = sec . ` − 1 and use Integral


X sec @ ` )` − X sec ` )`
5

1 1
Rule 4

sec ` tan ` − ln|sec ` + tan `| + Y


2 2 1 −2
6 Use the reduction formula:
X sec ) = sec .
tan + X sec .
)
−1 −1
Also, use ∫ sec ) = ln|sec + tan | + Y then
combine like terms
% 1 Substitute back ` = arcsec % , so we get sec ` = % and
7% . − 1 − ln ‘% + 7% . − 1‘ + Y
2 2 tan ` = √%. − 1
7

% 1 (OPTIONAL) ln % + √% . − 1 = arcosh %
7% . − 1 − arcosh % + Y
2 2
.
1 Substitute back % = .
7 r − 1 − ln ‘ .
+7 r − 1‘ + Y
2 2
8

.
1
7 r − 1 − arcosh .
+Y
2 2

USING TRIGONOMETRIC SUBSTITUTIONS TO FIND GENERAL FORMULAS

Find a general formula for:

X 70. − % . )%

106
Prepared by: Nathaniel M. Cabansay, BSCpE

No. STATEMENT REASON

X 70. − % . )%
1 Given

X 070. − 0. sin. ` cos ` )`


% = 0 sin ` and )% = 0 cos ` )`
2 Use the trigonometric substitution

Factor 0. from the expression inside


0. X 71 − sin. ` cos ` )`
3
the radical and pull the constant factor
out of the integral

Use the Pythagorean Identity sin. ` +


0. X 7cos . ` cos ` )`
cos . ` = 1
4

5
0. X cos . ` )` √ . = | |, but assume cos ` is
positive since we are evaluating an
indefinite integral

0.
X 1 + cos 2` )`
2 cosine: cos . ` = 1 + cos 2`
6 Use the power reducing formula for
.

0.
0 .
X 1 )` + X cos > )>
2 4 substitution: Let > = 2` and )> =
7 Use Integral Rule 4 and integrate by

2 )` or )> = )`
.

0 ` 0
. .
+ sin > + Y
2 4 ∫ cos % )% = sin % + Y
8 Use Integral Rule 2-bis and

0. ` 0. Substitute back > = 2`


+ sin 2` + Y
2 4
9

0. 0. Use the identity sin 2` = 2 cos ` sin `


` + sin ` cos ` + Y
2 2
10

0. % 0. % % Substitute ` = arcsin M Q
/
arcsin M Q + sin Marcsin M QQ cos Marcsin M QQ + Y
2 0 2 0 0
3
11

0. % 0. % √0. − % .
arcsin M Q + M Q j k+Y
2 0 2 0 0 sin Marcsin M3 QQ = 3 and
/ /
12 From the triangle above,

cos Marcsin M QQ =
/ 73 /
3 3

h ‚G h
7‚G − hG + •¤ŽŒ•• M Q + b
G G ‚
13 Simplify the polynomial

Find a general formula for:

X 70. + % . )%

No. STATEMENT REASON

X 70. + %. )%
1 Given

107
Prepared by: Nathaniel M. Cabansay, BSCpE

X 070. + 0. tan. ` sec . ` )`


% = 0 tan ` and )% = 0 sec . ` )`
2 Use the trigonometric substitution

Factor 0. from the expression


0. X 71 + tan. ` sec. ` )`
3
inside the radical and pull the
constant factor out of the integral

0. X 7sec . ` sec . ` )`
tan. ` + 1 = sec . `
4 Use the Pythagorean Identity

5 0. X sec @ ` )` √ . = | |, but assume sec ` is


positive since we are evaluating an

1 1
indefinite integral

0. z sec ` tan ` + ln|sec ` + tan `|{ + Y


2 2
6 Use the reduction formula:
1
X sec ) = sec .
tan
−1
−2
+ X sec .
)
−1

Substitute ` = arctan M Q
1 % % 1 % % /
0. z sec Marctan M QQ tan Marctan M QQ + ln ‘sec arctan M Q + tan Marctan M QQ‘{ + Y
2 0 0 2 0 0 3
7

1 √0. + %. % 1 √0. + % . + %
0. j j k M Q + ln ¦ ¦k + Y
2 0 0 2 0 tan Marctan M QQ = and
/ /
8 From the triangle above,

3 3

sec Marctan M QQ =
/ 73 8/
3 3

1 √0. + %. % 1 log 3 M Q = log 3 − log 3 ? and ln 0


0. j j k M Q + ln ‘% + 7% . + 0. ‘k + Y †
2 0 0 2
9

%
is a constant

1 √0. + % . % 1 % ln M% + 7%. + 0. Q = arsinh M Q


0. j j k M Q + arsinh M Qk + Y 0
2 0 0 2 0
10

h ‚G h
7‚G + hG + •¤Œ••§ M Q + b
G G ‚
11 Simplify the polynomial

h ‚ G %
7hG + ‚G − €• Mh + 7hG + ‚G Q + b ln M% + 7%. + 0. Q = arsinh M Q
OPTIONAL:

G G 0


Find a general formula for:

X 7% . − 0. )%

No. STATEMENT REASON

X 7% . − 0. )%
1 Given

X 070. sec . ` − 0. sec ` tan ` )`


substitution % = 0 sec ` and
2 Use the trigonometric

)% = 0 sec ` tan ` )`

108
Prepared by: Nathaniel M. Cabansay, BSCpE

Factor 0. from the


3 0. X 7sec . ` − 1 sec ` tan ` )`
expression inside the radical
and pull the constant factor
out of the integral

0. X 7tan. ` sec ` tan ` )`


tan. ` = sec . ` − 1
4 Use the Pythagorean Identity

5
0. X sec ` tan. ` )` √ . = | |, but assume tan `
is positive since we are
evaluating an indefinite
integral

0. X sec @ ` )` − 0. X sec ` )`
tan. ` = sec . ` − 1 and use
6 Use the Pythagorean Identity

Integral Rule 4
1 0.
0. z sec ` tan `{ + ln|sec ` + tan `| − 0. ln|sec ` + tan `| + Y
2 2
7 For the left integral, use the
reduction formula

X sec )
1
= sec . tan
−1
−2
+ X sec . )
−1

∫ sec ) = ln|sec +
For the right integral, use

tan | + Y
1 0.
0. z sec ` tan `{ − ln|sec ` + tan `| + Y
2 2
8 Simplify the polynomial

1 % % 0. % %
Substitute ` = arcsec M Q
/
0. z sec Marcsec M QQ tan Marcsec M QQ{ − ln ‘sec Marcsec M QQ + tan Marcsec M QQ‘
2 0 0 2 0 0 3
9
+Y

1 % √%. − 0. 0. % + √% . − 0.
0. ª M Q j k« − ln ¦ ¦+Y
tan Marcsec M QQ =
10 From the triangle above,
7/
2 0 0 2 0
/ 3
3 3
and sec Marcsec M QQ =
/ /
3 3

1 % √%. − 0. 0. log 3 M Q = log 3 − log 3 ?


0. ª M Q j k« − ln ‘% + 7%. − 0. ‘ + Y

11
2 0 0 2 and ln 0 is a constant

%
1 % √%. − 0. 0. % ln M% + 7%. − 0 . Q = arcosh M Q
0
0. ª M Q j k« − arcosh M Q + Y
12
2 0 0 2 0

h ‚G h
7hG − ‚G − •¤ŽƒŒ§ M Q + b
G G ‚
13 Simplify the polynomial

h ‚ G %
ln M% + 7%. − 0 . Q = arcosh M Q
7hG − ‚G − €• Mh + 7hG − ‚G Q + b
OPTIONAL:

G G 0

109
Prepared by: Nathaniel M. Cabansay, BSCpE

1 %
Prove:

X )% = arcsin M Q + Y
√0. − %. 0

1 %
No. STATEMENT REASON

X )% = arcsin M Q + Y
√0. − %. 0
1 Given

0 cos ` % Let % = 0 sin ` and )% = 0 cos ` on the left side. This


X )` = arcsin M Q + Y
70. 1 − sin. ` 0 means ` = arcsin M Q
/
2

0 cos ` % Use the Pythagorean Identity cos . ` + sin. ` = 1


X )` = arcsin M Q + Y
√0 cos ` 0
3
. .

0 cos ` % Get the square root of 0. cos . `


X )` = arcsin M Q + Y
0 cos ` 0
4

%
X 1 )` = arcsin M Q + Y
0
5 Simplify the left-hand side

%
` + Y = arcsin M Q + Y
0
6 Integral Rule 2-bis

% % %
arcsin M Q + Y = arcsin M Q + Y ` = arcsin M Q
0 0 0
7

1 1 %
Prove:

X )% = arctan M Q + Y
0. +% . 0 0

1 1 %
No. STATEMENT REASON

X )% = arctan M Q + Y
0. + % . 0 0
1 Given

0 sec . ` 1 % Let % = 0 tan ` and )% = 0 sec. ` )` on the left side.


X )` = arctan M Q + Y
0. 1 + tan. ` 0 0 This means ` = arctan M Q
/
2

0 sec . ` 1 % Use the Pythagorean identity 1 + tan. ` = sec . `


X )` = arctan M Q + Y
0. sec . ` 0 0
3

1 1 %
X 1 )` = arctan M Q + Y
0 0 0
4 Simplify the left-hand side and use Integral Rule 3

1 1 %
` + Y = arctan M Q + Y
0 0 0
5 Integral Rule 2-bis

1 % 1 % %
arctan M Q + Y = arctan M Q + Y ` = arctan M Q
0 0 0 0 0
6


Prove:

1 1 %
X )% = arcsec M Q + Y
%√% . − 0. 0 0

110
Prepared by: Nathaniel M. Cabansay, BSCpE

1 1 %
No. STATEMENT REASON

X )% = arcsec M Q + Y
%√% . − 0. 0 0
1 Given

0 sec ` tan ` 1 % Let % = 0 sec ` and )% = 0 sec ` tan `.


X )` = arcsec M Q + Y
0 sec ` 70. sec. `−1 0 0 This means ` = arcsec M Q
/
2

0 sec ` tan ` 1 % Use the Pythagorean identity sec . ` −


X )` = arcsec M Q + Y
0 sec ` √0. tan. ` 0 0 1 = tan. `
3

0 sec ` tan ` 1 % Get the square root of 0. tan. `


X )` = arcsec M Q + Y
0 sec ` tan `
. 0 0
4

1 1 %
X 1 )` = arcsec M Q + Y
0 0 0
5 Simplify the left-hand side and use Integral

1 1 %
Rule 3

` + Y = arcsec M Q + Y
0 0 0
6 Integral Rule 2-bis

1 % 1 % %
arcsec M Q + Y = arcsec M Q + Y ` = arcsec M Q
0 0 0 0 0
7

Prove:

1 %
X )% = ln M% + 7% . + 0. Q + Y = arsinh M Q + Y
√% . + 0. 0

1
No. STATEMENT REASON

X )% = ln M% + 7% . + 0. Q + Y
√% . + 0.
1 Given

0 sec . ` Let % = 0 tan ` and )% = 0 sec . `. This means


X )` = ln M% + 7% . + 0. Q + Y
70. tan. ` + 1 ` = arctan M Q
/
2

0 sec . ` Use the Pythagorean identity 1 + tan. ` =


X )` = ln M% + 7%. + 0. Q + Y
√0. sec . ` sec . `
3

0 sec . ` Get the square root of 0. sec . `


X )` = ln M% + 7% . + 0. Q + Y
0 sec `
4

X sec ` )` = ln M% + 7% . + 0. Q + Y
5 Simplify the left-hand side

6 ln sec ` + tan ` + Y = ln M% + 7% . + 0. Q + Y X sec % )% = ln|sec ` + tan `| + Y

% % . sec ` = œM Q + 1 and tan ` =


/ . /
ln ª + ŸM Q + 1« + Y = ln M% + 7% . + 0. Q + Y
7

0 0
3 3

% .
ln ª% + 0ŸM Q + 1« + Y = ln M% + 7% . + 0. Q + Y the left-hand side by 0
8 Multiply the expression inside the logarithm in

111
Prepared by: Nathaniel M. Cabansay, BSCpE

9 ln M% + 7%. + 0. Q + Y = ln M% + 7% . + 0. Q + Y / .
Bring 0 inside the radical to multiply M3 Q + 1

% % %
arsinh M Q + Y = arsinh M Q + Y ln M% + 7% . + 0. Q = arsinh M Q
0 0 0
10


Prove:

1 %
X )% = ln M% + 7% . − 0. Q + Y = arcosh M Q + Y
√% . − 0. 0

1
No. STATEMENT REASON

X )% = ln M% + 7% . − 0. Q + Y
√% . − 0.
1 Given

0 sec . ` Let % = 0 sec ` and )% = 0 sec ` tan `. This


X )` = ln M% + 7% . + 0. Q + Y
70. sec . `−1 means ` = arcsec M Q
/
2

0 sec ` tan ` Use the Pythagorean identity tan. ` = sec . ` −


X )` = ln M% + 7% . − 0. Q + Y
√0. tan. ` 1
3

0 sec ` tan ` Get the square root of 0. tan. `


X )` = ln M% + 7% . − 0. Q + Y
0 tan `
4

X sec ` )` = ln M% + 7% . − 0. Q + Y
5 Simplify the left-hand side

6 ln sec ` + tan ` + Y = ln M% + 7% . − 0. Q + Y X sec % )% = ln|sec ` + tan `| + Y

% % . sec ` =
/
and tan ` = œM Q − 1
/ .
ln ª + ŸM Q − 1« + Y = ln M% + 7% . − 0. Q + Y
7
0 0
3 3

% .
ln ª% + 0ŸM Q − 1« + Y = ln M% + 7% . − 0. Q + Y the left-hand side by 0
8 Multiply the expression inside the logarithm in

ln M% + 7%. − 0. Q + Y = ln M% + 7% . − 0. Q + Y / .
Bring 0 inside the radical to multiply M Q − 1
3
9


% % %
arcosh M Q + Y = arcosh M Q + Y ln M% + 7% . − 0. Q = arcosh M Q
0 0 0
10

112
Prepared by: Nathaniel M. Cabansay, BSCpE

Given the equation and the graph of the unit hyperbola:

.
− ?. = 1

Prove this:

ev + e v
cosh ] =
2

No. STATEMENT REASON

1 .
− ?. = 1 Given

the origin á(0, 0) to a point ¿ on


2 On the graph, draw a ray from

the hyperbola \, √\ . − 1 at an
angle from the x-axis. From that
point on the hyperbola, draw a
line segment perpendicular to the
x-axis, and from that point,
complete the triangle

= cosh ] , ? = sinh ]
− ?. = 1 defining two functions cosh ] and
3 Parametrize the hyperbola by
.
sinh ] and rewrite the equation
?. = .
−1
?=7 . −1
as a function of

113
Prepared by: Nathaniel M. Cabansay, BSCpE

[
] \√\ . − 1
= − X7 . −1)
4 Area from the graph seen in
2 2
point â greater than 1 and at a
statement 2, since we defined a

quantity \, the height of the


triangle is √\ . − 1. We are,
however, only interested in the

v
area in blue above, which we’ll
.
define as , so we have to
subtract the area in purple from
the whole triangle. The area
under a curve is the definite

1 and \.
integral of the function between

[
] \√\ . − 1
= − X 7sec . ` − 1 sec ` tan ` )` substitute = sec `, ) =
5 To evaluate the definite integral,
2 2
sec ` tan ` )`. We will not need
to use the substitution rule for
definite integrals here as we have
a variable for a limit and we will
return to using at some point
[
] \√\ . − 1
= − X 7tan. ` sec ` tan ` )` sec . ` − 1 = tan. `
6 Use the trigonometric identity
2 2

√ . = | |, but assume tan ` is


[
] \√\ . − 1
= − X sec ` tan. ` )`
7
2 2 positive since we are evaluating
an indefinite integral
[ [
] \√\ . − 1
= − ªX sec @ ` )` − X sec ` )` « tan. ` = sec . ` − 1 and use
8 Use the Pythagorean Identity
2 2
Integral Rule 4
] \√\. − 1 1 1 [
= − | sec ` tan ` + ln|sec ` + tan `| − ln|sec ` + tan `|}
2 2 2 2 sec @ `:
9 Use the reduction formula on

] \√\ . − 1 1 1 [
= − | sec ` tan ` − ln|sec ` + tan `|} X sec )
2 2 2 2
1
= sec . tan
−1
−2
+ X sec . )
−1
Use ∫ sec % )% = ln|sec % +
tan %| then combine like terms

] \√\ . − 1 √ .−1 1
[
Substitute ` = arcsec , so
= −~ − ln ‘ + 7 . − 1‘• sec arcsec = and
2 2 2 2
10

tan arcsec = √ . − 1
] \√\. − 1 \√\. − 1 1√1. − 1 1 1
= −j − − ln ‘\ + 7\. − 1‘ + ln ‘1 + 71. − 1‘k
2 2 2 2 2 2
11 Fundamental Theorem of
Calculus

114
Prepared by: Nathaniel M. Cabansay, BSCpE

] 1
= ln ‘\ + 7\ . − 1‘
2 2
12 Simplify the resulting polynomial

13 ] = ln ‘\ + 7\ . − 1‘ Multiply both sides by 2

14 ev = \ + 7\ . − 1 Reverse the logarithm

15 ev − \ = 7\ . − 1 Subtract \ from both sides

16 e.v − 2\ev + \ . = \ . − 1 Square both sides

17 e.v − 2\ev + 1 = 0 Subtract \ . − 1 from both sides

18 e.v + 1 = 2\ev Add 2\ev to both sides

e.v + 1 Divide both sides by 2ev


\=
2ev
19

ev + e v
Set \ = and divide e.v + 1 by
=
2 ev
20

OÎ + O Î
ŽƒŒ§ Î =
G
21 From the parametrization in


statement 3

Then prove:

ev − e v
sinh ] =
2

No. STATEMENT REASON

1 .
− ?. = 1 Unit hyperbola equation
= cosh ] , ? = sinh ]
cosh ] − sinh ] = 1
2 Parametrization of unit hyperbola, then substitute into
. . the equation

e.v + 2 + e .v
Use the exponential form of cosh ]
− sinh. ] = 1
4
3

e.v + 2 + e .v
Add sinh. ] to both sides and subtract 1 from both
− 1 = sinh. ]
4
4
sides

e.v − 2 + e .v ã 8.8 ã
sinh. ] =
4
r
5 Subtract 1 from

ev − e v
.
sinh ] = j
.
k
2
6 The numerator is a perfect square trinomial and the
denominator is a perfect square

OÎ − O Î
Œ••§ Î =
G
7 Get the square root of both sides

115
Prepared by: Nathaniel M. Cabansay, BSCpE

MODULE 15 INTEGRATION BY COMPLETING THE SQUARE AND PARTIAL FRACTIONS

DEALING WITH RATIONAL FUNCTIONS

In the previous modules, we’ve used certain techniques that allow us to deal with certain rational functions. Here
are some of the forms:

∫ % )% = + Y, ≠1
/äd!
8
1.
2. ∫ / )% = ln|%| + Y
∫7 )% = arcsin M Q + Y
/
3 / 3
3.

∫/ )% = arctan M Q + Y
/
83 3 3
4.

∫ )% = arcsec M Q + Y
/
/ 7/ 3 3 3
5.

∫7 )% = ln % + √% . + 0. + Y = arsinh M Q + Y
/
/ 83 3
6.

∫7 )% = ln % + √% . − 0. + Y = arcosh M Q + Y
/
/ 3 3
7.

∫3 )% = ln M Q + Y, %. < 0. = 3 artanh M3 Q + Y
38/ /
/ .3 3 /
8.

∫ )% = − ln z { + Y = − 3 arcsch M3 Q + Y
387/ 83 /
/7/ 83 3 /
9.

10. ∫ )% = − ln z { + Y = − 3 arsech M3 Q + Y
3873 / /
/ 73 / 3 /

11. ∫ )% = ln M Q = − 3 arcoth M3 Q + Y
/ 3 /
/ 3 .3 /83

Now, what if the integrand does not match any of those forms? For example, we have: ∫ ) or
3 8[ 8É

∫ ) or ∫ ) . We have to make the integrand match these forms. How? We have two ways to deal
[ J8 @
with integrals such as these.

INTEGRATION BY COMPLETING THE SQUARE

The first way we can deal with is by completing the square. Completing the square involves creating a perfect
square trinomial from an . term with a leading coefficient of 1 and an term. Specifically, if we have a quadratic
polynomial 0 + \ + , we first make sure 0 = 1 by dividing all terms by 0 to get + + . To produce a
. . [ É
3 3
perfect square trinomial, we remember the form 0 ± \ .
= 0. ± 20\ + \ . . Here, the square of our first term is
. É
3
, our middle term would then have to be divided by 2 and then squared. We cannot assume is already in that

+ + + −
[ . [ [ É [
r3 3 r3 3 r3
form, so we add and subtract so we get: . The first three terms already form a perfect
.
square trinomial, so we have: M + Q + M − Q. Completing the square can convert a quadratic in an
[ É [
.3 3 r3
integrand into an integrable form that can lead to either an inverse trigonometric function, an inverse hyperbolic
function, or a logarithm.

116
Prepared by: Nathaniel M. Cabansay, BSCpE

EXAMPLES OF INTEGRATION BY COMPLETING THE SQUARE

Evaluate:

1
X )
. +2 +2

STATEMENT REASON

1
No.

X )
. +2 +2
1 Given

1
X )
. +2 +1+2−1
2 Complete the square on the denominator. Add and
subtract 1 from the denominator
1
X )
+1 .+1
3 Simplify the denominator

1 We can now substitute % = + 1 and )% = ) and


X )%
%. + 0. 0=1
4

arctan % + Y 1 1 %
X )% = arctan M Q + Y
% . + 0. 0 0
5

6 arctan +1 +Y Substitute back % = +1

Evaluate:

1
X )
√4 − .

1
No. STATEMENT REASON

X )
√4 −
1 Given
.

1 .
X )
7− −4
2 Factor -1 inside the radical so the coefficient of
. becomes 1

1
X )
7− −4 +4 +4
3 Add 4 and subtract 4 from the radicand
.

1
X )
74 − −2
4 Simplify the denominator
.

1 We can now substitute % = − 2 and )% = ) and


X )%
√2.− %. 0=2
5

% 1 %
arcsin M Q + Y X )% = arcsin M Q + Y
2 √0. − %. 0
6

−2 Substitute back % = −2
arcsin z {+Y
2
7

117
Prepared by: Nathaniel M. Cabansay, BSCpE

PARTIAL FRACTION DECOMPOSITION


x r ×
. J Ù
Suppose we have a rational expression such as or . From the outset, looks like we can’t integrate
those functions, can we? Looks like we’ll need a way to make things simpler. In this case, we use partial fraction
decomposition, which is rewriting the rational expression as a sum of two or more fractions.

In partial fraction decomposition, we factor the polynomial in the denominator into linear and irreducible
quadratic factors. We have four cases to deal with:

¿
Factor in Denominator Term in Partial Fraction Decomposition
0 +\
0 +\
0 +\ ¿ ¿. ¿
+ +⋯+ , ∈ ℤ8
0 +\ 0 +\ . 0 +\
∈ ℤ8 means
â +Y
is a positive integer
0 .
+\ +
0 .+\ +
0 .
+\ + â +Y â. + Y. â +Y
+ +⋯+ , ∈ ℤ8
0 .+\ + 0 .+\ + . 0 .+\ +
∈ ℤ8 means is a positive integer

we have a linear factor ] that is repeated, such as . The exponent of ] in the numerators in the partial fraction
Now you might be asking why we “build up” powers with repeated linear or irreducible quadratic factors. Suppose

v
decomposition must be one less than those of the denominators. To decompose, we use:

1 ¿] + â
=
]. ].

We can simplify this, however, so:

1 ¿ â
= +
]. ] ].

Similarly, if we had:

1 ¿] . + â] + Y ¿ â Y
= = + .+ @
]@ ]@ ] ] ]

This is why if we have repeated factors, we “build up” powers (Chow, 2018).

EXAMPLES OF PARTIAL FRACTION DECOMPOSITIONS


Decompose into partial fractions:

5 −4
. − −2

118
Prepared by: Nathaniel M. Cabansay, BSCpE

No. STATEMENT REASON


5 −4
. − −2
1 Given

5 −4 .
− −2
+1 −2
2 Factor

5 −4 ¿ â
= +
+1 −2 +1 −2
3 We have two linear factors, so we split into two
fractions

4 5 −4=¿ −2 +â +1 Multiply both sides by +1 −2


5 −1 − 4 = ¿ −1 − 2 + â −1 + 1 To determine ¿, set = −1
−5 − 4 = −3¿
5

−9 = −3¿
¿=3
5 2 −4=¿ 2−2 +â 2+1 To determine â, set =2
10 − 4 = 3â
6

6 = 3â
â=2

5 −4 3 2
= +
+1 −2 +1 −2
7 Substitute into statement 3

Decompose into partial fractions:

8
r − 16

8
No. STATEMENT REASON

r − 16
1 Given

8 r
− 16
−2 +2 . +4
2 Factor

8 ¿ â Y +ç
= + + .
−2 +2 . +4 −2 +2 +4
3 We have two linear
factors and one irreducible
quadratic, so we split into
three fractions.

8 =¿ +2 .
+4 +â −2 .
+4 + Y +ç −2 +2
−2 +2 .+4
4 Multiply both sides by

8 =¿ @
+2 +4 +8 +â @−2
. .
+4 −8
+ Y +ç .−4
5 Multiply all polynomials

8 =¿ @
+ 2¿ .
+ 4¿ + 8¿ + â @
− 2â .
+ 4â − 8â + Y @
− 4Y
+ ç . − 4ç
6 Multiply all numerical
constants

8 = @
¿ + â + Y + . 2¿ − 2â + ç + 4¿ + 4â − 4Y
+ 8¿ − 8â − 4ç
7 Group powers of
together.

119
Prepared by: Nathaniel M. Cabansay, BSCpE

0=¿+â+Y
0 = 2¿ − 2â + ç
·
8 Use a system of linear

8 = 4¿ + 4â − 4Y ¿, â, Y, ç.
equations to determine

0 = 8¿ − 8â − 4ç
0=¿+â+Y
0 = 2¿ − 2â + ç
·
2=¿+â−Y
0 = 2¿ − 2â − ç
1=¿+â
è
0=¿−â
9 Solve the system of

1 = 2¿
equations

1
¿=
2
1
0= −â
2
1
â=
2
1 1
0= + +Y
2 2
0=1+Y
Y = −1
1 1
0 = 2z { − 2z { ± ç
2 2
ç=0

8 1 1
= + −
−2 +2 +4 2 −2 2 +2 +4
10 Substitute the values found
. . into statement 3

EXAMPLES OF INTEGRATION BY PARTIAL FRACTION DECOMPOSITION

Evaluate:

5 −4
X )
.− −2

No. STATEMENT REASON


5 −4
X )
. − −2
1 Given

3 2
Xz + {)
+1 −2
2 Decompose the integrand into partial fractions

3 2
X ) +X )
+1 −2
3 Use Integral Rule 4

1 1
3X ) + 2X )
+1 −2
4 Integral Rule 3

1 1 Integrate by substitution: Let % = + 1 and %. = −


3X )% + 2 X )%.
% %. 2. This means )% = ) and )%. = )
5

120
Prepared by: Nathaniel M. Cabansay, BSCpE

3 ln| + 1| + 2 ln| − 2| + Y ∫ / )% = ln|%| + Y, then substitute back % = + 1 and


%. = − 2
6
ln| + 1 @ | + ln| − 2 .| + Y
(OPTIONAL) ? log 3 = log 3 †
and log 3 + log 3 ? =
ln| +1 @
− 2 .| + Y
log 3 ?
Evaluate:

8
X )
r − 16

We can go about this in two ways:

We can use partial fractions…

8
No. STATEMENT REASON

X )
r − 16
1 Given

1 1
Xz + − {)
2 −2 2 +2 . +4
2 Decompose the integrand into partial fractions

1 1
X ) +X ) −X )
2 −2 2 +2 . +4
3 Use Integral Rule 4

1 1 1 1
X ) + X ) −X )
2 −2 2 +2 . +4
4 Use Integral Rule 3

1 1 1 1 1 1 Integrate by substitution: Let % = − 2, %. =


X )% + X )%. − X )%@
2 % 2 %. 2 %@ + 2, and %@ = . + 4, so we have )% = ) ,
5

)%. = ) , and )%@ = 2 ) or . )%@ = )

1 1 1 ∫ / )% = ln|%| + Y, then substitute back % =


ln| − 2| + ln| + 2| − ln| .
+ 4| + Y
2 2 2 − 2, %. = + 2, and %@ = . + 4
6

1
ln| − 2| + ln| + 2| − ln| . + 4| + Y
2 .
log 3 + log 3 ? = log 3 ? , log3 − log 3 ? =
(OPTIONAL) Factor from all terms, then

1
ln| . − 4| − ln| . + 4| + Y log 3 M†Q, and . ln M
83
Q = arcoth M3Q
2 3

1 .
−4
ln ¦ . ¦+Y
2 +4
1 .
+4
− ln ¦ ¦+Y
2 . −4
.
− arcoth j k+Y
4

121
Prepared by: Nathaniel M. Cabansay, BSCpE

…or we can use substitution to convert this to a form integrable to inverse hyperbolic cotangent.

8
No. STATEMENT REASON

X )
r − 16
1 Given

1 1 % 1 1 %−0 1 %
4X )% = − 4 arcoth M Q + Y X )% = ln M Q = − arcoth M Q + Y
% . − 0. 4 0 % . − 0. 20 %+0 0 0
2

Here, % = .
, and 0 = 4, so )% = 2 ) or 4 )% =
8 )
. Simplify and substitute back % = .
and 0 = 4
− arcoth j k+Y
4
3

USING PARTIAL FRACTIONS TO FIND GENERAL FORMULAS

Prove:

1 1 0+% 1 %
X )% = ln z { + Y = artanh M Q + Y
0. −% . 20 0−% 0 0

1 1 0+%
No. STATEMENT REASON

X )% = ln z {+Y
0. −% . 20 0−%
1 Given

1 1 0+%
X )% = ln z {+Y
0+% 0−% 20 0−%
2 Difference of two squares

1 ¿ â
= +
0+% 0−% 0−% 0+%
3 Decompose the integrand into partial
fractions

4 1 = ¿ 0 +% +â 0−% Multiply both sides by 0 + % 0 − %

1 = 20¿ Determine ¿ and â


1 = 20â
5

1 1
,â =¿=
20 20
1 1 1
= +
0+% 0−% 20 0 − % 20 0 + %
1 1 1 0+%
Xz + { )% = ln z {+Y
20 0 − % 20 0 + % 20 0−%
6 Rewrite the integrand as the partial fraction

1 1 1 1 1 0+%
decomposition

X )% + X )% = ln z {+Y
20 0 + % 20 0 − % 20 0−%
7 Integral Rules 3 and 4

1 1 1 1 1 0+% Integrate by substitution. Let > = 0 + % and


X )> − X )>. = ln z {+Y
20 > 20 >. 20 0−% >. = 0 − % so )> = )% and )>. = −)% or
8

−)>. = )%

122
Prepared by: Nathaniel M. Cabansay, BSCpE

1 1 1 0+% ∫ / )% = ln|%| + Y then substitute back > =


ln|0 + %| − ln|0 − %| + Y = ln z {+Y
20 20 20 0−% 0 + % and >. = 0 − %
9

1 1 0+%
ln|0 + %| − ln|0 − %| + Y = ln z {+Y
20 20 0−%
.3
10 Factor from the left-hand side

1 0+% 1 0+%
ln z {+Y = ln z {+Y log 3 − log 3 ? = log 3 z {
20 0−% 20 0−% ?
11


1 % 1 % 1 0+% %
artanh M Q + Y = artanh M Q + Y ln z { = artanh M Q
0 0 0 0 2 0−% 0
12


Prove:

1 1 %−0 1 %
X )% = ln M Q + Y = − arcoth M Q + Y
% . − 0. 20 %+0 0 0

1 1 %−0
No. STATEMENT REASON

X )% = ln M Q+Y
% . − 0. 20 %+0
1 Given

1 1 %−0
X )% = ln M Q+Y
%−0 %+0 20 %+0
2 Difference of two squares

1 ¿ â
= +
%−0 %+0 %−0 %+0
3 Decompose the integrand into partial
fractions

4 1=¿ %+0 +â %−0 Multiply both sides by 0 + % 0 − %

1 = 20¿ Determine ¿ and â


1 = −20â
5

1 1
¿=
,â = −
20 20
1 1 1
= −
%−0 % +0 20 % − 0 20 % + 0
1 1 1 %−0
Xz − { )% = ln M Q+Y
20 % − 0 20 % + 0 20 %+0
6 Rewrite the integrand as the partial fraction

1 1 1 1 1 %−0
decomposition

X )% − X )% = ln M Q+Y
20 % − 0 20 % + 0 20 %+0
7 Integral Rules 3 and 4

1 1 1 1 1 %−0 Integrate by substitution. Let > = % − 0 and


X )> − X )>. = ln M Q+Y
20 > 20 >. 20 %+0 >. = % + 0 so )> = )% and )>. = )%
8

1 1 1 %−0 ∫ / )% = ln|%| + Y then substitute back > =


ln|% − 0| − ln|% + 0| + Y = ln M Q+Y
20 20 20 %+0 % − 0 and >. = % + 0
9

1 1 %−0
ln|% − 0| − ln|% + 0| + Y = ln M Q+Y
20 20 %+0
.3
10 Factor from the left-hand side

123
Prepared by: Nathaniel M. Cabansay, BSCpE

1 %−0 1 %−0
ln M Q+Y = ln M Q+Y log 3 − log 3 ? = log 3 z {
20 %+0 20 %+0 ?
11


1 %+0 1 %+0 log 3 †
= ? log3
− ln z { + Y = − ln z {+Y
20 %−0 20 %−0
12

1 % 1 % 1 %+0 %
− arcoth M Q + Y = − arcoth M Q + Y ln z { = arcoth M Q
0 0 0 0 2 %−0 0
13

124
Prepared by: Nathaniel M. Cabansay, BSCpE

MODULE 16 INTEGRATION BY ALGEBRAIC SUBSTITUTION

ALGEBRAIC SUBSTITUTION

Radicals can prove tricky sometimes, but we’ve already had numerous ways to deal with these: power rule,
reciprocal rule, integrals leading to inverse trigonometric or inverse hyperbolic functions, substitution, completing

) as our integrand. We can’t use any of the methods we’ve used before. This is
the square, and trigonometric substitution. That still does not encompass many other integrands with radicals,

√y
however. Suppose we have
where algebraic substitution comes in. Algebraic substitution is similar to integration by substitution, but will
involve more algebraic manipulation and is more complicated than plain integration by substitution.

EXAMPLES OF INTEGRATION BY ALGEBRAIC SUBSTITUTION

Evaluate:

3
X )
2√ − 4

3
No. STATEMENT REASON

X )
2√4 −
1 Given

3 _. + 4 Integrate by substitution: _ = √ − 4. We then have:


X 2_ )_
2_ _ . = − 4, = _ . + 4 and ) = 2_ )_
2

X 3_ . + 12 )_
3 Simplify the integrand

4 _ @ + 12_ + Y Integral Rules 2 and 3


@ Substitute back _ = √ − 4
−4 . + 12 −4 . +Y
5

√ −4 + 12√ − 4 + Y
@

√ − 4 to get √ − 4 + 12 then simplify into


.
(OPTIONAL) We can simplify further by factoring

√ −4 − 4 + 12 + Y − 4 + 12 , which simplifies to + 8
√ −4 +8 +Y

Evaluate:

@7 .
X +3)

We can go about this in two ways:

We can use algebraic substitution…

No. STATEMENT REASON

@7 .
X +3)
1 Given

125
Prepared by: Nathaniel M. Cabansay, BSCpE

X _ . − 3 _ _ )_ Integrate by substitution: Let _ = √ . + 3. We then


have _ . = . + 3 to get . = _ . − 3, then we
2

differentiate to get 2 ) = 2_ )_ or ) = _ )_ then


split the @ in the integrand into . ⋅ then rewrite the
integrand in terms of _.

X _ r − 3_ . )_
3 Simplify the integrand

_x
− _@ + Y
5
4 Integral rules 2 and 3

√ . +3
x
@ Substitute back _ = √ . +3
− M7 . + 3Q + Y
5

5
s
M7 8@Q

+3 5 √ +3
x

x @
. .
+3 − 5 , which simplifies to + 3 − 5 and
(OPTIONAL) We can factor out to get

.
− +Y . .
5 5 .
−2
√ +3
@
.
simplifies further to
.
+3 −5 +Y
5

√ +3
@
.
.
−2 +Y
5

…or we can integrate by parts

No. STATEMENT REASON

@7 .
X +3)
1 Given

1 @ 1 @
Integrate by parts: Let % = .
and )> = √ +3) .
+3 − X2 +3 )
.
. . . . .
3 3
This gives us )% = 2 ) and > = @ +3
2
s
.

1 @ 1 @ Integrate by substitution: Let ¸ = .


+ 3 and )¸ =
. .
+3 . − X ¸ . )¸
3 3 2 )
3

1 @ 2 x
. .
+3 . − ¸. + Y
3 15
4 Integral Rule 2

1 @ 2 x Substitute back ¸ = .
+3
. .
+3 . − .
+3 . +Y
3 15
5
s
8@

+3
@
√ . x
5 −2 +3 +Y 5 −2 + 3 , which simplifies to 5 . − 2 . −
(OPTIONAL): We can factor out to get
. . . .
15
6 = 3 − 6 then factor out the 3 and multiply into
.

√ +3
@
. s s

5 −2 −6 +Y
. . 8@ M7 8@Q
15 the
x
to get
x

√ +3
@
.
3 .
−6 +Y
15

√ +3
@
.
.
−2 +Y
5

126
Prepared by: Nathaniel M. Cabansay, BSCpE

1
Evaluate:

X )
√ √ −√
s .

We will need partial fractions for this:

1
No. STATEMENT REASON

X )
√ √ −√
1 Given
s .

6_ x Integrate by substitution: Let _ = √ , so we get _ @ =


é
X )_
_ _r − _ @
@
√ , _ r = √ . , and _ Ù = , then we differentiate to get
2
s

6_ )_ = )
x

1
6X )_
_ _−1
3 Simplify the integrand and use Integral Rule 3

1 ¿ â
= +
_ _−1 _−1 _
4 Decompose into partial fractions and substitute into the

1 = ¿_ + â _ − 1
integrand

1=¿
−1 = â
1 1 1
= −
_ _−1 _−1 _
1 1
6Xz − { )_
_−1 _
1 1
6 zX )_ − X )_{ + Y
_−1 _
5 Integral Rule 4

6 ln¥ √ − 1¥ − ln¥ √ ¥ + Y ∫ / )% = ln|%| + Y, then substitute back _ = √


é é é
6

6 ln¥ √ − 1¥ − ln| | + Y
é (OPTIONAL) ? log 3 = log 3 † , then simplify the
expression inside the logarithm

Evaluate:

@
X )
. +1 @

We can go about this in two ways:

We can use algebraic substitution…

No. STATEMENT REASON


@
X )
. +1 @
1 Given

1 _−1 Integrate by substitution: Let _ = .


+ 1, so we get _ −
X )_
2 _@ 1 = . and )_ = 2 ) or )_ = ) . We split @ =
2
.
.

127
Prepared by: Nathaniel M. Cabansay, BSCpE

1
X_ .
−_ @
)_
2
3 Simplify the integrand

1 _ .
~−_ + •+Y
2 2
4 Integral Rules 2 and 4

1 1 Substitute back _ = .
+1
− +Y
4 .+1 2 .+1
5
.

1−2 +1
(OPTIONAL) Combine into one fraction
.
+Y
4 .+1 .
−2 . − 1
+Y
4 .+1 .
2 .
+1
− +Y
4 . +1 .

…or we can use partial fractions

No. STATEMENT REASON


@
X )
. +1 @
1 Given

@
¿ +â Y +ç ê +W
= + . + .
. +1 @ .+1 +1 . +1 @
2 Decompose into partial fractions

= ¿ +â +1 + Y +ç +1 + ê +W
and substitute into the integrand
@ . . .

@
= ¿ +â r
+2 .
+1 + Y +ç .
+1 + ê +W
@
=¿ x
+â r
+ 2¿ + Y @
+ 2â + ç r
+ ¿+Y+ê + â+ç+W
0=¿
⎧0 = â

1 = 2¿ + Y
⎨0 = 2â + ç
⎪0 = ¿ + Y + ê
⎩0 = â + ç + W
¿ = 0, â = 0, Y = 1, ç = 0, ê = −1, W = 0
@
= −
. +1 @ . +1 . . +1 @

X − )
. +1 . . +1 @

1 1
X% .
)% − X % @
)%
2 2 substitution: Let % = . + 1 and
3 Use Integral Rule 4 and integrate by

)% = 2 ) or )% = )
.

128
Prepared by: Nathaniel M. Cabansay, BSCpE

1 1
.
+1 + .
+1 .
+Y
2 4 back % = . + 1
4 Use Integral Rule 2 and substitute

1 1
− +Y
4 . +1 . 2 .+1
(OPTIONAL) Combine into one

1−2 .+1
fraction

+Y
4 .+1 .
−2 . − 1
+Y
4 .+1 .
2 .
+1
− +Y
4 . +1 .

129
Prepared by: Nathaniel M. Cabansay, BSCpE

MODULE 17 IMPROPER INTEGRALS

IMPROPER INTEGRALS

Improper integrals are definite integrals where either:

1. At least one limit of integration is infinity.


2. The integrand is discontinuous

CONVERGENT AND DIVERGENT INTEGRALS


Now, these limits when applied to integrals may or may not result in a finite value in the end.

• Convergent integrals are improper integrals where the result is a finite number (i.e., the limit exists)
• Divergent integrals are improper integrals where the result is not a finite number (i.e., the limit results
in infinity or does not exist)

CASES WHERE AT LEAST ONE LIMIT OF INTEGRATION IS INFINITY


1. Upper limit is positive infinity

v 8Å v
If X ) exists for every ] > 0, then, X = lim X )
3 3 v→8Å 3

2. Lower limit is negative infinity

[ [ [
If X ) exists for every ] < \, then, X = lim X )
v Å v→ Å v

3. Both limits are positive and negative infinity

É 8Å 8Å É 8Å
If X ) and X ) are both convergent then, X =X ) +X )
Å É Å Å É

This will require both limits to exist. If even one of the integrals is divergent then the entire integral is divergent.

CASES WHERE INTEGRANDS ARE DISCONTINUOUS


1. Discontinuous at the upper limit.

[ v
If is continuous on the interval 0, \ and not continuous at = \, then X ) = lim X )
3 v→[ 3

This is provided the limit exists and is finite. Note that we need to use a left-hand limit here since the interval of
integration is on the left side of the upper limit.

130
Prepared by: Nathaniel M. Cabansay, BSCpE

2. Discontinuous at the lower limit.

[ [
If is continuous on the interval 0, \ and not continuous at = 0, then X ) = limd X )
3 v→3 v

This is provided the limit exists and is finite. Note that we need to use a right-hand limit here since the interval of
integration is on the right side of the upper limit.

3. Discontinuous somewhere in between the limits.

É [
If is discontinuous at = , where 0 < < \ and X ) and X ) are both convergent,
3 É
[ É [
then X ) =X ) +X )
3 3 É

This requires both integrals to be convergent in order for this integral to be convergent. If even one of them is
divergent, the entire integral is divergent.

4. Discontinuous at both limits.

É [
If is discontinuous at = 0 and = \, and if X ) and X ) are both convergent,
3 É
[ É [
then X ) =X ) +X )
3 3 É

Again, this requires both integrals to be convergent in order for this integral to be convergent. If even one of them
is divergent, the entire integral is divergent.

EXAMPLES OF IMPROPER INTEGRALS

Evaluate:


1
X )
q
. +1

No. STATEMENT REASON



1
X )
. +1
1 Given
q

1 v
1
X ) = lim X )
. +1 . +1
2 Rewrite the integral as a limit
v→8Å
q q

1
X ) = lim arctan v
. +1 q
3 Fundamental Theorem of Calculus
q v→8Å


1 arctan 0 = 0
X ) = lim arctan ]
. +1
4
v→8Å
q

131
Prepared by: Nathaniel M. Cabansay, BSCpE

¡

1 ¡ lim arctan ] =
X .+1
) = 2
2
v→8Å
5
q

The integral converges

Evaluate:


1
X )

No. STATEMENT REASON



1
X )
1 Given

v

1 1
X ) = lim X )
2 Rewrite the integral as a limit
v→8Å


1
X ) = lim ln| | v
3 Natural Logarithm Result
v→8Å


1 ln 1 = 0
X ) = lim ln|]|
4
v→8Å


1 lim ln|]| = +∞, and an infinite result means the
X .+1
) = +∞ v→8Å
5
integral diverges.
The integral diverges

Evaluate:


e
X )
Å e. +1

No. STATEMENT REASON



e
X )
e. + 1
1 Given
Å

e q
e 8Å
e
X ) =X ) +X )
e. +1 e. +1 e. +1
2 Rewrite the integral into two parts,
Å Å q
breaking at 0 as a convenient value.

e q
e v
e
X ) = lim X ) + lim X )
e. +1 e. +1 e. +1
3 Rewrite as limits
Å v→ Å v v→8Å q


e q
1 v
1 Integrate by substitution. Let % =
X ) = lim X )% + lim X . )%
e. +1 %. +1 v→8Å q % + 1 e so )% = e ) .
4
Å v→ Å v


e
X ) = lim arctan e q
+ lim arctan e v
e. + 1 v q
then substitute back % = e
5 Fundamental Theorem of Calculus,
v→ Å v→8Å
Å

e ¡ ¡ ¡
X ) = lim − arctan ev + lim arctan ev − arctan eq = arctan 1 =
e. + 1 4 4 4
6
Å v→ Å v→8Å

132
Prepared by: Nathaniel M. Cabansay, BSCpE


e ¡ ¡ ¡ lim arctan ev = arctan 0 = 0 and
X ) = −0+ −
e. +1 4 2 4
v→ Å
lim arctan ev =
£
7
Å
v→8Å .


e ¡
X ) =
e. + 1 2
8 Combine like terms
Å

Evaluate:

1
X )

s
q

No. STATEMENT REASON

1
X )

1 Rewrite as a power
s
q

1 1 The integrand is discontinuous at the lower limit = 0,


X ) = lim X s )
2

√ √
s
v→q
q
so rewrite as a limit approaching 0 from the right.
v

1
X ) = lim X @ )
3 Rewrite as a power


s
q v→q
v

.
1 @
X ) = lim ó ô
4 Fundamental Theorem of Calculus


s
v→q 2
3
q
v

1 3 .
X ) = lim | @}
√ 2
5 Rewrite as a reciprocal
s
q v→q v

1 3 . .
X ) = lim | z 1 @ − ] @ {}
√ 2
6 Fundamental Theorem of Calculus
s
q v→q

1 3
Xs ) =
7 Direct substitution

√ 2
q

The integral converges

Evaluate:

@
1
X .
)
q

No. STATEMENT REASON


@
1
X )
1 Given
.
q

@
1 1@
X ) = lim |− }
2 Rewrite as a limit, then use the Fundamental Theorem
. v→q
q v
of Calculus

133
Prepared by: Nathaniel M. Cabansay, BSCpE

@
1 1 1
X ) = lim | − }
. v→q ] 3
3 Fundamental Theorem of Calculus
q
@
1 1 1 lim v does not exist
X ) = lim | − }
v→q ] 3
v→q
4
.
q

@
1
X ) = +∞
5 Since the limit does not exist, the limit does not
.
q
converge.

The integral diverges

134
Prepared by: Nathaniel M. Cabansay, BSCpE

MODULE 18 NUMERIC INTEGRATION

APPROXIMATING DEFINITE INTEGRALS


While we’ve seen many functions that we can integrate, there are many more still that we cannot integrate. These
elementary functions have integrals that are known as non-elementary functions. When this happens, we instead do
various techniques of numerical integration to approximate definite integrals.

THE MIDPOINT RULE

equal width Δ = = 0, = 0+Δ , =


We’ve done this back in Module 9. Here, we will divide the interval into subintervals of
[ 3
q .

0 + 2Δ , … , = 0 + ½Δ , … =0+ −1 Δ , = 0 + Δ = \. We then use the


. The endpoints of each sub-interval are at:
º
midpoints of those sub-intervals to define the height of the rectangle from the x-axis to the
function, as seen in the figure on the right. Here, we approximate the curve with piecewise
constant functions.

We can easily find the area for each of these rectangles. For a general we get:

No. STATEMENT REASON


[
+ +
X ) ≈Δ z {+Δ z {+⋯
q .
2 2
1 Adding up the rectangles

+
3

+Δ z {
2
[
+ + Factor Δ from the right side
X ) ≈Δ | z {+ z {+⋯
q .
2 2
2

+
3

+ z {}
2

THE TRAPEZOIDAL RULE

trapezoids. The width of the trapezoid is still Δ =


This works similarly to the Midpoint Rule earlier, but instead of using rectangles, we use
[ 3
, but the height of the trapezoid,
however, is now the mean of evaluated at the endpoints of each sub-interval:
õ 8 õ !
.
. Here, we approximate the curve with piecewise linear functions.

This means that the area of each trapezoid is

+ \−0
¿=j kz {
º º
2

135
Prepared by: Nathaniel M. Cabansay, BSCpE

For a general , we get:

No. STATEMENT REASON


[
Δ Δ
X ) ≈ + + + +⋯
2 q
2 .
1 Adding up the trapezoids
3
Δ
+ +
2
[
Δ ö
X ) ≈ +2 +⋯+2 +
2 q .
2 Factor from the right side and
3


combine like terms

SIMPSON’S RULE

Again, the width Δ =


[ 3
, but has to be even this time. Here, we approximate the curve with piecewise
quadratic functions. Note that we have to partition into an even number of subintervals because we approximate
over two subintervals rather than just one. Over the first pair of subintervals, we approximate:

X ) ≈X ø ) ,ø =¿ .
+â +Y
÷ ÷

And ø is the quadratic function passing through q, q , , and ., . . Over the next two
sub-intervals, we approximate:

X
J
) ≈X ù
J
) ,ù =¿ .
+â +Y

And ù is the quadratic function passing through ., . , @, @ and r, r . We continue this


over successive pairs of sub-intervals.

To get the formula, we start by deriving a formula for the approximation over the first two sub-intervals. We
establish:

No. STATEMENT REASON

q =ø q =¿ .
q +â q +Y
=ø =¿ +â +Y
1 Establish the relationships of the
. polynomials

. =ø . =¿ .
. +â . +Y
2 . − q = 2Δ Establish the length of a sub-
interval

. + q =2
=
8 ÷
3 From the length of the sub-
.
intervals,

X ) ≈X ø )
4 Approximating the integral
÷ ÷

136
Prepared by: Nathaniel M. Cabansay, BSCpE

X ) ≈X ¿ .
+â +Y )
5 From the relationships established
÷ ÷
in statement 1

¿ â
X ) ≈| @
+ .
+Y }
3 2
6 Integral Rules 2 and 4
÷ ÷

¿ â
X ) ≈ @
− @
+ .
− .
+Y −
3 . q
2 . q . q
7 Fundamental Theorem of Calculus
÷

¿ â
X ) ≈ − .
+ + .
+ − +
3 . q . . q q
2 . q . q
8 Factor difference of two cubes and

+Y −
÷ difference of two squares
. q

− ÷
X ) ≈ 2¿ + + + 3â + + 6Y
. q . .
6 . . q q . q Ù
9 Factor
÷

Δ
X ) ≈ 2¿ .
+ + .
+ 3â + + 6Y
3 . . q q . q
10 From statement 2.
÷

Δ
X ) ≈ 2¿ .
+ 2¿ + 2¿ .
+ 3â + 3â + 6Y
3 . . q q . q
11 Multiply all terms
÷

Δ
X ) ≈ ¿ .
+â +Y + ¿ .
+â +Y
3 . . q q
12 Rearrange terms

+¿ +2 +¿ + 2â + + 4Y
÷
. .
. . q q . q

Δ
X ) ≈ + +¿ + .
+ 2â + + 4Y
3 . q . q . q
13 From statement 1
÷

Δ
X ) ≈ + +¿ 2 .
+ 2â 2 + 4Y
3 . q
14 From statement 3
÷

Δ Square 2 then multiply by ¿ and


X ) ≈ + + 4¿ .
+ 4â + 4Y
3 . q
multiply 2 by 2â
15
÷

Δ
X ) ≈ +4 +
3 q .
16 Factor 4 and then use the
÷
relationship from statement 1

Δ
X
J
) ≈ +4 +
3 . @ r
17 For the next pair of sub-intervals,
we get the same result

Δ
X
J
) ≈ +4 +2 +4 +
3 q . @ r
18 Combine the two pairs using this
÷
formula
É [ [

X ) +X ) =X )
3 É 3

Δ
X ) ≈ +4 +2 +4 +2
ä

3 q . @ r
19 The pattern continues as we add

+ ⋯+ 2 +4 +
÷
consecutive pairs of sub-intervals
.

137
Prepared by: Nathaniel M. Cabansay, BSCpE

SERIES FORMS OF APPROXIMATION RULES

MIDPOINT RULE

The midpoint rule can be written in sigma notation as:

+ \−0
¹j z {k z {
º º
2
º»

We could also use:

+
Δ ú¹ j z {kû
º º
2
º»

TRAPEZOIDAL RULE

The trapezoidal rule can be written in sigma notation as:

\−0
¹ º + º z {
º»

Alternately, we could use:

Δ
ú +2¹ + û
2 q
º»

SIMPSON’S RULE

Simpson’s Rule can be written in sigma notation as:

Δ
. .

ü + 4¹ +2¹ + ý
3 q . .
º» º»

EXAMPLES OF APPROXIMATING DEFINITE INTEGRALS

Evaluate:

e .
X )
√2¡
q

Use all three rules.

Set = 10 and approximate to four decimal places.

138
Prepared by: Nathaniel M. Cabansay, BSCpE

MIDPOINT RULE

No. STATEMENT REASON

e .
X )
1 Given

q √2¡

Midpoint Rule, with Δ = =


q.qx q. x q.yx q
e
1 e .. e . e .
X ) ≈ ó + +⋯+ ô
q q
2

q √2¡ 10 √2¡ √2¡ √2¡

e 1
.
X ) ≈ 3.414
3 Evaluate all the terms inside the

q √2¡ 10
brackets

e . q
X ) ≈ 0.3414
4 Multiply 3.414 by

q √2¡

TRAPEZOIDAL RULE

No. STATEMENT REASON

e .
X )
1 Given

q √2¡
Trapezoidal Rule, with Δ = =
q q. q.y q
e 1 e .
. e . e . e .
X ) ≈ ó +2 + ⋯+2 + ô
q
2

q √2¡ 20 √2¡ √2¡ √2¡ √2¡ q

e . 1
X ) ≈ 6.8229
3 Evaluate all the terms inside the

q √2¡ 20
brackets

e . .q
X ) ≈ 0.3411
4 Multiply 6.8229 by

q √2¡

SIMPSON’S RULE

No. STATEMENT REASON

e .
X )
1 Given

q √2¡
q q. q.. q.y
e 1 e .
. e . e . e . e .
X ) ≈ ó +4 +2 +⋯+ 4 + ô Δ = =
q
2 Simpson’s Rule, with

q √2¡ 30 √2¡ √2¡ √2¡ √2¡ √2¡ q q

e 1 .
X ) ≈ 10.2403
3 Evaluate all the terms

q √2¡ 30
inside the brackets

139
Prepared by: Nathaniel M. Cabansay, BSCpE

e . @q
X ) ≈ 0.3413
4 Multiply 10.2403 by

q √2¡

There is actually a fourth way of doing this. The given function is the normal distribution function, so we look at a
Z-table for the area between z=0 and z=1. The area is 0.3413.

Note that of the three methods we’ve shown, Simpson’s method is the most accurate.

140
Prepared by: Nathaniel M. Cabansay, BSCpE

CHAPTER 5 APPLICATIONS OF INTEGRALS

MODULE 19 AVERAGE FUNCTION VALUE

AVERAGE VALUE

One main application of definite integrals is finding the average value of a function over a specific interval 0, \ .
The average value of a function over the interval 0, \ is given by:

1 [
= X )
3=
\−0 3

We can derive this from the limit definition of the definite integral

No. STATEMENT REASON


\−0
Δ =
1 Length of a subinterval

2 ∗
+ ∗
. + ∗
@ + ⋯+ ∗
Choose points ∗ , .∗ , … , ∗ from the subintervals, and
compute the average of these values by dividing by
\−0
=
Δ
3 From statement 1, we get this for our


+ ∗
. + @ + ⋯+
∗ ∗

\−0
4 From statement 3, substitute into statement 2

Δ

+ ∗
. + ∗
@ + ⋯+ ∗
Δ
\−0
5 Simplify the complex fraction

1 ∗
+ ∗
+⋯+ ∗
Δ
\−0 . [ 3
6 Factor out

1
¹ ∗
Δ [ 3
\−0
7
º
Rewrite as a sum and factor out of the sum

º»

1
= lim ¹ ∗
Δ
→8Å \ − 0
8 The average function value becomes more accurate with
3= º
º»
more and more values to average, so get the limit as
approaches infinity

1
= lim ¹ ∗
Δ
\ − 0 →8Å
9 Limit Theorem 3
3= º
º»

P Í
L‚if = X L F CF
Í−‚ ‚
10 Definition of the definite integral

141
Prepared by: Nathaniel M. Cabansay, BSCpE

THE MEAN VALUE THEOREM FOR INTEGRALS

is continuous on the closed interval 0, \ , then there is a


number in 0, \ such that the function will get its average value. We can write this as:
The mean value theorem for integrals states that if

1 [
= X )
\−0 3

We can also write this as:

[
X ) = \−0
3

We can derive this from the Fundamental Theorem of Calculus and the Mean Value Theorem for Derivatives

No. STATEMENT REASON

W =X ] )]
1 First part of the Fundamental Theorem of Calculus
3

W \ −W 0
W =
\−0
2 Mean Value Theorem for Derivatives

W \ −W 0 =W \−0
[
From statement 1, we know that W =
X ) = \−0 substituting \ and 0 into statement 1, we get
3 , then

[ [
W \ =X ] )] = X )
3

1 [
= X )
3 3
\−0 3 3
W 0 =X ] )] = 0
∎ 3

(OPTIONAL) Divide both sides by \ − 0

ROOT MEAN SQUARE VALUE (RMS) VALUE OF A FUNCTION

This is simply the mean value theorem modified as:

1 [
þÌ = Ÿ X . )
\−0 3

This is commonly used in mathematics, physics, and engineering.

EXAMPLES OF FINDING THE AVERAGE FUNCTION VALUE

Determine the average value of = @


on the interval 0, 2

No. STATEMENT REASON

1 = @
, 0,2 Given

142
Prepared by: Nathaniel M. Cabansay, BSCpE

1 [
= X )
3=
\−0 3
2 Formula for average function value

1 . = @ , \ = 2, and 0 = 0. The function is


= X @
)
3=
2 q
3 Substitute
a polynomial, so it is continuous over all real numbers.

1 2r 0r
= ~ − •
3=
2 4 4
4 Fundamental Theorem of Calculus.

1
= 4
3=
2
5 Simplify

3= =2

] = csc . ] on the interval ,


£ @£
r r
Determine the average value of :

¡ 3¡
No. STATEMENT REASON

] = csc . ] , | , }
4 4
1 Given

1 [
= X ] )]
3=
\−0 3
2 Formula for average function value

] = − csc . ] , \ = , and 0 = . The


@£ @£ £
1 r
= X csc ] )]. r r
3¡ ¡
3 Substitute
3=

£
4 4 r
function is continuous over the given interval, so we can


1 r
proceed.

3= = ¡ X csc . ] )]
£
2 r

2 r
= X csc . ] )]
3=
¡ £
r

2 ¡ 3¡
= |cot M Q − cot z {}
3=
¡ 4 4
4 Fundamental Theorem of Calculus

2
= 1 − −1
3=
¡
5 Evaluate and simplify

2
3= = ⋅2
¡
4
=
3=
¡

= −3 .

on the interval 0,6


Determine the value(s) of that satisfy the Mean Value Theorem for Integrals for the function

No. STATEMENT REASON

1 = − 3 , 0,6 .
Given

1 [
= X )
\−0 3
2 Mean Value Theorem for Definite Integrals

143
Prepared by: Nathaniel M. Cabansay, BSCpE

1 Ù = − 3 .,
−3 .
= X −3 .
)
6 q \ = 6, and 0 = 0
3 Evaluate at , then substitute

1 @ Integrate by substitution: Let % = − 3 , so )% = ) ,


−3 .
= X %. )
6 @ % 0 = −3 and % 6 = 3
4

1 @ Notice that %. is an even function, so we can use the


−3 .
= X %. )
3 q
5
property:
3 3
X ) = 2X )
3 q

1 3@ 0@
−3 .
= ~ − •
3 3 3
5 Fundamental Theorem of Calculus

1
−3 .
= 9
3
6 Simplify the right-hand side

−3 .
=3
7 − 3 = ±√3 Get the square root of both sides, taking both the
positive and negative in consideration for the right-hand
side

= 3 + √3
= 3 − √3
8 Add 3 to both sides. Both values are in the interval.

for a sine wave > ] = sin M Q over the interval 0, .


.£v
Find the average value

2¡]
No. STATEMENT REASON

> ] = sin z { , |0, }


2
1 Given

1 [
= X )
\−0 3
2 Mean Value Theorem for Definite Integrals

2 2¡] = = sin M Q,
.£v
.
= X sin z { )]
3
\ = , and 0 = 0
Let , then substitute
q
.

Let % = , then )% = )], so )% = )]


.£v .£
.
= X sin % )% .£
¡
4

= − cos % . X sin % )% = − cos %


¡ q
5

Substitute back % =
.£v
2¡] .
= |− cos z {}
v
6
¡ q

= − cos ¡ + cos 0
¡
7 Fundamental Theorem of Calculus

144
Prepared by: Nathaniel M. Cabansay, BSCpE

= 1+1
¡
8 Evaluate and simplify

2
=
¡

for a sine wave > ] = sin M Q over the interval − ,


.£v
Find the RMS value

2¡]
No. STATEMENT REASON

> ] = sin z { , 0,2¡


1 Given

1 [
þÌ = Ÿ X . )
2 Formula for RMS
\−0 3

Let þÌ = = sin M Q,
.£v
1 2¡] .
=Ÿ X | sin z {} )] \ = , and 0 = −
3
2
and substitute

sin M Q
.£v
1 2¡]
=Ÿ X . sin. z { )]
4
2
Square

1 . 4¡] Use the power-reducing formula sin. = 1−


=Ÿ X |1 − cos z {} )]
.
cos 2
5
2 2

. 4¡] from the integrand. Notice that 1 and


=Ÿ X |1 − cos z {} )] .
6
2 cos M Q are both even functions, so we can use:
Factor out
r£v
q

3 3
X ) = 2X )
3 q

. sin 4¡
=Ÿ | − }
7 Fundamental Theorem of Calculus

2 4

.

8 Evaluate and simplify

.

2

=
√2

The above two formulas are useful when computing for the average and RMS values of a sinusoidal waveform,

= =
commonly encountered in electrical circuits (these are specifically the ones for a sinusoidal AC voltage, and
.
£ √.
current works similarly: and )

145
Prepared by: Nathaniel M. Cabansay, BSCpE

MODULE 20 AREA BETWEEN CURVES

AREA BETWEEN CURVES

À
We’ve used the definite integral to find the area under a curve (specifically the area between that curve and the x-

from 0 to \ under the curve of


axis). Suppose we have two curves and , with . We certainly know how to find the area
and under the curve of . Why don’t we use those to find the area
between curves?

No. STATEMENT REASON

, , À
À
1 Suppose we have two curves and , with

[ [
X ) ,X )
case) from 0 to \
2 Area between a curve and an axis (the x-axis in this
3 3
[ [

¿=X ) −X )
3 Since all the area between and the x-
3 3
axis gets covered by the area of , so to get the area
between them, we subtract. If ever crosses the x-
axis, we are still covered as the area will be considered
negative, and adding that area is the same as subtracting
a negative.
[ [ [ [
¿=X − ) X ) −X ) =X − )
4
3 3 3 3

We can similarly derive an area between two curves from to ) between ? and ? , using the y-axis as the
basis this time

No. STATEMENT REASON


? , ? , ? À ? ? and ? , with
? À ?
1 Suppose we have two curves

X ? )? , X ? )?
case) from to )
2 Area between a curve and an axis (the y-axis in this
É É

Since ? ≤ ? all the area between ? and the y-


¿=X ? )? − X ? )?
axis gets covered by the area of ? , so to get the area
3
É É
between them, we subtract. If ? ever crosses the y-
axis, we are still covered as the area will be considered
negative, and adding that area is the same as subtracting
a negative.
[ [ [
¿=X ? − ? )? X ) −X ) =X − )
4
É 3 3 3

With that being said, however, we usually have to determine the limits of integration first. We are usually given
only the curves, and then we find the limits of integration based on where the curves intersect.

146
Prepared by: Nathaniel M. Cabansay, BSCpE

EXAMPLES OF AREAS BETWEEN CURVES

Determine the area of the region bounded by ? = .


and ? = √

No. STATEMENT REASON

1 ?= .
,? = √ Given

Graphs of both functions. The orange one is ? = √ ,


while the red one is ? = . . From the graphs, the
2

functions intersect at = 0 and = 1, so these will be


our limits of integration

? = √ was the “upper” function while ? = .


¿ = X l√ − .
m)
3 was the
q lower function

2 √ @
¿ = ~j − k•
3 3 q
4 Get the indefinite integral of the function

2 1 √1 1@ 2 0 √0 0@
¿ = ~j − k−j − k•
3 3 3 3
5 Fundamental Theorem of Calculus Part II

2 1 .
¿= −
3 3
@ @
6 Simplify the right-hand side and subtract from

1
¿=
3

Determine the area of the region bounded by ? = .


− 1 and ? = √1 − .

No. STATEMENT REASON

1 ?= .
− 1, ? = 71 − . Given

Graphs of both functions. The orange one is ? = . −


1 while the red one is ? = √1 − . . From the graphs,
2

the functions intersect at = −1 and = 1, so these


will be our limits of integration. Note that there is
some negative area here, or area below the x-axis.
We will be subtracting the functions so subtracting
the negative area is the same as adding positive area.

147
Prepared by: Nathaniel M. Cabansay, BSCpE

? = √1 − . is the “upper” function while ? = .


−1
¿=X 71 − . − .
−1 )
3
is the “lower” function.

¿ = X 71 − . ) −X .
) +X 1)
4 Integral Rule 4

¿ = 2 ~X 71 − . ) −X .
) +X 1) •
5 All of these functions are even functions, so we can
q q q
use the property:
3 3
X ) = 2X )
3 q

1 ¡ ∫q √1 − . ) is the area of a circular sector with a


¿ = 2~ 1 .
M Q−X .
) +X 1) •
2 2 £
6
q q
.
formula ¿ = `,
radius of 1 and central angle of . We then use the
.
.

1 ¡ @
¿ = 2ú 1 .
M Q−~ • + qû
2 2 3 q
7 Integral Rule 2

1 ¡ 1 0
¿ = 2~ 1 .
M Q−| − } + 1−0 •
2 2 3 3
8 Fundamental Theorem of Calculus

¡ 1
¿ = 2 | − + 1}
4 3
9 Simplify and combine like terms

¡ 2
¿ = 2| + }
4 3
3¡ + 8
¿ = 2| }
12
3¡ + 8
¿=
6

Determine the area of the region bounded by: ? = , ? = and =3

1 1
No. STATEMENT REASON

?= ,? = .
, =3
1 Given. We already have one of the limits

Graphs of all three functions. The line in black = 3 is


one of the limits of integration. The red graph is ? =
2

and the orange graph is ? =

@
1 1 ? = is the “upper” function and ? =
¿ = X | − .} )
3 is the “lower”
function

148
Prepared by: Nathaniel M. Cabansay, BSCpE

1@ Integral Rules 2 and 4 and ∫ )% = ln|%| + Y


¿ = |ln| | + } /
4

1 1
¿ = | ln|3| − ln|1| − z − {}
3 1
5 Fundamental Theorem of Calculus

1 log 3 1 = 0 , then simplify the right-hand side


¿ = ln|3| − + 1
3
6

2
¿ = ln|3| −
3

Determine the area of the region bounded by: = 3? . − 9, = 0, ? = 0, and ? = 1

No. STATEMENT REASON

1 = 3? . − 9, = 0, ? = 0, ? = 1 Given

2 Graphs of the functions. The area we need is the area in


orange.

¿ = X 0 − 3? . − 9 )?
of ?, so we have = 0 as the “right-hand” function, =
3 It will be easier to find the area in terms of the functions
q
3? . − 9 as the “left-hand” function, and the limits of
integration are given by ? = 0 and ? = 1

¿ = X 9 − 3? . )?
4 Simplify the integrand
q

5 ¿ = 9? − ? @ q Integral Rules 2 and 4


¿= 9 1 − 1 @
− 90 − 0 @

¿=9−1
6 Fundamental Theorem of Calculus

¿=8

Determine the area bounded by the curves ? = @


+2 .
− 5 − 6 and the x-axis (at ? = 0)

No. STATEMENT REASON

1 ?= @
+2 .
− 5 − 6, ? = 0 Given

149
Prepared by: Nathaniel M. Cabansay, BSCpE

2 Graph of the function. From the

zeros of ? = @ + 2 . − 5 − 6 are
Rational Root Theorem, the possible

±1, ±2, ±3, and ±6. By testing each

? = @ + 2 . − 5 − 6 is −1,
value, we find one of the zeroes of

because ? = −1 @ + 2 −1 . −
5 −1 − 6 = −1 + 2 + 5 − 6 = 0.
We then divide by + 1 to get . +
− 6, which we can factor into
+3 − 2 , giving us the other
two zeros are = −3 and = 2.

.
¿=X @
+2 .
−5 −6 −0) +X 0− @
+2 .
−5 −6 )
= −3 to = −1, @ + 2 . − 5 −
3 We have two areas to consider: From
@

6 À 0, and from = −1 to = 2,
@
+ 2 . − 5 − 6 ≤ 0.
.
¿=X @ +2 . −5 −6 ) −X @ +2 . −5 −6 )
4 Simplify the integrands
@

2 @ 5 . 2 @ 5 .
r r .
¿=~ + − −6 • −~ + − −6 •
4 3 2 4 3 2
5 Integral Rules 2 and 4
@

1 2 5 81 45 16 1 5
¿ = | − − +6− + 18 + − 18} − |4 + − 10 − 12 − + − 6}
4 3 2 4 2 3 4 2
6 Fundamental Theorem of Calculus

2 1 5 Simplify the right-hand side. −


×
=
¿ = |6 − } − |4 − 10 − 12 − + }
3 4 2 r r
7
−20 + M . − .Q = −20 + 20 = 0 and
rx x
16 9
¿ = | } − |−18 + }
+ −6=0
Ù .
3 4
@ @
16 63
¿= − z− {
3 4
16 63
¿= +
3 4
253
¿=
12

Determine the area bounded by ? = 4 − , ? = 0, and =0


s
.

No. STATEMENT REASON


@
1 ? = 4− . .
,? = 0, =0 Given

150
Prepared by: Nathaniel M. Cabansay, BSCpE

graph that the upper limit of integration is at = 2


2 Graph of the function. We can determine from the

. @
¿=X 4− . .
)
3 Area formula
q

= 2 sin `. This
£
. @
¿ = X 4 − 4 sin. ` 2 cos ` )`
gives us ) = 2 cos ` and ` = arcsin M Q. We can then
.
4 Use the trigonometric substitution:

q .
substitute the limits ` 0 = arcsin M Q = 0 and ` 2 =
q
.
arcsin M Q = arcsin 1 =
. £
. .

Use the Pythagorean Identity sin. ` + cos . ` = 1


£
. @
¿ = X 4 cos . ` . 2 cos ` )`
5
q
£
Evaluate 4 cos . `
s
.
¿ = X 8 cos ` 2 cos ` )`@
6

q
£
.
¿ = 16 X cos r ` )`
7 Simplify the integrand and use Integral Rule 3

¡ 0−1 ‼ 4−1 ‼
¿ = 16 M Q ~ •
2 0+4 ‼ so use Wallis’ formula with Ë = 0 and = 4 . Both Ë
8 The conditions for Wallis’ formula to be used are met,

and are even, so ˜ = .


£
.

3 1 Definition of the double factorial, and −1 ‼ = 1


¿ = 8¡ ~ •
4 2
9

10 ¿ = 3¡ Simplify the right-hand side

DERIVING AREA FORMULAS

Suppose we have to get areas of certain shapes and we do not know their formulas. Here’s where they’re from.

RECTANGLE

The functions we require here are ? = ℎ, where ℎ is the height of the rectangle, and ? = 0. We then define the
limits of our integration at = 0 and = ¸, where ¸ is the width of the rectangle

151
Prepared by: Nathaniel M. Cabansay, BSCpE

No. STATEMENT REASON

1 ? = ℎ, =¸ Given

Since height cannot be negative, ℎ > 0


¿=X ℎ)
2
q

¿=ℎ ¸−0 Since ℎ is a constant:


=
3
[
X ) = \−0
∎ 3

CIRCLE

The functions we require here are ? = √ . − . and ? = −√ . − .. To find the limits of integration, we set ? =
0 and solve for

No. STATEMENT REASON

1 ?=7 . − ., ? = −7 . − . Given

0=7 .− .

0= .− .
2 Solve for to determine the limits of integration

.
= .

¿=X 7 . − . − M−7 . − .Q )
3 Area between curves

¿ = 2X 7 . − . )
4 Simplify the integrand and use Integral Rule 3

= sin `,
£

¿ = 2X 7
.
. − . sin. ` cos ` )`
) = cos ` )`, and ` = arcsin M Q. We have ` =
5 Integrate by trigonometric substitution: Let
£
.
arcsin M Q = and ` − = arcsin M Q = −
£ £
. .

Use the identity sin. ` + cos . ` = 1 and simplify the


£

¿ = 2X 7
.
. − . sin. ` cos ` )`
6
£
.
integrand

¿ = 2X 7
.
. cos . ` cos ` )`
£
.
£
.
¿ = 2X .
cos . ` )`
£
.

cos ` and (a constant) are both even functions, so


£
.
¿=4 .
X cos . ` )`
7

q
their product is even, and we can use the property:
3 3
X ) = 2X )
3 q

152
Prepared by: Nathaniel M. Cabansay, BSCpE

¡ 0−1 ‼ 2−1 ‼
¿=4 .
M Q
2 2+0 ‼ so we use Wallis’ formula with Ë = 0 and = 2. Since
8 The conditions for Wallis’ formula to be used are met,

both are even, ˜ = .


£

¡ 1 Definition of the double factorial −1 ‼ = 1 and 1 ‼ =


¿=4 .
M Q
2 2 1
9

=’ G


10 Simplify the right-hand side

ELLIPSE

We will require the standard form of the horizontal ellipse centered at 0,0 or + = 1, which we rewrite into

3 [

a function of . ? = œ\ . − To get the full ellipse, we also include its negative: ? = −œ\ . −
[ .. [ .
3 3

No. STATEMENT REASON

\. \.
? = Ÿ\ . − ., ? = −Ÿ \ . − .
1 Given

0. 0.

\.
0 = Ÿ\ . − .
2 Solve for to determine the limits of integration

0.
\. .
0 = \. −
0.
0 = 0. \ . − \ . .

0 = 0. − .
.
= 0.
= ±0

3
\. \.
¿ = X óŸ \ . − . − ª−Ÿ\ . − . «ô )
3 Area between curves

3 0. 0.

3
.
¿ = 2\ X Ÿ1 − )
4 Simplify the integrand and use Integral Rule 3

0.
3

=
£
.

¿ = 20\ X 71 − sin. ` cos ` )` 0 sin `, ) = 0 cos ` )`, and ` = arcsin M Q. We


5 Integrate by trigonometric substitution: Let

3
have ` 0 = arcsin M Q = and ` −0 =
£ 3 £
. 3 .
arcsin M Q = −
3 £
3 .

Use the identity sin. ` + cos . ` = 1 and simplify


£
.

¿ = 20\ X 71 − sin. ` cos ` )`


6
the integrand
£
.

153
Prepared by: Nathaniel M. Cabansay, BSCpE

£
.

¿ = 20\ X 7cos . ` cos ` )`


£
.
£
.

¿ = 20\ X cos. ` )`
£
.

cos ` and are both even functions, so their


£
.

¿ = 40\ X cos ` )`
7
. product is even, and we can use the property:
3 3
q
X ) = 2X )
3 q

¡ 0−1 ‼ 2−1 ‼
¿ = 40\ M Q
2 2+0 ‼ met, so we use Wallis’ formula with Ë = 0 and =
8 The conditions for Wallis’ formula to be used are

2. Since both are even, ˜ = .


£

¡ 1 Definition of the double factorial −1 ‼ = 1 and


¿ = 40\ M Q
2 2 1 ‼=1
9

= ’‚Í

10 Simplify the right-hand side

The formula for the area of an ellipse will work for any orientation (horizontal or vertical).

RING/ANNULUS

We are basically just subtracting one circle from a larger one. This will prove useful for the next module.

with a larger radius þ. =’ −


From the formula of a circle, we simply subtract the area of a circle with radius from the area of a larger circle
G G

CIRCULAR SECTOR

We basically multiply ¿ = ¡ .
(since a sector with central angle ` gives us the area of
.£ .£
by of the circle) to
=G à
P G
determine the area of a circular sector. This results in

154
Prepared by: Nathaniel M. Cabansay, BSCpE

MODULE 21 MULTIPLE INTEGRALS

DEALING WITH MULTIPLE VARIABLES

Sometimes, we need to deal with functions of multiple variables, such as getting volumes. This is basically just
multiple definite integrals chained together. Here, we’ll see multiple cases of integrating multi-variable functions.

The double integral is as follows:

[
X X , ? ) )?
É 3

[
X X , ? )? )
É 3

Note that the order in which the integrals are done will matter. To summarize the order, it is “inside out”. This
means we evaluate the innermost integral first before we move on to the outer ones. In the above:

For ∫É ∫3 , ? ) )?, we integrate with respect to first, holding all other variables constant, and then ?. For
[

∫É ∫3 , ? )? ) , however, we integrate with respect to ? first, holding all other variables constant, and then .
[

In both cases, the limits of the integrals to be done first will be from 0 to \, then the limits of the integrals to be
done next will be from to ).

A triple integral is similar, only three integration signs and three variables are used.

EXAMPLES OF MULTIPLE INTEGRALS


Evaluate the double integral:

r
X X @
) )?
q q

No. STATEMENT REASON


r
X X @
) )?
1 Given
q q

r r
X ~ • )?
4
2 Integrate with respect to first.
q q
r
1
X )?
4
3 Fundamental Theorem of Calculus
q

1 r Integrate with respect to ?


| ?}
4 q
4

1
4 = 1
4
5 Fundamental Theorem of calculus

155
Prepared by: Nathaniel M. Cabansay, BSCpE

Evaluate the double integral:

r †
X X @
? ) )?
. q

No. STATEMENT REASON


r †
X X @
? ) )?
1 Given
. q

r r †
X ~ ?• )?
4
2 Integrate with respect to first.
. q
r
?r
X ? )?
4
3 Fundamental Theorem of Calculus
.
r
?x
X )?
. 4

1 Ù r Integrate with respect to ?


| ? }
24
4
.

1 1
4 Ù− 2 Ù
24 24
5 Fundamental Theorem of Calculus

4096 64

24 24
4032
= 168
24
Evaluate the double integral:

x –—5
1
X X )? )`
@ q 71 − ? .

No. STATEMENT REASON


x –—5
1
X X )? )`
71 − ?.
1 Given
@ q

x
Integrate with respect to ? first
X arcsin ? –—5
q )`
2
@
x
X arcsin sin ` )`
3 Fundamental Theorem of Calculus
@
x
X ` )`
4 Inverse functions
@

`.
x
Integrate with respect to `
~ •
2 @
5

5. 3.
~ − •
2 2
6 Fundamental Theorem of Calculus

156
Prepared by: Nathaniel M. Cabansay, BSCpE

25 − 9
| }
2
16
| }= 8
2
Evaluate the triple integral:
£
. –—5 •‡–
r
X X X )_ ) )`
q q q

No. STATEMENT REASON


£
. –—5 •‡–
r
X X X )_ ) )`
1 Given

q q q

Integrate with respect to _ first


£
. –—5
r
X X _ •‡–
) )`
2
q
q q
£
. –—5
r
X X .
cos ` ) )`
3 Fundamental Theorem of Calculus
q q
£ . –—5
r @
X ~ cos `• )`
3
4 Integrate with respect to
q q
£
r8
X sin@ ` cos ` )`
3
4 Fundamental Theorem of Calculus
q

Integrate with respect to ` by substitution: Let % =


8 . @
√.

X % )% sin ` and )% = cos ` )`. Substitute the limits as well.


5

3 q
% M Q = sin M Q = and % 0 = sin 0 = 0
£ £ √.
r r .

√.
8 %r .
~ •
6 Integral Rule 2

3 4 q
r
⎡j√2k
8⎢ 2
7 Fundamental Theorem of Calculus


3⎢ 4 ⎥
⎢ ⎥
⎣ ⎦
2 √2
r

j k
3 2
2 1 .
z {
3 2
2 1 1
z {=
3 4 6

157
Prepared by: Nathaniel M. Cabansay, BSCpE

MODULE 22 SOLIDS OF REVOLUTION

VOLUMES BY REVOLVING PART 1: DISKS

Similar to the use of side-by-side rectangles to get areas under and between curves, we use stacked disks, washers,
cells, or other shapes to get volumes. A solid of revolution is obtained by rotating a curve about a given axis, such
as the x-axis or y-axis, or any horizontal or vertical axis.

No. STATEMENT REASON

1 , 0, \ Suppose we have a curve and an interval 0, \


\−0
Δ =
2 If we rotate the curve about an axis, we get a 3D figure

width Δ .
which we can decompose into subintervals or slices of

3 º =¿ ∗
º Δ We then choose a point from each subinterval º∗ . Since

¿ º∗ .We then multiply by Δ to get the volume of a


we use disks for each subinterval, the area of each disk is

disk.

≈ ¹¿ ∗
Δ
4 We then approximate the volume of the region by
º
º»
adding enough disks

= lim ¹ ¿ ∗
Δ
5 We get closer and closer to the exact volume as we
º
→Å
º»
increase the number of disks

[
=X ¿ )
work the same way for a function of ?
6 This limit is the definition of a definite integral. This will
3

= X ¿ ? )?
É


[
=X ¡ .
)
7 Since we will usually be using circles, with the value of
3
¿ =¡ .
or ¿ ? = ¡ ? .
the function being the radius of each circle, we get

=X ¡ ? .
)?
É


Just like with areas between curves, sometimes we have to determine the limits of integration from the given
functions.

EXAMPLES OF SOLIDS OF REVOLUTION: DISK METHOD

Find the volume of the solid obtained by revolving the area bounded by ? = .
− 9 and the x-axis about ? = 0
using the disk method.

158
Prepared by: Nathaniel M. Cabansay, BSCpE

No. STATEMENT REASON

1 ?= .
− 9, ? = 0 Given

determine the limits of integration to be −3 and 3.


2 Graph of the functions. From the graph, we can

Also, from −3 to 3, . − 9 ≤ 0

¿ =¡ .

since the area of a circle is ¡ .


3 The radius of each disk is the value of the function,

@
9 − . is − .
− 9 and from the volume of a solid
=X ¡ 9− . .
)
4
@
of revolution
@
= ¡ X 81 − 18 .
+ r
)
5 Integral Rule 3, then expand the binomial
@
@ @ @
= ¡ jX 81 ) − X 18 .
) +X r
) k
6 Integral Rule 4
@ @ @
@ @ @
= 2¡ jX 81 ) − X 18 .
) +X r
) k
6 All three are even functions, so we can use:
3 3
X ) = 2X )
q q q

3 q

x @
= 2¡ ~81 − 6 @
+ •
5
7 Integral Rule 2

3 x
= 2¡ ~81 3 − 6 3 @
+ •
5
8 Fundamental Theorem of Calculus

243
= 2¡ |243 − 162 + }
5
9 Simplify the right-hand side

243
= 2¡ |81 + }
5
405 + 243
= 2¡ | }
5
648
= 2¡ | }
5
1296¡
=
5

159
Prepared by: Nathaniel M. Cabansay, BSCpE

Find the volume of the solid obtained by revolving the region bounded by = ? . − 4 and = 0 around the y-axis

No. STATEMENT REASON

1 = ? . − 4, =5 Given

2 Graphs of the functions. From the intersection

integration are at ? = −2 and ? = 2. From


of the graphs, we can determine the limits of

? = −2 to ? = 2, ≤ 0

¿ =¡ .

function, since the area of a circle is ¡ .


3 The radius of each disk is the value of the

.
9 − . is − . − 9 and from the volume of a
=X ¡ 4− . .
)
4
.
solid of revolution
.
= ¡ X 16 − 8 .
+ r
)
5 Integral Rule 3, then expand the binomial
.
. . .
= ¡ ~X 16 ) − X 8 .
) +X r
) •
6 Integral Rule 4
. . .
. . .
= 2¡ ~X 16 ) − X 8 .
) +X r
) •
7 All three are even functions, so we can use:
3 3
X ) = 2X )
q q q

3 q

8 x .
= 2¡ ~16 − @
+ •
3 5
8 Integral Rule 2

64 32
= 2¡ |32 − + }
3 5
9 Fundamental Theorem of Calculus

32 32
= 2¡ | + }
3 5
160 + 96
= 2¡ | }
15
256
= 2¡ | }
15
512¡
=
15
Find the volume of the solid obtained by revolving the region bounded by = ? . − 4 and = 5 around the x-axis

No. STATEMENT REASON

1 = ? . − 4, =5 Given

160
Prepared by: Nathaniel M. Cabansay, BSCpE

Graphs of the functions. Rewrite = ? . − 4 as a


function of x first, so ? = ±√ + 4 . We can then
2

determine the limits are at = −4 and = 5

3 ¿=¡ − .
We actually have two curves to consider here, so we
use the area between curves for this.
x
Those two curves are ? = √ + 4 and ? = −√ + 4
= X ¡l√ + 4 − −√ + 4 m )
4 .

r
x
= 4¡ X +4)
5 Simplify the integrand and use Integral Rule 3
r
x x
= 4¡ jX ) +X 4) k
6 Integral Rule 4
r r

. x
= 4¡ ~ +4 •
2
7 Integral Rule 2

25
= 4¡ |z + 20{ − 8 − 16 }
2
8 Fundamental Theorem of Calculus, then simplify the

65
= 4¡ z + 8{
right-hand side.

2
81
= 4¡ z {
2
= 162¡

Find the volume of the solid obtained by revolving ? = 2 + 1, = 4, and ? = 3 about the line ? = 3

No. STATEMENT REASON

1 ? = 2 + 1, = 4, ? = 3 Given

= 1 and = 4.
For = 1 to = 4, À 3.
2 Graphs of the functions. The limits are

3 ¿=¡ − .
Area of a disk
r
The functions are ? = 2 + 1 and ? = 3
=X ¡ 2 +1 −3 .
)
4

161
Prepared by: Nathaniel M. Cabansay, BSCpE

r
= ¡X 2 −2 .
)
5 Simplify the integrand

r
= ¡X 2 −1 )
.

r
= 4¡ X −1 .
)

@
Integrate by substitution: Let % = − 1 and )% = ) ,
= 4¡ X % . )% then % 1 = 1 − 1 = 0 and % 4 = 4 − 1 = 3
6

%@
@
= 4¡ ~ •
3 q
7 Integral Rule 2

3@
= 4¡ j k
3
8 Fundamental Rule of Calculus

27
= 4¡ z {
3
9 Simplify the right-hand side

= 4¡ 9
= 36¡

VOLUMES BY REVOLVING PART 2: WASH ERS

There are also other solids we can get by adding up washers or disks with holes. For the washer method, we
revolve an area between two curves. This simply changes the formulas to:

[
=X ¡ .
− .
)
3

=X ¡ ? .
− .
)?
É

EXAMPLES OF SOLIDS OF REVOLUTION: WASHERS METHOD


Find the volume of the solid obtained by revolving the region bounded by ? = e , ? = , = 0 and = 1, about
the x-axis

No. STATEMENT REASON

1 ? = e ,? = , = 0, =1 Given

162
Prepared by: Nathaniel M. Cabansay, BSCpE

= 0 and = 1. For the given interval, e À


2 Graphs of the functions. We are already given the limits

3 ¿=¡ .
− .
Area of a washer

=X ¡ e .
− .
)
4 Volume of the solid
q

= ¡ X e. − .
)
5 Simplify the integrand and use Integral Rules 3 and 4
q

= ¡ jX e. ) − X .
) k
q q

e. @
= ¡~ − •
2 3 q
6 Integral Rule 2 and

X e/ )% = e/ + Y

e. 1 1
= ¡ ~j − k − z − 0{•
2 3 2
7 Fundamental Theorem of Calculus

e. 2 + 3
= ¡ ~j − k•
2 6
8 Simplify the right-hand side

e.
5
= ¡~ − •
2 6
3e. − 5
= ¡~ •
6
3e. − 5 ¡
=
6
Find the volume of the solid obtained by revolving the regions bounded by ? = and ? = @
about the x-axis

No. STATEMENT REASON

1 ? = ,? = @
Given

163
Prepared by: Nathaniel M. Cabansay, BSCpE

Graphs of the functions. From = −1 to =


0, @ À and = 0 to = 1, @ ≤
2

3 ¿=¡ .
− .
Area of a washer
q
=X ¡ @ .
− .
) +X ¡ .
− @ .
)
4 Split the solid revolved into two regions
q
q
= ¡ jX Ù
− .
) +X .
− Ù
) k
5 Simplify the integrands
q

Ö @ q @ Ö
= ¡ Ç~ − • +~ − • È
7 3 3 7
6 Integral Rule 2

1 1 1 1
= ¡ z| − } + | − }{
3 7 3 7
7 Fundamental Theorem of Calculus

1 1
= 2¡ z| − }{
3 7
8 Simplify the right-hand side

7−3
= 2¡ z {
21
4
= 2¡ z {
21

=
21
Find the volume of the solid obtained by revolving the region bounded by = ? . − 4 and = 5 about the y-axis

No. STATEMENT REASON

1 = ? . − 4, =5 Given

= ? . − 4 and = 5 around the y-


2 Graphs of the functions. If we revolve

axis, we get a shape similar to the one


in the image below.
From the graphs, it appears we must

representing ? = −3 to ? = −2, one


split the solid into three regions: one

representing ? = −2 to ? = 2, and one


representing ? = 2 to ? = 3.

164
Prepared by: Nathaniel M. Cabansay, BSCpE

For ? = −3 to ? = −2 and ? = 2 to
? = 3, 0 ≤ ≤ 5, so the area between
= −5 and = 5 is not completely

For ? = −2 to ? = 2, the area between


covered.

both = ? . − 4 and = 4 − ? . is
completely bounded by = −5 and
= 5, as the areas overlap so we can
simply revolve the line = 5 around
the y-axis

3 ¿=¡ .
− .
The area of each washer
. .
= X ¡ 25 − ? . − 4 .
)? + X ¡ 5 .
)?
4 Volume of the solid.
@ .
@
+ X ¡ 25 − ? − 4 . .
)?
.
. .
= ¡ jX 25 − ? r + 8? . − 16 )? + X 25 )?
5 Simplify the integrands by using Integral
@ .
@
Rule 3 and combining like terms

+ X 25 − ? + 8? − 16 )? k r .
.
. .
= ¡ jX 9 + 8? . − ? r )? + X 25 )?
@ .
@
+ X 9 + 8? . − ? r )?k
.
. . @
6 = ¡ jX 9 + 8?. − ? r )? + 2 X 25 )? + X 9 + 8?. − ?r )?k A constant is an even function so we
@ q . can use the property:
3 3
X ) = 2X )
3 q

8? @ ? x 8? @ ? x
. @
= ¡ Ç~9? + − • + 2 25? .
+ ~81 + − • È
3 5 3 5 .
7 Integral Rule 2
q
@

8 32 8 243
= ¡ jz|9 −2 + −8 + } − |9 −3 + −27 + }{ + 2 50
3 5 3 5
8 Fundamental Theorem of Calculus
8 243 8 32
+ z|9 3 + 27 − } − |9 2 + 8 − }{k
3 5 3 5

165
Prepared by: Nathaniel M. Cabansay, BSCpE

64 32 243
= ¡ jz|−18 − + } − |−27 − 72 + }{ + 100
3 5 5
10 Simplify the right-hand side
243 64 32
+ z|27 + 72 − } − |18 + − }{k
5 3 5
96 − 320 − 270 243 − 495
= ¡ Çz| }−| }{ + 100
15 5
495 − 243 270 + 320 − 96
+ z| }−| }{È
15 15
494 252 252 494
= ¡ z− + + 100 + − {
15 5 5 15
756 494
= ¡ z2 z − { + 100{
15 15
262
= ¡ z2 z { + 100{
15
524
= ¡z + 100{
15
2024
= ¡z {
15
2024¡
=
15

VOLUMES BY REVOLVING PART 3: SH ELLS

In the previous two parts of this module, we’ve been using disks or washers as cross-sections to find volumes of
solids of revolution. This does not always need to be the case, however. We can also use any shape for the cross
sections as long as it can be expanded or contracted to cover the solid completely.

For the next examples, the area will now be the surface area of a cylinder, which is ¿ = 2¡ ℎ, where is a linear
expression in x or y

EXAMPLES OF SOLIDS OF REVOLUTION: SHELLS METHOD

Find the volume of the solid obtained by revolving the region bounded by ? = and ? = @

where À 0
about the y-axis,

No. STATEMENT REASON

1 ? = ,? = @
Given

2 Graph of the functions. Since À 0, we set one of the

which is at 1,1 , so the other limit of integration is at


limits at 0, then find the intersection point to its right,

=1

3 ¿ = 2¡ ℎ The surface area of a cylinder

= X 2¡ − @
)
cylinder is − @
4 The radius of a cylinder is and the height of the
q

166
Prepared by: Nathaniel M. Cabansay, BSCpE

= 2¡ X .
− r
)
5 Simplify the integrand and use Integral Rule 3
q

@ x
= 2¡ ~ − •
3 5
6 Integral Rule 2

1 1
= 2¡ | − }
3 5
7 Fundamental Theorem of Calculus

5−3
= 2¡ | }
15
2
= 2¡ | }
15

=
15

= ?+2 .

? = 0, and = 0 around the x-axis


Find the volume of the solid obtained by revolving the region bounded by the parabola and the lines

No. STATEMENT REASON

1 = ? + 2 . , ? = 0, =0 Given

points, we can determine the limits of ? are at ? = −2


2 Graphs of the functions. From the intersections of the

and ? = 0

3 ¿ = 2¡ ℎ The surface area of a cylinder


q
The radius of a shell is −? (since the area is below the
= X 2¡ −? ? + 2 .
)?
x-axis) and its height is ? + 2 .
4
.
q
= −2¡ X ? @ + 4? . + 4? )?
5 Simplify the integrand and use Integral Rule 3
.

? r 4? @
q
= −2¡ ~ + + 2? . •
4 3
6 Integral Rules 2 and 4

−2 r
4 −2 @
= −2¡ ~− j + + 2 −2 . k•
4 3
7 Fundamental Theorem of Calculus

32
= −2¡ |− z4 − + 8{}
3
8 Simplify the right-hand side

32
= −2¡ |− z12 − {}
3
4
= −2¡ |− z {}
3

=
3

167
Prepared by: Nathaniel M. Cabansay, BSCpE

VOLUMES BY REVOLVING PART 4: OTHER CROSS-SECTIONAL AREAS

In the first two modules, we have been using circles or annuli (plural of annulus, meaning ring, which was the
washer method we did) as cross sections for these solids. Here, we can also have other shapes such as squares
and triangles as cross-sectional areas. In this case, we have the general formula for volume by integrating a cross-
sectional area:

[
=X ¿ )
3

= X ¿ ? )?
É

EXAMPLES OF SOLIDS OF REVOLUTION: OTHER CROSS-SECTIONAL AREAS

Find the volume of a solid if the base of the solid is the unit circle .
+ ? . = 1 and every perpendicular cross
section is a square.

No. STATEMENT REASON

1 .
+ ?. = 1 Given

2 ^ = 2? The side length of an arbitrary square cross section for


this function

3 ¿ = 4? . Square of the side length is the area of a square

4 ¿ =4 1− .
Substitute ? . with 1 − .

=X 4 1− .
)
volume formula. The limits are from = −1 to = 1
5 Substitute the expression found in statement 4 into the

=4 X 1− .
)
6 Integral Rule 3

@
= 4~ − •
3
7 Integral Rules 2 and 4

1 @ −1 @
= 4 ~j1 − k − j−1 − k•
3 3
8 Fundamental Theorem of Calculus

1 1
= 4 |z1 − { − z−1 + {}
3 3
9 Simplify the right-hand side

2 2
= 4 | − z− {}
3 3
4
= 4| }
3
16
=
3

168
Prepared by: Nathaniel M. Cabansay, BSCpE

DERIVING VOLUME FORMULAS

CYLINDER

Suppose we have a rectangle bounded by the function ? = and the lines = 0 and = ℎ which we want to
revolve around the x-axis, we get the volume of a cylinder as follows:

No. STATEMENT REASON

1 ?= , = 0, =ℎ Given
Ó
=X ¡ .
)
2 We want to revolve the curve around the x-axis, so the
q
radius of each disk is equal to

=¡ .
ℎ−0 ¡ .

=’
3 is constant, so we use:
G [
X ) = \−0
∎ 3

CONE

Suppose we have a triangle enclosed by the origin and points ℎ, 0 and ℎ,


triangle is bounded by the lines ? = 0, ? = ℎ, and ? =
. We can then deduce that the

Ó
from the slope formula.

No. STATEMENT REASON

?= , ? = ℎ, ? = 0

1 Given

Ó .
=X ¡ .
) Ó
ℎ.
2 The radius of each disk is equal to and we want to
q revolve the curve around the x-axis

¡ . @
Ó
= . ~ •
ℎ 3 q
3 Integral Rules 2 and 3

¡ . ℎ@
= ~ •
ℎ. 3
4 Fundamental Theorem of Calculus

P
= ’ G
E
5 Simplify the right-hand side

PRISM

Suppose we have a generic polygon with an area of ¿ and we are to extend it to the height of ℎ, we can use the
formula for volumes with known cross-sections to get the following formula:

No. STATEMENT REASON

1 ¿, = 0, =ℎ Given

169
Prepared by: Nathaniel M. Cabansay, BSCpE

Ó
=X ¿)
2 We already have the area needed, and this stays
q
constant throughout

= ¿ is constant, so we use:

3
[
X ) = \−0
3

Note that a cylinder is a sub-case of prism here, as we can just put the area of the circle in for ¿ and we get the
familiar formula = ¡ . ℎ. For a cube, we put the area of the square in for ¿ and the side length ^ in for ℎ and we
get = ^. ^ = ^@ .

PYRAMID

Suppose we have a regular polygon with an area of ¿ =


.
sides, ^ is the side length, and is the apothem or distance from the center to the
, where is the number of

midpoint of one side that varies from 0 to ℎ.

We can then rewrite ^ using . = tan M Q then solving for ^ we get ^ = 2 tan M Q,
£ £

leaving our final equation for area as ¿ = tan M Q


. £

Then, suppose we have a triangle enclosed by the origin and the points ℎ, 0
and ℎ, ¢ , we have the line ? =
Ó

No. STATEMENT REASON


¡ ¢
¿= .
tan M Q , = 0, ? =

1 Given

Ó
¢ . ¡
=X z { tan M Q )

2 We already have the area needed.
q

¢. ¡ Ó Use Integral Rule 3 (tan M Q is constant)


£
= tan M QX .
)
ℎ.
3
q

¢. ¡ @ Ó
= tan M Q ~ •
ℎ. 3 q
4 Integral Rule 2

¢. ¡ ℎ@
= tan M Qj k
ℎ. 3
5 Fundamental Theorem of Calculus

¡ ℎ
= ¢ . tan M Q z {
3
6 Rewrite the right-hand side

Area of a polygon with an apothem of ¢


=
E
7

170
Prepared by: Nathaniel M. Cabansay, BSCpE

Note that a cone is a sub-case of pyramid here, as we can just put in the area of the circle in for ¿ and we get the
=
£ Ó
@
familiar formula .

FRUSTUM OF A PYRAMID OR CONE

Basically, we just subtract a smaller pyramid or cone from a larger one.

We can use method of cylindrical shells to solve for the volume.

We have ? = ! − , ? = 0, ? = ℎ as the boundaries.


Ó

No. STATEMENT REASON

1 ¿ = 2¡ ℎ, ? = 0, ? = ℎ, = , =þ Given

¿ = 2¡ z! − {
þ− height of each shell is ! −
Ó
2 The radius of each shell is and the

2¡ℎ
¿ = 2¡! − .
þ−
2¡ℎ
=¡ .
ℎ + X 2¡! − .
)
þ− with limits from to þ to add to an
3 We are to rotate this about the y-axis

2¡ℎ
=¡ .
ℎ + 2¡! X ) − X .
) and a height ℎ
already-existing cylinder with a radius
þ−
.
2¡ℎ @
= ¡ . ℎ + 2¡! ~ • − ~ •
2 þ− 3
4 Integral Rule 2

þ. .
2¡ℎ þ @ @
=¡ .
ℎ + 2¡! ~ − • − ~ − •
2 2 þ− 3 3
5 Fundamental Theorem of Calculus

2¡ℎ
= ¡ . ℎ + ¡! þ. − .
− þ@ − @
3 þ−
6 Rewrite the right-hand side

2¡ℎ
= ¡ . ℎ + ¡! þ − þ+ − þ− þ. + þ + .
3 þ−
7 Difference of two squares and difference
of two cubes
¡þℎ 2¡ℎ
= ¡ .ℎ + þ− þ+ − þ− þ. + þ + .
þ− 3 þ−
Solve for ! using similar triangles:

8

=
!
þ þ−
2¡ℎ
= ¡ . ℎ + ¡þℎ þ + − þ. + þ + .
3
9 Simplify the right-hand side

2¡þ . ℎ 2¡ þℎ 2¡ . ℎ
= ¡ . ℎ + ¡þ. ℎ + ¡ þℎ − − −
3 3 3
10 Multiply out all terms

¡ .
ℎ ¡ þℎ ¡þ. ℎ
= + +
3 3 3
11 Combine like terms

’ £Ó
= G
+ + G
E
@
12 Factor out

171
Prepared by: Nathaniel M. Cabansay, BSCpE

To generalize this to a pyramid, we note that ¿ = ¢ . tan M Q for a polygon of apothem ¢, and the volume of a
£

prism of height ℎ with area ¿ is = ¢ . tan M Q ℎ. We then get its derivative with respect to ¢ to get a surface
£

area: ¿ = 2 ¢ tan M Q ℎ
£

¡
No. STATEMENT REASON

¿ = 2 ¢ tan M Q ℎ, ? = 0, ? = ℎ, = ¢, =" =¢
and the larger apothem = ".
1 Given. We let the smaller apothem

¡ ℎ
¿ = 2 tan M Q z! − {
"−¢ height of each shell is ! −
Ó
2 The apothem of each shell is and the

¡
¡ 2 ¢ tan M Q ℎ
#

¿ = 2 tan M Q ! − .
"−¢
¡
¡ ¡ 2 ¢ tan M Q ℎ
= ¢ . tan M Q ℎ + X 2 tan M Q ! − .
) with limits from ¢ to " to add to the
# We are to revolve this about the y-axis
"−¢
3

¡ already-existing prism with an apothem ¢


2 ¢ tan M Q ℎ #
¡ ¡
= ¢ . tan M Q ℎ + 2 tan M Q ! X ) − X .
) and a height ℎ
#

"−¢
¡
¡ ¡ . # 2 ¢ tan M Q ℎ @ #
= ¢. tan M Q ℎ + 2 tan M Q ! ~ • − ~ •
2 "−¢ 3
4 Integral Rule 2

¡ ¡ ". ¢. 2 ℎ "@ ¢@
= ¢ . tan M Q ℎ + 2 tan M Q ! ~ − • − ~ − •
2 2 "−¢ 3 3
5 Fundamental Theorem of Calculus

¡
¡ ¡ 2 tan M Q ℎ
= ¢ . tan M Q ℎ + tan M Q ! " . − ¢ . − "@ − ¢@
3 "−¢
6 Rewrite the right-hand side

¡ ¡
= ¢ . tan M Q ℎ + tan M Q ! " − ¢ " + ¢
¡
7 Difference of two squares and difference
2 tan M Q ℎ
− " − ¢ " . + ¢" + ¢ .
of two cubes
3 "−¢
¡ ¡ "ℎ
= ¢ . tan M Q ℎ + tan M Q "−¢ "+¢
"−¢
Solve for ! using similar triangles:
¡ ℎ
8
2 tan M Q ℎ
=
!

3 "−¢
" − ¢ " . + ¢" + ¢ .
" "−¢
¡ ¡
= ¢ . tan M Q ℎ + tan M Q "ℎ " + ¢
¡
9 Simplify the right-hand side

2 tan M Q ℎ
− " . + ¢" + ¢ .
3
¡
¡ ¡ ¡ 2 tan M Q " . ℎ
= ¢ . tan M Q ℎ + tan M Q " . ℎ + tan M Q ¢"ℎ −
3
10 Multiply out all terms
¡ ¡
2 tan M Q ¢"ℎ 2 tan M Q ¢ . ℎ
− −
3 3
¡ ¡ ¡
tan M Q ¢ . ℎ tan M Q ¢"ℎ tan M Q " . ℎ
= + +
11 Combine like terms
3 3 3
ℎ ¡ ¡ ¡ Ó
= M tan M Q ¢ . + tan M Q ¢" + tan M Q " . Q
3
@
12 Factor out

172
Prepared by: Nathaniel M. Cabansay, BSCpE

Set ¿ = tan M Q ¢ . and ¿. =


£
= +7 +
E P P G G
13

tan M Q " . . Get the square roots and


£

multiply together. 7¿ = ¢œ tan M Q ,
£

then 7¿. = "œ tan M Q and 7¿ ¿. =


£

tan M Q ¢"
£

SPHERE

Suppose we have the area bounded ? = √ . − . and ? = 0 which we want to revolve around the x-axis. We get
the volume of a sphere as follows:

No. STATEMENT REASON

1 ?=7 . − . Given
.
= X ¡ M7 . − .Q )
function about the x-axis, with the intersections at −
2 This is already the radius of a disk when we revolve the

and

= ¡X .
− .
)
3 Simplify the integrand and use Integral Rule 3

.
− .
= 2¡ X .
− .
)
4 is even, so we can use:
q 3 3
X ) = 2X )
3 q

@
= 2¡ ~ .
− •
3
5 Integral Rules 2 and 4
q
@
= 2¡ ~ @
− •
3
6 Fundamental Theorem of Calculus

2 @
= 2¡ ~ •
3
7 Simplify the right-hand side

N
= ’ E
E

SPHEROID

Suppose we have the equation of an ellipse ? = œ\ . −


[ .
3
and we are to revolve it about the x-axis. We get the
volume of a spheroid as follows.

173
Prepared by: Nathaniel M. Cabansay, BSCpE

No. STATEMENT REASON

\.
? = Ÿ\ . − .
1 Given

0.
.
3
\.
= X ¡ ª Ÿ\ . − . .« )
2 This is already the radius of a disk when we revolve the

0
function about the x-axis
3

3
.
= ¡\ X 1 −
.
)
0
3 Simplify the integrand and use Integral Rule 3
.
3
3
.
The function 1 −
= 2¡\ X 1 − .
) 3
0.
4 is even, so we can use:
3 3
q
X ) = 2X )
3 q

@ 3
= 2¡\ ~ − .

30.
5 Integral Rules 2 and 4
q

0@
= 2¡\ . ~0 − •
30.
6 Fundamental Theorem of Calculus

0
= 2¡\ . 0 −
3
20
7 Simplify the right-hand side

= 2¡\ . | }
3
N
= ’‚ÍG
E

ELLIPSOID

+ + = 1 and the equation of an ellipse + = 1. We then


† $ †
3 [ É 3 [
Suppose we have the equation of an ellipsoid

use a double integral to convert _ = œ1 − 3 − [ , but that is just half the ellipsoid, so we multiply this by 2, and

integrate with respect to ? from ? = \ œ1 −


3
then integrate the result with respect to

No. STATEMENT REASON

. ?. .
_ = Ÿ1 − − , ? = \ Ÿ1 −
1 Given

0. \ . 0.

3 [œ . ?.
=X X 2 Ÿ1 − − )? )
3
2 The integral. Note that this is just the same as

0 . \.
[œ integral (with respect to _) already completed
deriving the volume with a triple integral with one
3
3

174
Prepared by: Nathaniel M. Cabansay, BSCpE

3 [œ . ?. Hold 1 − constant first and pull 2 out of the


=8 X X Ÿ1 − − )? )
3 3
3

q q 0. \. integrals. All of these functions are even, so we use

3 3
the property:
X ) = 2X )
3 q

.
\ . z1 − { − ?.
¯ 0.
4 We rewrite the radical first so that we can combine
3 [œ
=8 X X )? )
3
\.
into one fraction

q q

8 3 [œ \. . Pull out the œ


= X X Ÿ\ . − − ? . )? )
3
5
[
\ q q 0.

8 3 v .
= X X 7] − ? . )? ) Substitute ] = \ œ1 − so we get ] . = \ . M1 − 3 Q
\ q q
6
3

Integrate by trigonometric substitution: Let ? =


£
8 3 . .
= X ] X cos . ` )` ) ] sin ` so we get )? = ] cos ` )`, 7] . − ? . =
\ q
7
q
] cos `, and ` = arcsin M Q. We can then substitute
v
the limit ` ] = arcsin M Q = . Move the ] . out of
v £
v .
ONE integral (as we still need to substitute back)
8 3 . ¡ 0−1 ‼ 2−1 ‼
= X ] ~ •)
\ q 2 0+2 ‼ now met, so we use Wallis’ formula with Ë = 0 and
8 The conditions for Wallis’ formula to be used are

= 2. Since both are even, ˜ =


£
.

8 ¡ 1
3
Definition of double factorial. −1 ‼ = 1 and 1‼ =
= X ]. | ⋅ } )
\ q 2 2 1
9

8 3 . ¡
= X ] )
\ q 4
10 Simplify the integrand and use Integral Rule 3

3

= X ]. )
\
q

2¡ 3 .
Substitute back ] . = \ . M1 − Q
= X \ . j1 − . k )
\ q 0
3
11

3 .
= 2¡\ X 1 − )
0.
12 Integral Rule 3
q

@ 3
= 2¡\ ~ − •
30.
13 Integral Rules 2 and 4
q

0 @
= 2¡\ ~0 − •
30.
14 Fundamental Theorem of Calculus

0
= 2¡\ 0−
3
20
15 Simplify the right-hand side

= 2¡\ | }
3

175
Prepared by: Nathaniel M. Cabansay, BSCpE

N
= ’‚Íe
E

PARABOLOID

Suppose we have the area bounded by the half-parabola ? = œ 3 = 0 which we want to revolve around
[
and
the x-axis, we get the volume as follows:

No. STATEMENT REASON

\.
?=Ÿ , =0
1 Given

2 ¿=¡ .
We use the method of disks for the area

3
\.
= X ¡j k)
0
3 Volume formula
q

¡\ . 3
= X )
0 q
4 Simplify the integrand and use Integral Rule 3

¡\ . .
3
= ~ •
0 2 q
5 Integral Rule 2

¡\ . 0.
= ~ •
0 2
6 Fundamental Theorem of Calculus

P G
= ’Í ‚
G
7 Simplify the right-hand side

TORUS

Suppose we want to find the volume of the torus with outer radius þ and inner

−þ + ?. = −þ = − ?., −þ =
radius similar to shown on the right, we get the equation of a circle:
. . . .

±7 − ?., =þ±7 − ?. .
. We then solve for .
. .

No. STATEMENT REASON

1 =þ±7 . − ?. Given
. .
2 ¿ = ¡ |Mþ + 7 . − ? . Q − Mþ − 7 . − ?.Q } Cross-sectional area

176
Prepared by: Nathaniel M. Cabansay, BSCpE

¿ = ¡ Mþ . + 2þ7 . − ?. + .
− ?.Q
− Mþ. − 2þ7 . − ?. + .
− ?.Q

¿ = 4¡þ7 . − ?.

= X 4¡þ7 . − ? . )?
respect to ?, adding the cross-sections from
3 The volume is obtained by integrating with

? = − to ? =

4
= 8¡þ X 7 . − ? . )? Use Integral Rule 3. Since 7 . − ? . is an even
q function, we can use:
3 3
X ) = 2X )
3 q
£
.
= 8¡þ X .
cos . ` )` ? = sin `, )? = cos ` )` and 7 . − ? . =
5 Integrate by trigonometric substitution: Let

q
cos `. Then, let ` = arcsin M Q, so ` =

arcsin M Q =
£
.
£
.
= 8¡þ .
X cos . ` )`
6 Use Integral Rule 3

¡ 0−1 ‼ 2−1 ‼
= 8¡þ .
M Qj k
2 0+2 ‼ are met, so we use Wallis’ Formula with Ë =
7 The conditions for Wallis’ Formula to be used

0 and = 2. Since both are even, ˜ =


£
.

¡ 1 Simplify the right-hand side. −1 ‼ = 1 and


= 8¡þ M Qz { .
2 2 1‼ = 1
8

¡
= 8¡þ . M Q
4
= G’G G = G’ ’ G

177
Prepared by: Nathaniel M. Cabansay, BSCpE

MODULE 23 ARC LENGTH

ARC LENGTH

given the radius and the angle, or ^ = `. Here, we’ll use a different sense of arc length. The arc length of a curve
You may have heard of the term “arc length” in your previous trigonometry classes, which is the length of the arc

is the length of a curve from 0 to \ or a curve ? from to ).

We derive the formula for arc length as follows:

No. STATEMENT REASON

?= , 0, \
are continuous over the interval 0, \
1 Given. We assume that the function and its derivative

\−0
Δ =
2 We will need to estimate the length of the curve by

Δ .
dividing the interval into sub-intervals each of width

3 In each sub-interval, we can draw line segments (in


black) between two points on the function (in green)

|%º %º | = 7 º − º
. + ?º − ?º .
|%º %º |. We can then use the distance formula to find
4 Let the length of each line segment be denoted as

the length of each line segment.

5 |%º %º8 | = 7 Δ . + Δ?º . The difference in the x-coordinates we denote as Δ

the y-coordinates we denote as Δ?º


(from statement 2) and the corresponding difference in

Factor out Δ .
Δ?º .
|%º %º8 | = Ÿ1 + z { Δ
6 from the radical

Δ?º

=
º
Δ
7 Mean Value Theorem for Derivatives

178
Prepared by: Nathaniel M. Cabansay, BSCpE

|%º %º8 | = œ1 + M
.

º Q Δ
8 Substitute statement 7 into the formula in statement 6

.
^ = ¹ œ1 + M ∗
Q Δ
9 We add up the lengths of these line segments
º
º»

.
^ = lim ¹ œ1 + M ∗
Q Δ
10 The length becomes more accurate as more and more
→Å º
º»
segments of shorter subintervals are added.

[
^ = X œ1 + )
11 . We can then use the definition of the definite integral
3

[
)? .
^ = X Ÿ1 + z { )
3 )

If we were to use a function of ?, we instead use:

) .
^ = X Ÿ1 + z { )?
É )?

If we had a parametric curve (a curve where and ? are defined in terms of a third variable, commonly ], called
the parameter), the formula would instead be:

[
)? . ) .
^ = X Ÿz { + z { )]
3 )] )]

Note that we will not always be given the limits of integration right away, we may sometimes have to determine
from the graph or function’s domain.

EXAMPLES OF ARC LENGTH

Determine the length of the parabola ? = .


, −2 ≤ 0 ≤ 2

No. STATEMENT REASON

1 ?= .
, −2 ≤ 0 ≤ 2 Given
)?
=2
)
2 Differentiate both sides with respect to

[
)? .
^ = X Ÿ1 + z { )
3 Arc length formula

3 )

= 2 and the endpoints −2 and 2


. †
^ = X 71 + 2 . )
4 Substitute
.
.
^ = X 71 + 4 . )
5 Square 2x
.

179
Prepared by: Nathaniel M. Cabansay, BSCpE

.
1 1 The integrand is of the form √\ . . + 0. , so our
^ = X Ÿ1 + 4 z tan. ` { sec . ` )` trigonometric substitution here is = tan ` , ` =
3
6

. 4 2 [
arctan M Q, and ) = sec . ` )`, where 0 = 1
[ 3
3 [
and \ = 2

1 .
^ = X 71 + tan. ` sec. ` )`
2 .
7 Simplify the expression inside the radical

1 . Use the Pythagorean identity 1 + tan. ` = sec . `


^= X sec ` sec . ` )`
2 .
8
and simplify the radical

1 1 1 .
Integrate by parts: Let % = sec ` and )> = sec . ` )`,
^= | sec ` tan ` + ln|sec ` + tan `|}
2 2 2 so we have )% = sec ` tan ` and > = tan `.
9
.

sec @ ` we found back in Module 12:


However, we’ll just use the formula we’ve found for

1 1
X sec @ ) = sec tan + ln|sec + tan | + Y
2 2
1 1 .
Substitute ` = arctan 2 , so we get tan ` = 2 ,
^= | 74 . + 1 + ln ‘74 . + 1 + 2 ‘}
2 2 and sec ` = √4 . + 1
10
.

1
^ = ~z274 2 . +1
2
11 Substitute both values into the right-hand side

1
+ ln ‘74 2 . + 1 + 2 2 ‘{
2
− j −2 74 −2 . +1
1
+ ln ‘74 −2 . +1
2
+ 2 −2 ‘k•

1 1
^ = ~z2√17 + ln¥√17 + 4¥{
2 2
12 Evaluate

− j −2 √17
1
+ ln¥√17 − 4¥k•
2
1 1
^= |2√17 + ln¥√17 + 4¥ + 2√17
2 2
1
− ln¥√17 − 4¥}
2
1 1
^ = 2√17 + ln¥√17 + 4¥ − ln¥√17 − 4¥
4 4

P √P' + N
& = G√P' + €• ¦ ¦
N
13 Simplify further
√P' − N

Determine the length of the circular arc ? = √4 − .

180
Prepared by: Nathaniel M. Cabansay, BSCpE

No. STATEMENT REASON

1 ? = 74 − . Given

2 ? = 74 − . , −2 ≤ ≤2 The function is defined only on −2, 2

)?
=−
) √4 −
3 Differentiate both sides with respect to
.

[
)? .
^ = X Ÿ1 + z { )
4 Arc length formula

3 )

. .
^ = X Ÿ1 + z− { )
5 Substitute statement 3 and the endpoints found in

. √4 − .
statement 2 into the integral

. . Square −
^ = X Ÿ1 + )
7r
6

. 4− .

.
4− .+ .
^=X Ÿ )
7 Simplify the expression in the radical

. 4− .

.
4
^=X Ÿ )
. 4− .

.
1
^ = 2X )
. √4 −
8 Simplify the expression in the integral and use Integral
. Rule 3 to move 2 outside the integral.
. ) %
^ = 2 arcsin X = arcsin
2 √0. − 0
9
. .

2 2 Substitute the boundaries into arcsin and subtract


^ = 2 |arcsin z { − arcsin z− {}
2 2
.
10

^ = 2 arcsin 1 − arcsin −1
¡ ¡ arcsin 1 =
£
and arcsin −1 = −
£
^=2 − M− Q
2 2 . .
11

& = G’ Subtract − from


£ £ £ £
. . . .
12 (in effect adding to ) and multiply
by 2.

Prove that the circumference of a circle with radius is 2¡

No. STATEMENT REASON

1 ^ = 2¡ Given

?=7 . − ., − ≤ ≤
solved for ?. The equation for the lower half is ? =
2 Equation of the upper half of a circle with radius when

−√ . − .
)?
=−
) √ −
3 Differentiate both sides with respect to
. .

181
Prepared by: Nathaniel M. Cabansay, BSCpE

[
)? .
^ = X Ÿ1 + z { )
4 Arc length formula

3 )

.
^ = 2 X Ÿ1 + z− { )
5 Substitute statement 3 and the endpoints found in

√ . − .
statement 2 into the integral. Since the equation in
statement 2 was for half the circle, multiply by 2 to get
the circumference of the whole circle.

. Square −
^ = 2 X Ÿ1 + )
7
6
. − .

. − . + .
^ = 2X Ÿ )
7 Simplify the expression in the radical
. − .

.
^ = 2X Ÿ )
. − .

1
^=2 X )
√ −
8 Simplify the expression in the integral and use Integral
. . Rule 3 to move 2 outside the integral.
) %
^=2 arcsin X = arcsin
√0. − 0
9
.

10 ^=2 arcsin M Q − arcsin M− Q Substitute the boundaries into arcsin and subtract

^ = 2 arcsin 1 − arcsin −1
¡ ¡ arcsin 1 =
£
and arcsin −1 = −
£
^=2 − M− Q
2 2 . .
11

^ = 2¡ Subtract − from
£ £ £ £
. . . .

12 (in effect adding to ) and multiply
by 2.

182
Prepared by: Nathaniel M. Cabansay, BSCpE

MODULE 24 SURFACES OF REVOLUTION

SURFACES BY REVOLVING: CIRCLES

Here, we now combine the concepts from solids of revolution (Module 22) and arc length (Module 23) to get
surface areas of solids of revolution. A surface of revolution is obtained by rotating a curve about an axis without
considering the area between it and the axis it is rotated on.

No. STATEMENT REASON

1 , 0, \ Suppose we have a curve and an interval 0, \ and


it is to be rotated about the x-axis
\−0
Δ =
2 If we rotate the curve about an axis, we get a 3D figure

x-coordinate of %q + Δ = x-coordinate of % width Δ . At each subinterval, we denote points


which we can decompose into subintervals or slices of

%q , % , %. and so on to denote points demarcating our


subintervals
(º = 2¡ )
+ .
3 We then use the surface area of a cylinder, or more

=
2 is ( = 2¡ ), then we have = ! and ) = |%º %º |
8
specifically the lateral surface area of a frustum. This area
.
= º (distance between points %º and %º ), then =
= º and . = º
. º

) = |%º %º |
+
(º = 2¡ j k |%º %º |
º º
2

|%º %º | = œ1 + M
.

º Q Δ
4 From our derivation of the arc length formula

º ≈ ∗
º As Δ gets closer and closer to 0, º º



5 and get
∗ º
º º
closer and closer to

œ1 + M
.
(º ≈ 2¡ ∗
º

º Q Δ
6 Substitute everything in statements 4 and 5 into
statement 3.

œ1 + M
.
( ≈ ¹ 2¡ ∗ ∗
Q Δ
7 The total surface area is obtained by summing up all
º º
º»
frusta (plural of frustum)

œ1 + M
.
( = lim ¹ 2¡ ∗ ∗
Q Δ
8 The area becomes more accurate as more and more
º º
→Å
º»
frusta of shorter lengths are added.

[
( = X 2¡ œ1 + )
.
This will work the same way for a function of ? rotated
9 We can then use the definition of the definite integral.
3

( = X 2¡ ? œ1 + ? )?
.
about the y-axis.

É
NOTE: The first formula is used when a function of is


when a function of ? is rotated about the y-axis.
rotated about the x-axis, the second formula is used

183
Prepared by: Nathaniel M. Cabansay, BSCpE

Again, we may sometimes have to determine the limits of integration from the given functions.

Also note that sometimes we are to rotate a function of about the y-axis or a function of ? about the x-axis. In
these instances, we get two additional formulas:

[
( = X 2¡ œ1 + )
.
about the ?-axis
We use this formula when rotating a function of
3
We use this formula when rotating a function of ?
( = X 2¡? œ1 + ? )?
.

É
about the -axis
Again, we may also have to rotate a parametric curve about either axis. We then get two more formulas:

[
)? . ) .
( = X 2¡?Ÿz { + z { )]
We use this formula when rotating a parametric curve

3 )] )]
about the -axis

)? . ) .
( = X 2¡ Ÿz { + z { )] about the ?-axis
We use this formula when rotating a parametric curve

É )] )]

EXAMPLES OF SURFACES OF REVOLUTION

Find the area of the surface obtained by rotating ? = 9 − .


,0≤ ≤ 3 about = 0.

No. STATEMENT REASON

1 ? =9− .
,0 ≤ ≤ 3, =0 Given. We already have everything needed.
)? Get the derivative of ? with respect to .
= −2
)
2

@
( = X 2¡ 71 + −2 . )
3 From the formula for area of a surface of revolution
q
for a function of rotated about the y-axis.
@
Integral Rule 3, then square −2
( = 2¡ X 71 + 4 . )
5
q

¡ @ Let % = 1 + 4 .
, so )% = 8 ) then )% = )
(= X √% )%
4 q
×
6

¡ @ @
(= |% . }
4
7 Integral Rule 2
q

¡ @ @ Substitute back % = 1 + 4 .
(= | 1+4 . .
}
4
8
q

¡ @ @
(= | 37 . − 1 .}
4
9 Fundamental Theorem of Calculus

¡
(= l37√37 − 1m
4
10 Simplify the right-hand side

184
Prepared by: Nathaniel M. Cabansay, BSCpE

No. STATEMENT REASON

1 Given

DERIVING LATERAL SURFACE AREA AND TOTAL SURFACE AREA FORMULAS

Lateral surface area ( refers to the surface area of the solid not counting the area of its base(s).

Total surface area ¿ refers to the surface area of the solid including the area of its base(s).

CYLINDER

Suppose we are to rotate a line bounded by the function ? = , 0 ≤ ≤ ℎ, we get the surface area of a cylinder
as follows:

No. STATEMENT REASON

1 ?= , = 0, =ℎ Given
)? Differentiate both sides of ? =
=0
)
2 with respect to

Ó
( = X 2¡ )
3 We want to revolve the curve around the x-axis, so the
q
radius of each circle is equal to

( = 2¡ ℎ − 0 2¡ is constant, so we use:
* = G’
4
[
X ) = \−0
3

= G’ + G’ G

= G’ + we add 2¡ . to the lateral surface area and simplify the


5 We have two circles for the cylinder’s base and top, so


result.

CONE

Suppose we have a triangle enclosed by the origin and points ℎ, 0 and ℎ,


rotate ? = ,0≤ ≤ ℎ.
. We can then deduce that we are to

No. STATEMENT REASON

?= , ? = ℎ, ? = 0

1 Given

)? Differentiate both sides of ? =


=
) ℎ
Ó
2 with respect to

Ó .
( = X 2¡ M Q Ÿ1 + M Q )
Ó
3 The radius of each circle is equal to and we want to

q ℎ ℎ revolve the curve around the x-axis.

185
Prepared by: Nathaniel M. Cabansay, BSCpE

Ó
2¡ .
Ÿ1 +
Ó
(=X )
4 Square

q ℎ ℎ.
Ó

(=X 7ℎ. + . )
ℎ.
Ó
5 Factor from the radical
q

2¡ √ℎ. + . . Ó
(= ~ •
ℎ. 2
6 Integral Rules 2 and 3

2¡ √ℎ. + . ℎ .
(= ~ •
ℎ. 2
7 Fundamental Theorem of Calculus

*=’ 7 G + G =’ + Simplify the right-hand side. Let slant height ℓ =


√ℎ. + .
8

9 =’ 7 G + G +’ G
=’ ++’ G We have one circle for the cone’s base, so we add ¡ .

= ’ M7 G + G + Q=’ ++
to the lateral surface area, and simplify the resulting


formula.

PRISM

Suppose we have a generic polygon with a perimeter of % and an area of 0 and we are to extend it to the height
of ℎ, we can use the formula for areas of revolutions to get the following formula:

No. STATEMENT REASON

1 %, = 0, =ℎ Given

2 % =0 Differentiate %. Note that % is constant


Ó
(=X %)
3 We already have the area needed, and this stays
q
constant throughout

4 *=- % is constant, so we use:


[
X ) = \−0
3

= - + G‚ We have two polygons of area 0 for the base and the


∎ top, so we add 20 to the lateral surface area.
5

Note that a cylinder is a sub-case of prism here, as we can put in the circumference of the circle in for % and the
area of the circle in for 0 and we get the formula ¿ = 2¡ ℎ + 2¡ . = 2¡ ℎ + . For the cube: We have a
square of side ^ so we have 0 = ^ . , ℎ = ^ and % = 4^, so ( = 4^ ^ = 4^ . , then ¿ = 4^ . + 2^ . = 6^ . .

186
Prepared by: Nathaniel M. Cabansay, BSCpE

PYRAMID

Suppose we have a regular polygon with an area of ¿ = .


and perimeter of % = ^, where
is the number of sides, ^ is the side length, and is the apothem or distance from the
center to the midpoint of one side that varies from 0 to ℎ.

= tan M Q, then solving for ^ we get ^ = 2 tan M Q.


£ £
.
From the figure on the right,

Substitute this into the area and the perimeter and we get ¿ = tan M Q and % =
. £

2 tan M Q, respectively.
£

Then, suppose we have a triangle enclosed by the origin and the points ℎ, 0
and ℎ, ¢ , we have the line ? =
Ó

a regular polygon with apothem ¢.


We can use these to find the surface area of a pyramid with a base consisting of

No. STATEMENT REASON


¡ ¢
%=2 tan M Q , = 0, ? =

1 Given

)? ¢ Differentiate ? with respect to


=
) ℎ
2

We are rotating about the -axis, so we use ( =


Ó
¢ ¡ ¢ .
( = X 2 z { tan M Q Ÿ1 + z { )
3

∫3 % ? œ1 + M Q )
.
ℎ ℎ
[ †
q

Ó2
¡
¢ Mtan M QQ ¢.
(=X Ÿ1 +
Ó
4

ℎ ℎ.
Square

Ó2
¡
¢ Mtan M QQ
(=X 7ℎ. + ¢ .
5 Factor Ó from the radical

q ℎ.
¡
2 ¢ Mtan M QQ √ℎ. + ¢ . . Ó
(= ~ •
6 Integral Rules 2 and 3
ℎ. 2 q

¡
2 ¢ Mtan M QQ √ℎ. + ¢ . ℎ.
(= ~ •
7 Fundamental Theorem of Calculus
ℎ. 2

* = c. M••• M QQ 7 G + .G =
-+
c G height ℓ = √ℎ. + ¢ . and substitute perimeter % =
8 Simplify the right-hand side. We can define slant

2 ¢ Mtan M QQ, but we only have = ¢ tan M Q


£ / £
.

’ ’ We have one polygon with area 0 =


= c. M••• M QQ 7 G + .G + c.G M••• M QQ
c c ¢ . tan M Q =
£
9
/

= +‚= +
-+ -+ -. .
G G G
for the pyramid’s base, so add
that to the lateral surface area, and simplify the
resulting formula.

187
Prepared by: Nathaniel M. Cabansay, BSCpE


= c. M••• M QQ M7 G + .G + .Q = ++.
-
c G

Note that a cone is a special case of this, as we can just substitute % = 2¡ and ¢ = to get ¿ = ¡ ℓ + .

FRUSTUM OF A PYRAMID OR CONE

Basically, we subtract the lateral area of a smaller pyramid or cone from a larger one, and
add the areas of the base and top.

=þ− ?, ? = 0, ? = ℎ as the boundaries.


Ó
We have

No. STATEMENT REASON


þ−
% = 2¡ , ? = 0, ? = ℎ, =þ− ?

1 Given

þ− with respect to ?
=−

2 Differentiate

We are to rotate this about the ?-axis, so


Ó
þ− þ− .
( = X 2¡ jþ − ?k Ÿ1 + z− { )?
3
.
q ℎ ℎ we use ( = ∫É % ? œ1 + M Q )?

Ó
þ− þ− . Square −
( = 2¡ X jþ − ?k Ÿ1 + )?
Ó
pull out the 2¡
4

ℎ ℎ.
and use Integral Rule 3 to

Ó
þ þ−
( = 2¡ X z − . ?{ 7 þ − . + ℎ. )?
ℎ ℎ
Ó
5 Factor from the radical
q

2¡7 þ − . + ℎ. Ó
þ− ? 7 8Ó
(= X jþ − k )?
ℎ ℎ
Ó
6 Factor from the integral
q

2¡7 þ − + ℎ. þ − ?.
. Ó
(= ~þ? − •
ℎ 2ℎ
7 Integral Rules 2, 3, and 4

2¡7 þ − . + ℎ. þ− ℎ
(= ~þℎ − •
ℎ 2
8 Fundamental Theorem of Calculus

2¡7 þ − . + ℎ. þℎ ℎ Simplify þℎ −
Ó
(= |þℎ − + } .
ℎ 2 2
9

2¡7 þ − . + ℎ. þℎ ℎ
(= | + }
ℎ 2 2

* = ’7 − G + G + = ’+ + Simplify the right-hand side. Let ℓ =


7 þ − . + ℎ.
10

188
Prepared by: Nathaniel M. Cabansay, BSCpE

= ’ z7 − G + G + + G
+ G
{ We have two circles – one with area ¡þ .
and one with area ¡ . . Add these to the
11

= ’M+ + + G
+ G
Q

lateral surface area to get the total surface
area of the frustum of a cone

To generalize this to a pyramid, we note that % = 2 ¢ tan M Q for a polygon of apothem ¢.


£

¡
No. STATEMENT REASON

% = 2 ¢ tan M Q , ? = 0, ? = ℎ, = ¢, ="
polygon of the larger base and ¢ be the
1 Given. We let " be the apothem of the

apothem of the polygon of the smaller base


¡ "−¢ The apothem of each shell is " − ?
% = 2 tan M Q z" − ?{

#
Ó
2

We are to rotate this about the ?-axis, so


Ó
¡ "−¢ "−¢ .
( = X 2 tan M Q j" − ?k Ÿ1 + z− { )?
3
.
q ℎ ℎ we use ( = ∫É % ? œ1 + M Q )?

¡ Ó "−¢ "−¢ . Square −


?k Ÿ1 +
#

( = 2 tan M Q X j" − )? Ó
4
ℎ ℎ. pull out the 2 tan M Q
and use Integral Rule 3 to
£
q

¡ Ó " "−¢
( = 2 tan M Q X j − ?k 7 " − ¢ . + ℎ. )?
ℎ ℎ.
Ó
5 Factor from the radical
q

¡
2 tan M Q 7 " − ¢ . + ℎ. Ó
"−¢ ?
7 # 8Ó
(= X j" − k )? Ó
6
ℎ ℎ
Factor from the integral
q

¡
2 tan M Q 7 " − ¢ . + ℎ. " − ¢ ?.
Ó
(= ~"? − •
7 Integral Rules 2, 3, and 4
ℎ 2ℎ q

¡
2 tan M Q 7 " − ¢ . + ℎ. "−¢ ℎ
(= ~"ℎ − •
8 Fundamental Theorem of Calculus
ℎ 2
¡
2 tan M Q 7 " − ¢ + ℎ. Simplify "ℎ −
Ó
"ℎ ¢ℎ
.
(= |"ℎ − + }
#
.
9
ℎ 2 2
¡
2 tan M Q 7 " − ¢ . + ℎ. "ℎ ¢ℎ
(= | + }
ℎ 2 2

* = c ••• M Q 7 0 − . G + G 0+. =
+
- + -G Simplify the right-hand side. Let ℓ =
c G P
7 "−¢ + ℎ. , % = 2 " tan M Q
£
10
.

"] and %. = 2 ¢ tan M Q [perimeter of the


£
[perimeter of the polygon with apothem

polygon with apothem ¢]

189
Prepared by: Nathaniel M. Cabansay, BSCpE

’ ’ We have two polygons with areas 0 =


= c ••• M Q 7 0 − . G +
0 + . + c0G ••• M Q
G
c c
" . tan M Q = and 0. = ¢ . tan M Q =
£ £

11
/! #
+ c.G ••• M Q = -P + -G + ‚P + ‚G .
+
c G /
.
= -P + -G + +
+ -P 0 -G . for the frustum’s bases, so add those to
G G G

the lateral surface area and simplify the
= c ••• M Q M7 0 − . G + G 0 + . + 0G + .G Q
c
resulting formula.

-P + -G
= ++0+.
G

SPHERE

We will use a parametrized curve, specifically the circle, which is parametrized as ? = sin ] and = cos ] and
we want to revolve this circle around the x-axis. We get the surface area of a sphere as follows.

No. STATEMENT REASON

1 ? = sin ] , = cos ] Given

? = cos ] Differentiate both sides with respect to ]


= − sin ]
2

£
¿ = X 2¡ sin ] 7 cos ] . + − sin ] . )]
3 We rotate the curve around the x-axis, so we use

¿ = ∫3 2¡? œM Q + M Q )] and a half-circle


q [ † . .

v v
ranges from ] = 0 to ] = ¡

Square cos ] and − sin ]


£
¿ = X 2¡ sin ] 7 . cos . ] + . sin. ] )]
4
q
£
¿ = X 2¡ sin ] 7 . )]
5 Simplify the radical
q
£
¿ = X 2¡ .
sin ] )]
6 Simplify the integrand
q

Use Integral Rule 3 to pull out 2¡


£ .
¿ = 2¡ .
X sin ] )]
7 from the
q integral

8 ¿ = 2¡ .
− cos ] £
q ∫ sin ] )] = − cos ]. We can ignore the +Y since
we are doing a definite integral.

9 ¿ = 2¡ .
− cos ¡ − − cos 0 Fundamental Theorem of Calculus

10 ¿ = 2¡ .
1+1 cos ¡ = −1 and cos 0 = 1

11 = N’ G
Simplify the right-hand side

190
Prepared by: Nathaniel M. Cabansay, BSCpE

TORUS

Suppose we want to find the surface area of the torus with outer radius þ and

− þ . + ? . = . . We then solve for . − þ . = . − ? . , − þ =


inner radius similar to shown on the right, we get the equation of a circle:

±7 . − ?., =þ±7 . − ?. .

No. STATEMENT REASON

=þ±7 . − ?.
?
1 Given

=∓ Differentiate with respect to ?


7 . − ?.
2

?
.
¿ = X 2¡ Mþ − 7 . − ? . Q Ÿ1 + j k )? respect to the y-axis, so use ( =
3 We are rotating this circle with
7 . − ?.
.
?
.
∫É % ? œ1 + M †Q )? . Split the
+ X 2¡ Mþ + 7 . − ? . Q Ÿ1 + j− k )?
7 . − ?. ±7 . − ? . so we have two
integrals.

Square ∓

?.
¿ = X 2¡ Mþ − 7 − ? . Q Ÿ1 + . )?
. 7 †
4
− ?.

?.
+ X 2¡ Mþ + 7 . − ? . Q Ÿ1 + . − ?.
)?

Use Integral Rule 3 to factor 2¡


?.
¿ = 2¡ ªX Mþ − 7 . − ? . Q Ÿ1 + )?
5
. − ?.
from the integrals

?.
+ X Mþ + 7 . − ? . Q Ÿ1 + . − ?.
)?«

þ
¿ = 2¡ jX j − 1k 7 . − ? . + ? . )? †
7 − ?.
6 Factor from the radical
.

þ
+X j + 1k 7 . − ? . + ? . )? k
7 . − ?.
þ
¿ = 2¡ jX j − 1k 7 . )?
7 − ?.
7 Simplify the radical and use
.

þ
Integral Rule 3 to factor from

+X j + 1k 7 . )?k
the integral
7 . − ?.

191
Prepared by: Nathaniel M. Cabansay, BSCpE

þ þ
¿ = 2¡ jX j − 1k )? + X j + 1k )?k
7 . − ?. 7 . − ?.
þ
¿ = 2¡ ~jX )? − X 1 )?k
7 − ?.
8 Use Integral Rule 4 to split the
.

þ
integral

+ jX )? + X 1 )?k•
7 . − ?.
þ
¿ = 4¡ ~jX )?k•
7 − ?.
9 Simplify the integrals
.

1 Use Integral Rule 3 to factor þ


¿ = 4¡þ jX )?k
7 − ?.
9
.

?
∫7 = arcsin M Q and
/
10 ¿ = 4¡þ z arcsin M Q { 3 / 3
∫1) =
¡ ¡ ¡ ¡
¿ = 4¡þ j − M− Qk arcsin 1 = , arcsin −1 = −
2 2 2 2
11

¿ = 4¡þ ¡
= N’
12 Simplify the right-hand side
G

192
Prepared by: Nathaniel M. Cabansay, BSCpE

REFERENCES

Books and E-books

Ayres, F. & Mendelson, E. (2013). Schaum’s outlines: Calculus (6th ed.). https://pdf.zlibcdn.com/dtoken/

28acdf753fa086d09607191f9c51a1f3/Schaums_Outline_of_Calculus,_6th_Edition_by_Frank_2593091_(z-

lib.org).pdf

Capote, R. & Mandawe, J. (2007). Mathematics & basic engineering sciences. JAM Publisher.

Herman, E. & Strang, G. (2020). Calculus (Vol. 1). OpenStax. http://cnx.org/content/col11964/1.12

Herman, E. & Strang, G. (2020). Calculus (Vol. 2). OpenStax. https://openstax.org/books/calculus-volume-2/pages/1-

introduction

Larson, R. & Edwards, B. (2018). Calculus (11th ed.). Cengage Learning. https://pdf.zlibcdn.com/dtoken/

eea697e8e9318d4317bf64903ba4910f/Calculus_by_Ron_Larson,_Bruce_Edwards)_3403567_(z-lib.org).pdf

Mercado, J. P. & Orines, F. B. (2016). Next century mathematics 11: Basic calculus. Phoenix Publishing House, Inc.

Online Sources

Apothem. (2021, June 27). In Wikipedia. https://en.wikipedia.org/w/index.php?title=Apothem&oldid=1030715866

Chow, S. [blackpenredpen] (2016, February 20). Integration by parts, DI method, VERY EASY [Video]. YouTube.
https://www.youtube.com/watch?v=2I-_SV8cwsw

Chow, S. [blackpenredpen] (2018, January 19). This is how we partial fraction, repeated linear factors, “build up the
power” [Video]. YouTube. https://www.youtube.com/watch?v=pYUTZD1GVyU

Common Integrals. (n.d.). SOS Math. http://www.sosmath.com/tables/integral/integ1/integ1.html

Dawkins, P. (2018, May 30). Area and Volume Formulas. Paul’s Online Notes. https://tutorial.math.lamar.edu/
Classes/CalcI/Area_Volume_Formulas.aspx

Dawkins, P. (2018, May 30). Average Function Value. Paul’s Online Notes. https://tutorial.math.lamar.edu/Classes/
CalcI/AvgFcnValue.aspx

Dawkins, P. (2018, May 30). Differentials. Paul’s Online Notes. https://tutorial.math.lamar.edu/Classes/CalcI/


Differentials.aspx
Prepared by: Nathaniel M. Cabansay, BSCpE

Dawkins, P. (2018, May 30). Integrals Involving Trig Functions. Paul’s Online Notes. https://tutorial.math.lamar.edu/
Classes/CalcII/IntegralsWithTrig.aspx

Dawkins, P. (2018, May 30). More Substitution Rule. Paul’s Online Notes. https://tutorial.math.lamar.edu/Classes/
CalcI/SubstitutionRuleIndefinitePtII.aspx

Dawkins, P. (2018, May 30). More Volume Problems. Paul’s Online Notes. https://tutorial.math.lamar.edu/Classes/
CalcI/MoreVolume.aspx

Dawkins, P. (2018, May 30). Partial Fractions. Paul’s Online Notes. https://tutorial.math.lamar.edu/Classes/CalcII/
PartialFractions.aspx

Dawkins, P. (2018, May 30). Proof of Various Integral Properties. Paul’s Online Notes.
https://tutorial.math.lamar.edu/Classes/CalcI/ProofIntProp.aspx

Dawkins, P. (2018, May 30). Substitution Rule for Definite Integrals. Paul’s Online Notes.
https://tutorial.math.lamar.edu/Classes/CalcI/SubstitutionRuleDefinite.aspx

Dawkins, P. (2018, May 30). Summation Notation. Paul’s Online Notes. https://tutorial.math.lamar.edu/Classes/
CalcI/SummationNotation.aspx

Dawkins, P. (2018, May 30). Surface area. Paul’s Online Notes. https://tutorial.math.lamar.edu/classes/
calcii/SurfaceArea.aspx

Dawkins, P. (2018, May 30). Trigonometric Integrands. Paul’s Online Notes. https://tutorial.math.lamar.edu/Classes/
CalcII/IntegralsWithTrig.aspx

Dawkins, P. (2018, May 30). Volume with Cylinders. Paul’s Online Notes. https://tutorial.math.lamar.edu/Classes/
CalcI/VolumeWithCylinder.aspx

Dawkins, P. (2019, September 16). Area Problem. Paul’s Online Notes. https://tutorial.math.lamar.edu/classes/
calci/areaproblem.aspx

Dawkins, P. (2019, September 23). Computing Definite Integrals. Paul’s Online Notes.
https://tutorial.math.lamar.edu/Classes/CalcI/ComputingDefiniteIntegrals.aspx

Dawkins, P. (2020, January 11). Approximating Definite Integrals. Paul’s Online Notes.
https://tutorial.math.lamar.edu/Classes/CalcII/ApproximatingDefIntegrals.aspx

Dawkins, P. (2020, January 21). Improper Integrals. Paul’s Online Notes. https://tutorial.math.lamar.edu/Classes/
CalcII/ImproperIntegrals.aspx

Dawkins, P. (2020, May 26). Integrals Involving Quadratics. Paul’s Online Notes. https://tutorial.math.lamar.edu/
Classes/CalcII/IntegralsWithQuadratics.aspx

Dawkins, P. (2020, May 26). Substitution Rule for Indefinite Integrals. Paul’s Online Notes. https://
tutorial.math.lamar.edu/Classes/CalcI/SubstitutionRuleIndefinite.aspx
Prepared by: Nathaniel M. Cabansay, BSCpE

Dawkins, P. (2020, May 26). Surface area with parametric equations. Paul’s Online Notes.
https://tutorial.math.lamar.edu/classes/calcii/ParaSurfaceArea.aspx

Dawkins, P. (2020, May 26). Trigonometric Substitutions. Paul’s Online Notes.


https://tutorial.math.lamar.edu/Classes/CalcII/TrigSubstitutions.aspx

Dawkins, P. (2021, March 5). Volume with Rings. Paul’s Online Notes.
https://tutorial.math.lamar.edu/Classes/CalcI/VolumeWithRings.aspx

Double Angle Formulas/Sine/Corollary (2020, March 5). In ProofWiki. https://proofwiki.org/w/index.php?


title=Double_Angle_Formulas/Sine/Corollary&oldid=452789

Fundamental theorem of calculus (2021, January 15). In Wikipedia. https://en.wikipedia.org/w/


index.php?title=Fundamental_theorem_of_calculus&oldid=1000554170

Hyperbolic Tangent Half-Angle Substitution (2020, October 14). In ProofWiki. https://proofwiki.org/w/


index.php?title=Hyperbolic_Tangent_Half-Angle_Substitution&oldid=494648

Hyperbolic Tangent Half-Angle Substitution for Cosine (2020, October 14). In ProofWiki. https://proofwiki.org/
w/index.php?title=Hyperbolic_Tangent_Half-Angle_Substitution_for_Cosine&oldid=494645

Hyperbolic Tangent Half-Angle Substitution for Sine (2020, October 14). In ProofWiki. https://proofwiki.org/
w/index.php?title=Hyperbolic_Tangent_Half-Angle_Substitution_for_Sine&oldid=494642

Inverse hyperbolic functions. (2020, December 29). In Wikipedia. https://en.wikipedia.org/w/index.php?


title=Inverse_hyperbolic_functions&oldid=996882998

Kumar, G. [M4 MATHS]. (2019, November 18). Multiple Integrals Part 16 (Volume).
https://www.youtube.com/watch?v=_NT6fZTJzW0

Partial Fractions. (n.d.). MathsIsFun. https://www.mathsisfun.com/algebra/partial-fractions.html

Primitive of Cosecant Function/Tangent Form. (2020, October 13). In ProofWiki. https://proofwiki.org/w/


index.php?title=Primitive_of_Cosecant_Function/Tangent_Form&oldid=494590

Primitive of Hyperbolic Cosecant Function/Hyperbolic Tangent Form (2020, October 14). In ProofWiki. https://
proofwiki.org/w/index.php?title=Primitive_of_Hyperbolic_Cosecant_Function/Hyperbolic_Tangent_Form
&oldid=494674

Primitive of Hyperbolic Cosecant Function/Inverse Hyperbolic Cotangent Form (2020, September 26). In ProofWiki.
https://proofwiki.org/w/index.php?title=Primitive_of_Hyperbolic_Cosecant_Function/Inverse_Hyperbolic_
Cotangent_Form&oldid=490921

Primitive of Hyperbolic Cosecant Function/Inverse Hyperbolic Cotangent of Hyperbolic Cosine Form (2020, September
26). In ProofWiki. https://proofwiki.org/w/index.php?title=Primitive_of_Hyperbolic_Cosecant_Function/
Inverse_Hyperbolic_Cotangent_of_Hyperbolic_Cosine_Form&oldid=490920
Prepared by: Nathaniel M. Cabansay, BSCpE

Primitive of Hyperbolic Secant Function/Arcsine Form (2020, October 14). In ProofWiki. https://
proofwiki.org/w/index.php?title=Primitive_of_Hyperbolic_Secant_Function/Arcsine_Form&oldid=494675

Primitive of Hyperbolic Secant Function/Arctangent of Exponential Form (2020, October 14). In ProofWiki. https://
proofwiki.org/w/index.php?title=Primitive_of_Hyperbolic_Secant_Function/Arctangent_of_Exponential_F
orm&oldid=494661

Primitive of Hyperbolic Secant Function/Arctangent of Half Hyperbolic Tangent Form (2020, October 14). In ProofWiki.
https://proofwiki.org/w/index.php?title=Primitive_of_Hyperbolic_Secant_Function/Arctangent_of_Half_Hy
perbolic_Tangent_Form&oldid=494663

Primitive of Hyperbolic Secant Function/Arctangent of Hyperbolic Sine Form (2020, October 14). In ProofWiki. https://
proofwiki.org/w/index.php?title=Primitive_of_Hyperbolic_Secant_Function/Arctangent_of_Hyperbolic_Sin
e_Form&oldid=494635

Primitive of Inverse Hyperbolic Cosecant of x over a (2021, January 6). In ProofWiki. https://proofwiki.org/w/
index.php?title=Primitive_of_Inverse_Hyperbolic_Cosecant_of_x_over_a&oldid=505093

Primitive of Inverse Hyperbolic Cosine of x over a (2021, January 6). In ProofWiki. https://proofwiki.org/w/
index.php?title=Primitive_of_Inverse_Hyperbolic_Cosine_of_x_over_a&oldid=505058

Primitive of Inverse Hyperbolic Cotangent of x over a (2021, January 3). In ProofWiki. https://proofwiki.org/w/
index.php?title=Primitive_of_Inverse_Hyperbolic_Cotangent_of_x_over_a&oldid=504932

Primitive of Inverse Hyperbolic Secant of x over a (2021, January 6). In ProofWiki. https://proofwiki.org/w/
index.php?title=Primitive_of_Inverse_Hyperbolic_Secant_of_x_over_a&oldid=505077

Primitive of Inverse Hyperbolic Sine of x over a (2021, January 6). In ProofWiki. https://proofwiki.org/w/
index.php?title=Primitive_of_Inverse_Hyperbolic_Sine_of_x_over_a&oldid=505105

Primitive of Inverse Hyperbolic Tangent of x over a (2021, January 3). In ProofWiki. https://proofwiki.org/w/
index.php?title=Primitive_of_Inverse_Hyperbolic_Tangent_of_x_over_a&oldid=504922

Primitive of Secant Function/Tangent plus Angle Form (2020, October 13). In ProofWiki. https://proofwiki.org/w/
index.php?title=Primitive_of_Secant_Function/Tangent_plus_Angle_Form&oldid=494619

Properties of the Definite Integral (n.d.). SOS Math. http://www.sosmath.com/calculus/integ/integ02/integ02.html

Regular polygon (2022, January 21). In Wikipedia. https://en.wikipedia.org/w/index.php?title=Regular_polygon


&oldid=1067106260

Solids of Revolution by Disks (n.d.). MathsIsFun. https://www.mathsisfun.com/calculus/solids-revolution-disk-


washer.html

Solids of Revolution by Shells (n.d.). MathsIsFun. https://www.mathsisfun.com/calculus/solids-revolution-shells.html

Spheroid (2022, January 6). In Wikipedia. https://en.wikipedia.org/w/index.php?title=Spheroid&oldid=1064029406


Prepared by: Nathaniel M. Cabansay, BSCpE

Surface Area of a Sphere (n.d.). Brilliant.org. https://brilliant.org/wiki/surface-area-sphere/

Svirin, A. (2021). Average Value of a Function. Math24. https://www.math24.net/average-value-function

Svirin, A. (2021). Trigonometric Integrals. Math24. https://www.math24.net/trigonometric-integrals/

Svirin, A. (2021). Volume of a Solid of Revolution: Cylindrical Shells. Math24. https://www.math24.net/volume-solid-of-


revolution-cylindrical-shells

Svirin, A. (2021). Weierstrass Substitution. Math24. https://www.math24.net/weierstrass-substitution

Tangent Half-Angle Substitution for Cosine (2020, March 5). In ProofWiki. https://proofwiki.org/w/index.php?
title=Tangent_Half-Angle_Substitution_for_Cosine&oldid=452788

Tangent of Half Angle plus Quarter Pi (2020, March 17). In ProofWiki. https://proofwiki.org/w/
index.php?title=Tangent_of_Half_Angle_plus_Quarter_Pi&oldid=455071

Torus (n.d.). MathsIsFun. https://www.mathsisfun.com/geometry/torus.html

Weierstrass Substitution (2020, December 24). In ProofWiki. https://proofwiki.org/w/index.php?


title=Weierstrass_Substitution&oldid=503307

Weisstein, E. W. (n.d.). Conical frustum. MatthWorld – A Wolfram Web Resource.


https://mathworld.wolfram.com/ConicalFrustum.html

Weisstein, E. W. (n.d.). Double Factorial. MathWorld – A Wolfram Web Resource.


https://mathworld.wolfram.com/DoubleFactorial.html

Weisstein, E. W. (n.d.). Pyramidal frustum. MathWorld – A Wolfram Web Resource.


https://mathworld.wolfram.com/PyramidalFrustum.html
Prepared by: Nathaniel M. Cabansay, BSCpE

EXTRAS

ABOUT THIS SET OF MODULES

This set of modules deals with integrals, the third set of Calculus-related topics dealt with in many Calculus
courses, be it in Engineering, Computer Science, Physics, or other courses requiring Calculus. This concept is
needed in further topics such as differential equations, which will in turn have more applications in fields such as
Engineering, Sciences, Computer Science, and other courses.

SOME QUOTES TO LIVE BY

“Don’t judge the book by its cover, but look inside and discover.” -Lorraine Leigh Lacumba (Former MNL48 Lei)

“Give a man a fish, he will eat for a day. Teach a man to fish, he will eat every day.” – Unknown origin

“It is not of the essence of mathematics to be conversant with the ideas of number and quantity.” – George Boole

“Don’t walk behind me; I may not lead. Don’t walk in front of me; I may not follow. Just walk beside me and be my
friend.” – Albert Camus

You might also like